user8's blog

IBPS (PO) Previous Year Exam Paper - 2012 "English Language"

IBPS logo


IBPS (PO) Previous Year Exam Paper - 2012

Subject: Computer Knowledge


151. _________ allows users to upload files to an online site so they can be viewed and edited from another location.

(1) General-purpose applications
(2) Microsoft outlook
(3) Web-hosted technology
(4) Office Live
(5) None of these

152. What feature adjusts the top and bottom margins so that the text is centered vertically on the printed page?

(1) Vertical justifying
(2) Vertical adjusting
(3) Dual centering
(4) Horizontal centering
(5) Vertical centering

153. Which of these is not a means of personal communication on the Internet?

(1) chat
(2) instant messaging
(3) instanotes
(4) electronic mail
(5) None of these

154. What is the overall term for creating, editing, formatting, storing, retrieving, and printing a text document?

(1) Word processing
(2) Spreadsheet design
(3) Web design
(4) Database management
(5) Presentation generation

155. Fourth-generation mobile technology provides enhanced capabilities allowing the transfer of both data, including full-motion video, high-speed Internet access, and videoconferencing.

(1) video data and information
(2) voice and non-voice
(3) music and video
(4) video and audio
(5) None of these

156. _________ is a form of denial of service attack in which a hostile client repeatedly  attack in which a hostile client repeatedly sends SYN packets to every port on the server, using fake IP addresses.

(1) Cybergaming crime
(2) Memory shaving
(3) Syn flooding
(4) Software piracy
(5) None of these

157. Which of these is a point-and-draw device?

(1) mouse
(2) scanner
(3) printer
(4) CD-ROM
(5) keyboard

158. The letter and number of the intersecting column and row is the

(1) cell location
(2) cell position
(3) cell address
(4) cell coordinates
(5) cell contents

159. A set of rules for telling the computer what operations to perform is called a

(1) procedural language
(2) structures
(3) natural language
(4) command language
(5) programming language

160. A detailed written description of the programming cycle and the program, along with the test results and a printout of the program is called

(1) documentation
(2) output
(3) reporting
(4) spec sheets
(5) directory

161. Forms that are used to organise business data into rows and columns are called

(1) transaction sheets
(2) registers
(3) business forms
(4) sheet-spreads
(5) spreadsheets

162. In PowerPoint, the Header & Footer button can be found on the Insert tab in what group?

(1) Illustrations group
(2) Object group
(3) Text group
(4) Tables group (5) None of these

163. A(n)_____ is a set of programs designed to manage the resources of a computer, including starting the computer, managing programs, managing memory, and coordin-ating tasks between input and output devices.

(1) application suite
(2) compiler
(3) input/output system
(4) interface
(5) operating system (OS)

164. A typical slide in a slide presentation would not include

(1) photo images, charts and graphs
(2) graphs and clip art
(3) clip art and audio clips
(4) full-motion video
(5) content templates

165. The PC productivity tool that manipulates data organised in rows and columns is called a(n)

(1) spreadsheet
(2) word processing document
(3) presentation mechanism
(4) database record manager
(5) EDI creator

166. In the absence of parentheses, the order of operation is

(1) exponentiation, addition or subtraction, multiplication or division
(2) addition or subtraction, multiplication or division, exponentiation
(3) multiplication or division, exponen-tiation, addition or subtraction
(4) exponentiation, multiplication or divi-sion, addition or subtraction
(5) addition or subtraction, exponentiation, multiplication or division

167. To find the Paste Special option, you use the Clipboard group on the _______ tab of PowerPoint.

(1) design
(2) slide Show
(3) page layout
(4) insert
(5) home

168. A(n) program is one that is ready to run and does not need to be altered in any way.

(1) interpreter
(2) high-level
(3) compiler
(4) COBOL
(5) executable

169. Usually downloaded into folders that hold temporary Internet files,_________ are written to your computer’s hard disk by some of the Web sites you visit.

(1) anonymous files
(2) behaviour files
(3) banner ads
(4) large files
(5) cookies

170. What is the easiest way to change the phrase “revenues, profits, gross margin,” to read “revenues, profits, and gross margin”?

(1) Use the insert mode, position the cursor before the g in gross, then type the word and followed by a space
(2) Use the insert mode, position the cursor after the g in gross, then type the word and followed by a space
(3) Use the overtype mode, position the cursor before the g in gross, then type the word and followed by a space
(4) Use the overtype mode, position the cursor after the g in gross, then type the word and followed by a space
(5) None of these

171. A program, either talk or music, that is made available in digital format for automatic download over the Internet is called a

(1) wiki
(2) broadcast
(3) vodcast
(4) bIog
(5) podcast

172. Which PowerPoint view displays each slide of the presentation as a thumbnail and is useful for rearranging slides?

(1) Slide sorter
(2) Slide show
(3) Slide master
(4) Notes page
(5) Slide design

173. Different components on the motherboard of a PC unit are linked together by sets of parallel electrical conducting lines. What are these lines called?

(1) Conductors
(2) Buses
(3) Connectors
(4) Consecutives
(5) None of these

174. What is the name given to those applications that combine text, sound, graphics, motion video, and/or animation?

(1) motionware
(2) anigraphics
(3) videoscapes
(4) multimedia
(5) maxomedia

175. A USB communication device that supports data encryption for secure wireless commu-nication for notebook users is called a

(1) USB wireless network adapter
(2) wireless switch
(3) wireless hub
(4) router
(5) None of these

DOWNLOAD IBPS PO Question Papers PDF

DOWNLOAD IBPS CLERK Question Papers PDF

DOWNLOAD ALL BANK EXAMS PAPERS E-BOOKS

Printed Study Material for IBPS PO Exam

176. A(n) language reflects the way people think mathematically.

(1) cross-platform programming
(2) 3GL business programming
(3) event-driven programming
(4) functional
(5) None of these

177. When entering text within a document, the Enter key is normally pressed at the end of every

(1) line
(2) sentence
(3) paragraph
(4) word
(5) file

178. When a real-time telephone call between people is made over the Internet using computers, it is called

(1) a chat session
(2) an e-mail
(3) an instant message
(4) internet telephony
(5) None of these

179. Which of the following is the first step in sizing a window?

(1) Point to the title bar
(2) Pull down the View menu to display the toolbar
(3) Point to any corner or border
(4) Pull down the View menu and change to large icons
(5) None of these

180. Which of the following software could assist someone who cannot use their hands for computer input?

(1) Videoconferencing
(2) Speech recognition
(3) Audio digitizer
(4) Synthesizer
(5) None of these

181. ......... a document means the file is transferred from another computer to your computer.

(1) Uploading
(2) Really Simple Syndication (RSS)
(3) Accessing
(4) Downloading
(5) Upgrading

182. Which computer memory is used for storing programs and data currently being processed by the CPU?

(1) Mass memory
(2) Internal memory
(3) Non-volatile memory
(4) PROM
(5) None of these

183. Computers that control processes accept data in a continuous

(1) data traffic pattern
(2) data highway
(3) infinite loop
(4) feedback loop
(5) slot

184. What refers to a set of characters of a particular design?

(1) Keyface
(2) Formation
(3) Calligraph
(4) Stencil
(5) Typeface

185. ........ is used by public and private enterprises to publish and share financial information with each other and industry analysts across all computer platforms and the Internet.

(1) Extensible Markup Language (XML)
(2) Extensible Business Reporting Language (XBRL)
(3) Enterprise Application Integration (EAI)
(4) Sales Force Automation (SKA) software
(5) None of these

186. Which part of the computer is used for calculating and comparing?

(1) ALU
(2) Control unit
(3) Disk unit
(4) Modem
(5) None of these

187. The method of Internet access that requires a phone line, but offers faster access speeds than dial-up is the connection.

(1) cable access
(2) satellite access
(3) fibre-optic service
(4) digital subscriber line (DSL)
(5) modem

188. ............. software creates a minor image of the entire hard disk, including the operating system, applications, files, and data.

(1) Operating system
(2) Backup software
(3) Utility programs
(4) Driver imaging
(5) None of these

189. What is a URL?

(1) A computer software program
(2) A type of programming object
(3) The address of a document or “page” on the World Wide Web
(4) An acronym for Unlimited Resources for Learning
(5) A piece of hard ware

190. What is the significance of a faded (dimmed) command in a pull-down menu?

(1) The command is not currently accessible.
(2) A dialog box appears if the command is selected.
(3) A Help window appears if the command is selected.
(4) There are no equivalent keystrokes for the particular command.
(5) None of these

191. Your business has contracted with another company to have thorn host and run an application for your company over the Internet. The company providing this service to your business is called an

(1) internet service provider
(2) internet access provider
(3) application service provider
(4) application access provider
(5) outsource agency

192. A(n) ............. allows you to access your e-mail from anywhere.

(1) forum
(2) webmail interface
(3) message board
(4) weblog
(5) None of these

193. Which of the following would you find on Linkedln?

(1) Games
(2) Connections
(3) Chat
(4) Applications
(5) None of these

194. ............. is a technique that is used to send more than one call over a single line.

(1) Digital transmission
(2) Infrared transmission
(3) Digitizing
(4) Streaming
(5) Multiplexing

195. The Search Companion can

(1) locate all files containing a specified phrase
(2) restrict its search to a specified set of folders
(3) locate all files containing a specified phrase and restrict its search to a specified set of folders
(4) cannot locate all files containing a specified phrase or restrict its search to a specified set of folders
(5) None of these

196. Which of the following cannot be part of an email address?

(1) Period (.)
(2) At sign (@)
(3) Space ()
(4) Underscore (_)
(5) None of these

197. Which of the following must be contained in a URL?

(1) A protocol identifier
(2) The letters www
(3) The unique registered domain name
(4) www. and the unique registered domain name
(5) A protocol identifier, www. and the unique registered domain name

198. Which of the following information systems focuses on making manufacturing processes more efficient and of higher quality?

(1) Computer-aided manufacturing
(2) Computer-integrated manufacturing
(3) Computer-aided software engineering
(4) Computer-aided system engineering
(5) None of these

199. A mistake in an algorithm that causes incorrect results is called a

(1) logical error
(2) syntax error
(3) procedural error
(4) compiler error
(5) machine error

200. A device for changing the connection on a connector to a different configuration is

(1) a converter
(2) a component
(3) an attachment
(4) an adapter
(5) voltmeter

DOWNLOAD IBPS PO Question Papers PDF

DOWNLOAD IBPS CLERK Question Papers PDF

DOWNLOAD ALL BANK EXAMS PAPERS E-BOOKS

Printed Study Material for IBPS PO Exam

IBPS / SBI Special TX: 
General: 
STUDY KITS: 

IBPS (PO) Previous Year Exam Paper - 2012 "Computer Knowledge"

IBPS logo


IBPS (PO) Previous Year Exam Paper - 2012

Subject: Computer Knowledge


151. _________ allows users to upload files to an online site so they can be viewed and edited from another location.

(1) General-purpose applications
(2) Microsoft outlook
(3) Web-hosted technology
(4) Office Live
(5) None of these

152. What feature adjusts the top and bottom margins so that the text is centered vertically on the printed page?

(1) Vertical justifying
(2) Vertical adjusting
(3) Dual centering
(4) Horizontal centering
(5) Vertical centering

153. Which of these is not a means of personal communication on the Internet?

(1) chat
(2) instant messaging
(3) instanotes
(4) electronic mail
(5) None of these

154. What is the overall term for creating, editing, formatting, storing, retrieving, and printing a text document?

(1) Word processing
(2) Spreadsheet design
(3) Web design
(4) Database management
(5) Presentation generation

155. Fourth-generation mobile technology provides enhanced capabilities allowing the transfer of both data, including full-motion video, high-speed Internet access, and videoconferencing.

(1) video data and information
(2) voice and non-voice
(3) music and video
(4) video and audio
(5) None of these

156. _________ is a form of denial of service attack in which a hostile client repeatedly  attack in which a hostile client repeatedly sends SYN packets to every port on the server, using fake IP addresses.

(1) Cybergaming crime
(2) Memory shaving
(3) Syn flooding
(4) Software piracy
(5) None of these

157. Which of these is a point-and-draw device?

(1) mouse
(2) scanner
(3) printer
(4) CD-ROM
(5) keyboard

158. The letter and number of the intersecting column and row is the

(1) cell location
(2) cell position
(3) cell address
(4) cell coordinates
(5) cell contents

159. A set of rules for telling the computer what operations to perform is called a

(1) procedural language
(2) structures
(3) natural language
(4) command language
(5) programming language

160. A detailed written description of the programming cycle and the program, along with the test results and a printout of the program is called

(1) documentation
(2) output
(3) reporting
(4) spec sheets
(5) directory

161. Forms that are used to organise business data into rows and columns are called

(1) transaction sheets
(2) registers
(3) business forms
(4) sheet-spreads
(5) spreadsheets

162. In PowerPoint, the Header & Footer button can be found on the Insert tab in what group?

(1) Illustrations group
(2) Object group
(3) Text group
(4) Tables group (5) None of these

163. A(n)_____ is a set of programs designed to manage the resources of a computer, including starting the computer, managing programs, managing memory, and coordin-ating tasks between input and output devices.

(1) application suite
(2) compiler
(3) input/output system
(4) interface
(5) operating system (OS)

164. A typical slide in a slide presentation would not include

(1) photo images, charts and graphs
(2) graphs and clip art
(3) clip art and audio clips
(4) full-motion video
(5) content templates

165. The PC productivity tool that manipulates data organised in rows and columns is called a(n)

(1) spreadsheet
(2) word processing document
(3) presentation mechanism
(4) database record manager
(5) EDI creator

166. In the absence of parentheses, the order of operation is

(1) exponentiation, addition or subtraction, multiplication or division
(2) addition or subtraction, multiplication or division, exponentiation
(3) multiplication or division, exponen-tiation, addition or subtraction
(4) exponentiation, multiplication or divi-sion, addition or subtraction
(5) addition or subtraction, exponentiation, multiplication or division

167. To find the Paste Special option, you use the Clipboard group on the _______ tab of PowerPoint.

(1) design
(2) slide Show
(3) page layout
(4) insert
(5) home

168. A(n) program is one that is ready to run and does not need to be altered in any way.

(1) interpreter
(2) high-level
(3) compiler
(4) COBOL
(5) executable

169. Usually downloaded into folders that hold temporary Internet files,_________ are written to your computer’s hard disk by some of the Web sites you visit.

(1) anonymous files
(2) behaviour files
(3) banner ads
(4) large files
(5) cookies

170. What is the easiest way to change the phrase “revenues, profits, gross margin,” to read “revenues, profits, and gross margin”?

(1) Use the insert mode, position the cursor before the g in gross, then type the word and followed by a space
(2) Use the insert mode, position the cursor after the g in gross, then type the word and followed by a space
(3) Use the overtype mode, position the cursor before the g in gross, then type the word and followed by a space
(4) Use the overtype mode, position the cursor after the g in gross, then type the word and followed by a space
(5) None of these

171. A program, either talk or music, that is made available in digital format for automatic download over the Internet is called a

(1) wiki
(2) broadcast
(3) vodcast
(4) bIog
(5) podcast

DOWNLOAD IBPS PO Question Papers PDF

DOWNLOAD IBPS CLERK Question Papers PDF

DOWNLOAD ALL BANK EXAMS PAPERS E-BOOKS

Printed Study Material for IBPS PO Exam

172. Which PowerPoint view displays each slide of the presentation as a thumbnail and is useful for rearranging slides?

(1) Slide sorter
(2) Slide show
(3) Slide master
(4) Notes page
(5) Slide design

173. Different components on the motherboard of a PC unit are linked together by sets of parallel electrical conducting lines. What are these lines called?

(1) Conductors
(2) Buses
(3) Connectors
(4) Consecutives
(5) None of these

174. What is the name given to those applications that combine text, sound, graphics, motion video, and/or animation?

(1) motionware
(2) anigraphics
(3) videoscapes
(4) multimedia
(5) maxomedia

175. A USB communication device that supports data encryption for secure wireless commu-nication for notebook users is called a

(1) USB wireless network adapter
(2) wireless switch
(3) wireless hub
(4) router
(5) None of these

176. A(n) language reflects the way people think mathematically.

(1) cross-platform programming
(2) 3GL business programming
(3) event-driven programming
(4) functional
(5) None of these

177. When entering text within a document, the Enter key is normally pressed at the end of every

(1) line
(2) sentence
(3) paragraph
(4) word
(5) file

178. When a real-time telephone call between people is made over the Internet using computers, it is called

(1) a chat session
(2) an e-mail
(3) an instant message
(4) internet telephony
(5) None of these

179. Which of the following is the first step in sizing a window?

(1) Point to the title bar
(2) Pull down the View menu to display the toolbar
(3) Point to any corner or border
(4) Pull down the View menu and change to large icons
(5) None of these

180. Which of the following software could assist someone who cannot use their hands for computer input?

(1) Videoconferencing
(2) Speech recognition
(3) Audio digitizer
(4) Synthesizer
(5) None of these

181. ......... a document means the file is transferred from another computer to your computer.

(1) Uploading
(2) Really Simple Syndication (RSS)
(3) Accessing
(4) Downloading
(5) Upgrading

182. Which computer memory is used for storing programs and data currently being processed by the CPU?

(1) Mass memory
(2) Internal memory
(3) Non-volatile memory
(4) PROM
(5) None of these

183. Computers that control processes accept data in a continuous

(1) data traffic pattern
(2) data highway
(3) infinite loop
(4) feedback loop
(5) slot

184. What refers to a set of characters of a particular design?

(1) Keyface
(2) Formation
(3) Calligraph
(4) Stencil
(5) Typeface

185. ........ is used by public and private enterprises to publish and share financial information with each other and industry analysts across all computer platforms and the Internet.

(1) Extensible Markup Language (XML)
(2) Extensible Business Reporting Language (XBRL)
(3) Enterprise Application Integration (EAI)
(4) Sales Force Automation (SKA) software
(5) None of these

186. Which part of the computer is used for calculating and comparing?

(1) ALU
(2) Control unit
(3) Disk unit
(4) Modem
(5) None of these

187. The method of Internet access that requires a phone line, but offers faster access speeds than dial-up is the connection.

(1) cable access
(2) satellite access
(3) fibre-optic service
(4) digital subscriber line (DSL)
(5) modem

188. ............. software creates a minor image of the entire hard disk, including the operating system, applications, files, and data.

(1) Operating system
(2) Backup software
(3) Utility programs
(4) Driver imaging
(5) None of these

189. What is a URL?

(1) A computer software program
(2) A type of programming object
(3) The address of a document or “page” on the World Wide Web
(4) An acronym for Unlimited Resources for Learning
(5) A piece of hard ware

190. What is the significance of a faded (dimmed) command in a pull-down menu?

(1) The command is not currently accessible.
(2) A dialog box appears if the command is selected.
(3) A Help window appears if the command is selected.
(4) There are no equivalent keystrokes for the particular command.
(5) None of these

191. Your business has contracted with another company to have thorn host and run an application for your company over the Internet. The company providing this service to your business is called an

(1) internet service provider
(2) internet access provider
(3) application service provider
(4) application access provider
(5) outsource agency

192. A(n) ............. allows you to access your e-mail from anywhere.

(1) forum
(2) webmail interface
(3) message board
(4) weblog
(5) None of these

193. Which of the following would you find on Linkedln?

(1) Games
(2) Connections
(3) Chat
(4) Applications
(5) None of these

194. ............. is a technique that is used to send more than one call over a single line.

(1) Digital transmission
(2) Infrared transmission
(3) Digitizing
(4) Streaming
(5) Multiplexing

195. The Search Companion can

(1) locate all files containing a specified phrase
(2) restrict its search to a specified set of folders
(3) locate all files containing a specified phrase and restrict its search to a specified set of folders
(4) cannot locate all files containing a specified phrase or restrict its search to a specified set of folders
(5) None of these

196. Which of the following cannot be part of an email address?

(1) Period (.)
(2) At sign (@)
(3) Space ()
(4) Underscore (_)
(5) None of these

197. Which of the following must be contained in a URL?

(1) A protocol identifier
(2) The letters www
(3) The unique registered domain name
(4) www. and the unique registered domain name
(5) A protocol identifier, www. and the unique registered domain name

198. Which of the following information systems focuses on making manufacturing processes more efficient and of higher quality?

(1) Computer-aided manufacturing
(2) Computer-integrated manufacturing
(3) Computer-aided software engineering
(4) Computer-aided system engineering
(5) None of these

199. A mistake in an algorithm that causes incorrect results is called a

(1) logical error
(2) syntax error
(3) procedural error
(4) compiler error
(5) machine error

200. A device for changing the connection on a connector to a different configuration is

(1) a converter
(2) a component
(3) an attachment
(4) an adapter
(5) voltmeter

DOWNLOAD IBPS PO Question Papers PDF

DOWNLOAD IBPS CLERK Question Papers PDF

DOWNLOAD ALL BANK EXAMS PAPERS E-BOOKS

Printed Study Material for IBPS PO Exam

IBPS / SBI Special TX: 
General: 
STUDY KITS: 

IBPS (PO) Previous Year Exam Paper - 2012 "General Awareness"

IBPS logo


IBPS (PO) Previous Year Exam Paper - 2012

Subject: General Awareness


101. A money deposited in a bank that cannot be withdrawn for a preset fixed period of time is known as a

(1) Term deposit
(2) Checking Account
(3) Savings Bank Deposit
(4) No Frills Account
(5) Current Deposit

102. A worldwide financial messaging network which exchanges messages between banks and financial institutions is known as

(1) CHAPS
(2) SWIFT
(3) NEFT
(4) SFMS
(5) CHIPS

103. Which of the following ministries alongwith the Planning Commission of India has decided to set up a Corpus Fund of Rs 500 crore, so that tribals in Naxal-hit areas can be provided proper means of livelihood?
 

(1) Ministry of Rural Development
(2) Ministry of Home Affairs
(3) Ministry of Tribal Affairs
(4) Ministry of Corporate Affairs
(5) Ministry of Finance

104. Which of the following was the issue over which India decided to vote against Sri Lanka in the meeting of one of the UN-governed bodies/agencies?

(1) Violations of human rights in Sri Lanka
(2) Allowing China to establish a military base in the Indian Ocean
(3) Issue of subsidy on agricultural products in the meeting of the WTO
(4) Allowing part of Sri Lanka to become an independent country governed by LTTE
(5) Sri Lanka’s claim to become a permanent member of UN Security Council

105. The term ‘Smart Money’ refers to

(1) Foreign Currency
(2) Internet Banking
(3) US Dollars
(4) Travellers’ Cheques
(5) Credit Cards

106. Which of the following is not a ‘Money Market Instrument’?

(1) Treasury Bills
(2) Commercial Paper
(3) Certificate of Deposit
(4) Equity Shares
(5) None of these

107. Which of the following is a retail banking product?

(1) Home Loans
(2) Working capital finance
(3) Corporate term loans
(4) Infrastructure financing
(5) Export Credit

108. Which of the following statements is TRUE about the political situation in Mali, where a military coup burst out recently?

(1) General elections were due there in March/April 2012 but did not take place.
(2) The country has been under the control of the US army for the last 18 months.
(3) The army of the country was not happy as Amadou Toure was made President without elections for the next five years.
(4) A coup broke out in Mali when Amadou Toure, the military chief, got seriously injured in a bomb blast.
(5) NATO and its associates had planned a coup there.

109. In the summit of which of the following organizations/ group of nations it was decided that all members should enforce Budget Discipline?

(1) G-8 (2) OPEC
(3) European Union (4) SAARC
(5) G-20

110. As per newspaper reports, India is planning to use ‘SEU’ as fuel in its 700MW nuclear reactors being developed in new planets. What is the full form of 'SEU’ as used here?

(1) Safe Electrical Units
(2) Small Electrical Units
(3) Slightly Enriched Uranium
(4) Sufficiently Enriched Units
(5) Safely Enriched Uranium

111. Technological advancement in recent times has given a new dimension to banks, mainly to which of the following aspects?

(1) New Age Financial Derivatives
(2) Service Delivery Mechanism
(3) Any Banking
(4) Any Type Banking
(5) Multilevel Marketing

112. When there is a difference between all receipts and expenditure of the Govt of India, both capital and revenue, it is called

(1) Revenue Deficit
(2) Budgetary Deficit
(3) Zero Budgeting
(4) Trade Gap
(5) Balance of payments problem

113. Which of the following is NOT a function of the Reserve Bank of India?

(1) Fiscal Policy Functions
(2) Exchange Control Functions
(3) Issuance, Exchange and Destruction of currency notes
(4) Monetary Authority Functions
(5) Supervisory and Control Functions

114. Which of the following is NOT required for opening a bank account?

(1) Identity Proof
(2) Address Proof
(3) Recent Photographs
(4) Domicile Certificate
(5) None of these

115. The Golden Jubilee of Afro-Asian Rural Development Organisation was organized in which of the following places in March 2012?

(1) Dhaka (2) Tokyo
(3) Cairo (4) Kuala Lampur
(5) New Delhi

116. What is the maximum deposit amount insured by DICGC?

(1) Rs 2,00,000 per depositor per bank
(2) Rs 2,00,000 per depositor across all banks
(3) Rs 1,00,000 per depositor per bank
(4) Rs 1,00,000 per depositor across all banks
(5) None of these

117. The present Foreign Trade policy of India will continue till

(1) December 2012
(2) March 2013
(3) March 2014
(4) June 2013
(5) December 2014

118. With reference to a cheque, which of the following is the ‘drawee bank’?

(1) The bank that collects the cheque
(2) The payee’s bank
(3) The endorsee’s bank
(4) The endorser’s bank
(5) The bank upon which the cheque is drawn

119. In which of the following fund transfer mechanisms, can funds be moved from one bank to another and where the transaction is settled instantly without being bunched with any other transaction?

(1) RTGS
(2) NEFT
(3) TT
(4) EFT
(5) MT

120. What was the reason owing to which Enrica Lexie, an Italian ship, was detained by the Port Authorities in Kerala and was brought to Cochin port for inspection and search?

(1) It was carrying objectionable material.
(2) It was involved in sea piracy.
(3) It was detained as the crew killed two Indian fishermen.
(4) The ship started sailing without making payments of iron ore it loaded.
(5) It was detained as it was dumping nuclear waste in deep sea.

DOWNLOAD IBPS PO Question Papers PDF

DOWNLOAD IBPS CLERK Question Papers PDF

DOWNLOAD ALL BANK EXAMS PAPERS E-BOOKS

Printed Study Material for IBPS PO Exam

121. Which of the following agencies/organiz-ations recently gave a ‘go-ahead’ verdict to India’s one of the most controversial project of interlinking rivers? (Some of the rivers are international.)

(1) UN Food and Agriculture Organisation
(2) World Meteorological Organisation
(3) International Court of Justice
(4) Central Water Commission
(5) Supreme Court of India

122. Who among the following was the Captain of the Australian Team which played four Test matches against India in January 2012 and won all of them?

(1) Ricky Ponting
(2) Michael Clarke
(3) Nathan Lyon
(4) Stuart Clark
(5) Andrew Symonds

123. The committee on review of National Small Savings Fund (NSSF) was headed by

(1) Dr C Rangarajan
(2) UK Sinha
(3) Dr YV Reddy
(4) Shyamala Gopinath
(5) Dr Usha Thorat

124. Banking Ombudsman Scheme is applicable to the business of

(1) All scheduled commercial banks exclu-ding RRBs
(2) All scheduled commercial banks inclu-ding RRBs
(3) Only public sector banks
(4) All banking companies
(5) All scheduled banks except private banks

125. The 5th Asian Indoor Athletics Champio-nship was organised in February 2012 in

(1) Bangladesh
(2) India
(3) Qatar
(4) China
(5) South Korea

126. Nationalisations of banks aimed at all of the following except

(1) Provision of adequate credit for agriculture, SME & exports
(2) Removal of control by a few capitalists
(3) Provision of credit to big industries only
(4) Access of banking to masses
(5) Encouragement of a new class of entrepreneurs

127. Who among the following was the Chief Guest at the three-day Pravasi Bharatiya Divas function held in January 2012?

(1) Kamla Persad Bissessar
(2) Ram Baran Yadav
(3) Lakshmi Mittal
(4) Salman Rushdie
(5) Benjamin Netanyahu

128. Which of the following banks is headed by a woman CEO?

(1) Bank of Baroda
(2) HDFC Bank
(3) Central Bank of India
(4) Punjab National Bank
(5) lCICI Bank

129. As per revised RBI guidelines, Provision on secured portion of loan accounts classified as Doubtful Assets for more than one year and upto 3 (three) years is to be made at the rate of

(1) 15%
(2) 20%
(3) 40%
(4) 25%
(5) 30%

130. As per the provisions in the Food Security Bill 2011, how much food grain should be given per month to each person of the target group?

(1) 5 kg
(2) 7 kg
(3) 9 kg
(4) 10 kg
(5) 20 kg

131. Which of the following acts is in vogue in India is against the thinking of raising school fee as per demand of the market forces?

(1) Prevention of Corruption Act
(2) Child Labour (Prohibition & Regulation) Act
(3) Sharda Act
(4) Right to Education Act
(5) MG National Rural Employment Guarantee Act

132. Which of the following states has launched ‘Panch Parmeshwar Yojana’,under which panchayats are allowed to use the fund for developing infrastructure and basic amenities in villages?

(1) Uttar Pradesh
(2) Maharashtra
(3) Gujarat
(4) Madhya Pradesh
(5) Tamil Nadu

133. Base Rate is the rate below which no Bank can allow their lending to anyone. Who sets up this ‘Base Rate’ for Banks?

(1) Individual Banks’ Board
(2) Ministry of Commerce
(3) Ministry of Finance
(4) RBI
(5) Interest Rate Commission of India

134. National Table Tennis Championship was organised in January 2012 in

(1) Mumbai
(2) Delhi
(3) Hyderabad
(4) Lucknow
(5) Jaipur

135. Who among the following is the author of the book Nirbasan

(1) Mahashweta Devi
(2) Taslima Nasreen
(3) Sunil Gangopadhyay
(4) Vikram Seth
(5) Kiran Desai

136. What is a’Debit Card’?

(1) It is a card issued by a Rating Agency.
(2) It is a card which can be used for withdrawing cash or making payment even in the absence of any balance in the account.
(3) It is a card which can be used for withdrawing cash or making payment if there is balance in the account.
(4) It is a card which carries prepaid balance.
(5) It is a card which can be used for making STD calls.

137. Who among the following Indian Lawn Tennis players won a Doubles of Australian Open 2012?

(1) Mahesh Bhupathi
(2) Kamlesh Mehta
(3) Leander Paes (4) Sania Mirza
(5) Achanta Sarath Kamal

138. ‘Akula-11’ Class ‘K.-152 Nerpa was inducted in Indian Navy recently. These are

(1) Aircraft
(2) Radar Systems
(3) Submarines Missile
(4) Warships
(5) Submarines

139. Which of the following awards was conferred upon late Mario de Miranda (posthumously) in January 2012?

(1) Padma Vibhushan
(2) Bharat Ratna
(3) Kalidas Samman
(4) Saraswati Samman
(5) Padmashri

140. Bad advances of a Bank are called

(1) Bad debt
(2) Book debt
(3) Non-Performing Asset
(4) Out-of-order accounts
(5) Overdrawn accounts

141. Axis Bank is a

(1) Public Sector Bank
(2) Private Sector Bank
(3) Co-operative Bank
(4) Foreign Bank (5) Gramin Bank

142. By increasing repo rate, the economy may observe the following effect(s):

(1) Rate of interest on loans and advances will be costlier.
(2) Industrial output would be affected to an extent.
(3) Banks will increase rate of interest on deposits.
(4) Industry houses may borrow money from foreign countries.
(5) All of these

143. Increased interest rates, as is existing in the economy at present, will

(1) Lead to higher GDP growth
(2) Lead to lower GDP growth
(3) Mean higher cost of raw materials
(4) Mean lower cost of raw materials
(5) Mean higher wage bill

144. Which of the following schemes has been launched to provide pension to people in unorganised sectors in India?

(1) Swabhiman
(2) Jeevan Dhara
(3) Jeevan Kalyan
(4) ASHA
(5) Swavalamban

145. The 10th Basic Ministerial Meeting on Climate Change was organised in February 2012 in

(1) Tokyo
(2) Beijing
(3) Manila
(4) Moscow
(5) New Delhi

146. Finance Ministry has asked the Reserve Bank of India to allow common ATMs that will be owned and managed by non-banking entities, hoping to cut transaction costs for banks. Such ATMs are known as

(1) Black Label ATMs
(2) Offsite ATMs
(3) On-site ATM’s or Red ATMs
(4) Third Party ATMs
(5) White Label ATMs

147. Which of the following schemes of the Govt of India has provided electricity to 99000 villages and total 1.7 crore households uptil now?

(1) Kutir Jyoti
(2) Rajiv Gandhi Grameen Vidyutikaran Yojana
(3) Bharat Nirman
(4) PURA (5) SEWA

148. Ranbir Kapoor was awarded Best Actor Award in 57th Filmfare Award Function for his performance in

(1) No One Killed Jessica
(2) Stanley Ka Dabba
(3) 7 Khoon Maaf (4) Rockstar
(5) Zindagi Na Milegi Dobara

149. An ECS transaction gets bounced and you are unable to recover your money from your customer. Under which Act can criminal action be initiated?

(1) Indian Penal Code
(2) Negotiable Instruments Act
(3) Criminal Procedure Code
(4) Payment and Settlements Act
(4) Indian Contract Act

150. Rajendra had filed a complaint with Banking Ombudsman but is not satisfied with the decision. What is the next option before him for getting his matter resolved?

(1) Write to the CMD of the Bank
(2) File an appeal before the Finance Minister
(3) File an appeal before the Banking Ombudsman again
(4) File an appeal before the Dy Governor of RBI
(5) Simply close the matter as going to court involves time and money

DOWNLOAD IBPS PO Question Papers PDF

DOWNLOAD IBPS CLERK Question Papers PDF

DOWNLOAD ALL BANK EXAMS PAPERS E-BOOKS

Printed Study Material for IBPS PO Exam

IBPS / SBI Special TX: 
General: 
STUDY KITS: 

(Papers) IBPS PO Exam Paper - 2012 "Held on: 17-06-2012" ::Quantitative Aptitude::

IBPS logo


(Papers) IBPS PO Exam Paper - 2012 "Held on: 17-06-2012"

::Quantitative Aptitude::


DIRECTIONS (Qs. 51-55) : What will come in place of the question mark (?) in the following questions ?

1. 4003 × 77 – 21015 = ? × 116

(a) 2477
(b) 2478
(c) 2467
(d) 2476
(e) None of these

(a) 143
(b) 72 7
(c) 134
(d) 70 7
(e) None of these

3. (4444 ÷ 40) + (645 ÷ 25) + (3991÷ 26) = ?

(a) 280.4
(b) 290.4
(c) 295.4
(d) 285.4
(e) None of these

(a) 37
(b) 33
(c) 34
(d) 28
(e) None of these

(a) 303.75
(b) 305.75
(c) 303
(d) 305
(e) None of these

DIRECTIONS (Qs. 56-60) : What approximate value should come in place of the question mark (?) in the following questions? (Note : You are not expected to calculate the exact value.)

6. 8787 ÷ 343 × 50 = ?

(a) 250
(b) 140
(c) 180
(d) 100
(e) 280

7. 3 54821 × (303 ÷ 8) = (?)2

(a) 48
(b) 38
(c) 28
(d) 18
(e) 58

8.5/8 of 4011.33 +7/10 of 3411.22 = ?

(a) 4810
(b) 4980
(c) 4890
(d) 4930
(e) 4850

9. 23% of 6783 + 57% of 8431 = ?

(a) 6460
(b) 6420
(c) 6320
(d) 6630
(e) 6360

10. 335.01 × 244.99 ÷ 55 = ?

(a) 1490
(b) 1550
(c) 1420
(d) 1590
(e) 1400

DIRECTIONS (Qs. 11-15) : In each of these questions a number series is given. In each series only one number is wrong. Find out the wrong number.

11. 5531 5506 5425 5304 5135 4910 4621

(a) 5531
(b) 5425
(c) 4621
(d) 5135
(e) 5506

12. 6 7 9 13 26 37 69

(a) 7
(b) 26
(c) 69
(d) 37
(e) 9

13. 1 3 10 36 152 760 4632

(a) 3
(b) 36
(c) 4632
(d) 760
(e) 152

14. 7 4 5 9 20 51 160.5

(a) 4
(b) 5
(c) 9
(d) 20
(e) 51

15. 157.5 45 15 6 3 2 1

(a) 1
(b) 2
(c) 6
(d) 157.5
(e) 45

Click Here To Download Full Paper PDF

DOWNLOAD IBPS PO Question Papers PDF

DOWNLOAD IBPS CLERK Question Papers PDF

DOWNLOAD ALL BANK EXAMS PAPERS E-BOOKS

Printed Study Material for IBPS PO Exam

DIRECTIONS (Qs. 16-20) : Study the following graph and table carefully and answer the questions given below :

16. Which of the following vehicles travelled at the same speed on both the days ?

(a) Vehicle A
(b) Vehicle C
(c) Vehicle F
(d) Vehicle B
(e) None of these

17. What was the difference between the speed of vehicle A on day 1 and the speed of vehicle C on the same day ?

(a) 7 km/hr.
(b) 12 km/hr.
(c) 11 km/hr.
(d) 8 km/hr.
(e) None of these

18. What was the speed of vehicle C on day 2 in terms of meters per second ?

(a) 15.3
(b) 12.8
(c) 11.5
(d) 13.8
(e) None of these

19. The distance travelled by vehicle F on day 2 was approximately what percent of the distance travelled by it on day 1 ?

(a) 80
(b) 65
(c) 85
(d) 95
(e) 90

20. What is the respective ratio between the speeds of vehicle D and vehicle E on day 2 ?

(a) 15:13
(b) 17:13
(c) 13:11
(d) 17:14
(e) None of these

21. An article was purchased for ` 78,350/-. Its price was marked up by 30%. It was sold at a discount of 20% on the marked up price. What was the profit percent on the cost price ?

(a) 4
(b) 7
(c) 5
(d) 3
(e) 6

22. When X is subtracted from the numbers 9,15 and 27, the remainders are in continued proportion. What is the value of X ?

(a) 8
(b) 6
(c) 4
(d) 5
(e) None of these

23. What is the difference between the simple and compound interest on ? ` 7,300/- at the rate of 6 p.c.p.a. in 2 years ?

(a) ` 29.37/-
(b) ` 26.28/-
(c) ` 31.41/-
(d) ` 23.22/-
(e) ` 21.34/-

24. Sum of three consecutive numbers is 2262. What is 41 % of the highest number ?

(a) 301.51
(b) 303.14
(c) 308.73
(d) 306.35
(e) 309.55

25. In how many different ways can the letters of the word ‘THERAPY’ be arranged so that the vowels never come together ?

(a) 720
(b) 1440
(c) 5040
(d) 3600
(e) 4800

Edit your toolbar now!

IBPS / SBI Special TX: 
General: 

(Papers) IBPS PO Exam Paper - 2012 "Held on: 17-06-2012" ::General Awareness::

IBPS logo


(Papers) IBPS PO Exam Paper - 2012 "Held on: 17-06-2012"

::General Awareness::


1. A money deposited at a bank that cannot be withdrawn for a preset fixed period of time is known as a __________ .

(a) Term deposit
(b) Checking Account
(c) Savings Bank Deposit
(d) No Frills Account
(e) Current Deposit

2. A worldwide financial messaging network which exchanges messages between banks and financial institutions is known as __________ .

(a) CHAPS
(b) SWIFT
(c) NEFT
(d) SFMS
(e) CHIPS

3. Which of the following Ministries along with Planning Commission of India has decided to set up a Corpus Fund of ` 500 crore, so that Tribals in Naxal-hit areas can be provided proper means of livelihood ?

(a) Ministry of Rural Development
(b) Ministry of Home Affairs
(c) Ministry of Tribal Affairs
(d) Ministry of Corporate Affairs
(e) Ministry of Finance

4. Which of the following was the issue over which India decided to vote against Sri Lanka in the meeting of one of the UN governed bodies/agencies ?

(a) Violations of human rights in Sri Lanka
(b) Allowing China to establish a military base in Indian ocean
(c) Issue of subsidy on agricultural products in the meeting of the WTO
(d) Allowing part of Sri Lanka to become an independent country governed by LTTE
(e) Sri Lanka's claim to become a permanent member of UN Security Council

5. The term 'Smart Money" refers to __________ .

(a) Foreign Currency
(b) Internet Banking
(c) US Dollars
(d) Travelers' cheques
(e) Credit Cards

6. Which one of the following is not a 'Money Market Instrument' ?

(a) Treasury Bills
(b) Commercial Paper
(c) Certificate of Deposit
(d) Equity Shares
(e) None of these

7. Which one of the following is a retail banking product ?

(a) Home Loans
(b) Working capital finance
(c) Corporate term loans
(d) Infrastructure financing
(e) Export Credit

8. Which of the following Statements is TRUE about political situation in Mali, where a military coup burst out recently ?

(a) General elections were due there in March/April 2012 but did not take place
(b) The country was under the control of US army since last 18 months
(c) Army of the country was not happy as Amadou Toure was made President without elections for the next five years
(d) Coup broke out in Mali when Amadou Toure. the Military Chief got seriously injured in a bomb blast
(e) NATO and its associates had planned a coup there

9. In the summit of which of the following organizations/group of Nations it was decided that all members should enforce Budget Discipline ?

(a) G-8
(b) OPEC
(c) European Union
(d) SAARC
(e) G-20

10. As per newspaper reports, India is planning to use 'SEU' as fuel in its 700 MW nuclear reactors being developed in new plants. What is full form of 'SEU' as used here ?

(a) Safe Electrical Units
(b) Small Electrical Unite
(c) Slightly Enriched Uranium
(d) Sufficiently Enriched Units
(e) Safely Enriched Uranium

11. Technological Advancement in the recent times has given a new dimension to banks, mainly to which one of the following aspects ?

(a) New Age Financial Derivatives
(b) Service Delivery Mechanism
(c) Any Banking
(d) Any Type Banking
(e) Multilevel Marketing

12. When there is a difference between all receipts and expenditure of the Govt. of India, both capital and revenue, it is called __________ .

(a) Revenue Deficit
(b) Budgetary Deficit
(c) Zero Budgeting
(d) Trade Gap
(e) Balance of payment problem

13. Which of the following is NOT a function of the Reserve Bank of India ?

(a) Fiscal Policy Functions
(b) Exchange Control Functions
(c) Issuance, Exchange and destruction of currency notes
(d) Monetary Authority Functions
(e) Supervisory and Control Functions

14. Which of the following is NOT required for opening a bank account ?

(a) Identity Proof
(b) Address Proof
(c) Recent Photographs
(d) Domicile Certificate
(e) None of these

15. The Golden Jubilee of Afro-Asian Rural Development organization was organized in which of the following places in March 2012 ?

(a) Dhaka
(b) Tokyo
(c) Cairo
(d) Kuala Lampur
(e) New Delhi

16. What is the maximum deposit amount insured by DICGC ?

(a) ` 2,00,000 per depositor per bank
(b) ` 2,00,000 per depositor across all banks
(c) ` 1,00,000 per depositor per bank
(d) ` 1,00,000 per depositor across all banks
(e) None of these

Click Here To Download Full Paper PDF

DOWNLOAD IBPS PO Question Papers PDF

DOWNLOAD IBPS CLERK Question Papers PDF

DOWNLOAD ALL BANK EXAMS PAPERS E-BOOKS

Printed Study Material for IBPS PO Exam

17. The present Foreign Trade policy of India will continue till-

(a) December 2012
(b) March 2013
(c) March 2014
(d) June 2013
(e) December 2014

18. With reference to a cheque which of the following is the “drawee bank” ?

(a) The bank that collects the cheque
(b) The payee's bank
(c) The endorsee's bank
(d) The endorser's bank
(e) The bank upon which the cheque is drawn

19. In which of the following fund transfer mechanisms, can funds be moved from one bank to another and where the transaction is settled instantly without being bunched with any other transaction ?

(a) RTGS
(b) NEFT
(c) TT
(d) EFT
(e) MT

20. What was the reason owing to which Enrica Lexie, an Italian ship was detained by the Port Authorities in Kerala and was brought to Cochin port for inspection and search ?

(a) It was carrying objectionable material
(b) It was involved in sea piracy
(c) It was detained as the crew killed two Indian fishermen
(d) The ship started sailing without making payments of iron ore it loaded
(e) It was detained as it was dumping nuclear waste in deep sea

21. Which of the following agencies/organizations recently gave 'go-ahead' verdict to India's one of the most controversial project of inter linking rivers? (some of the rivers are international rivers)

(a) UN Food and Agriculture Organisation
(b) World Meteorological Organisation
(c) International Court of Justice
(d) Central Water Commission
(e) Supreme Court of India

22. Who among the following was the Captain of the Australian Team which played 4 Test matches against India in January 2012 and won all of them ?

(a) Ricky Ponting
(b) Michael Clarke
(c) Nathan Lyon
(d) Stuart Clark
(e) Andrew Symonds

23. The committee on review of National Small Saving Fund (NSSF) was headed by __________ .

(a) Dr. C. Rangarajan
(b) Mr. U. K. Sinha
(c) Dr. Y. V. Reddy
(d) Mrs.Shyamala Gopinath
(e) Dr. Usha Thorat

24. Banking Ombudsman Scheme is applicable to the business of __________ .

(a) All scheduled commercial banks excluding RRBs
(b) All scheduled commercial banks including RRBs
(c) Only Public Sector Banks
(d) All Banking Companies
(e) All scheduled banks except private banks

25. The 5th Asian tndoor Athletics Championship was organized in February 2012 in __________ .

(a) Bangladesh
(b) India
(c) Qatar
(d) China
(e) South Korea

IBPS / SBI Special TX: 
General: 

(Papers) IBPS PO Exam Paper - 2012 "Held on: 17-06-2012" ::COMPUTER KNOWLEDGE::

IBPS logo


(Papers) IBPS PO Exam Paper - 2012 "Held on: 17-06-2012"

::COMPUTER KNOWLEDGE::


1. __________ allows users to upload files to an online site so they can be viewed and edited from another location.

(a) General-purpose applications
(b) Microsoft Outlook
(c) Web-hosted technology
(d) Office Live
(e) None of these

2. What feature adjusts the top and bottom margins so that the text is centered vertically on the printed page ?

(a) Vertical justifying
(b) Vertical adjusting
(c) Dual centering
(d) Horizontal centering
(e) Vertical centering

3. Which of these is not a means of personal communication on the Internet ?

(a) chat
(b) instant messaging
(c) instanotes
(d) electronic mail
(e) None of these

4. What is the overall term for creating, editing, formatting, storing, retrieving, and printing a text document ?

(a) Word processing
(b) Spreadsheet design
(c) Web design
(d) Database management
(e) Presentation generation

5. Fourth-generation mobile technology provides enhanced capabilities allowing the transfer of both __________ data, including full-motion video, high-speed Internet access, and video conferencing.

(a) video data and information
(b) voice and nonvoice
(c) music and video
(d) video and audio
(e) None of these

6. __________ is a form of denial of service attack in which a hostile client repeatedly sends SYN packets to every port on the server using fake IP addresses.

(a) Cyborgaming crime
(b) Memory shaving
(c) Syn flooding
(d) Software piracy
(e) None of these

7. Which of these is a point-and-draw device ?

(a) mouse
(b) scanner
(c) printer
(d) CD-ROM
(e) Keyboard

8. The letter and number of the intersecting column and row is the __________.

(a) cell location
(b) cell position
(c) cell address
(d) cell coordinates
(e) cell contents

9. A set of rules for telling the computer what operations to perform is called a __________.

(a) procedural language
(b) structures
(c) natural language
(d) command language
(e) programming language

10. A detailed written description of the programming cycle and the program, along with the test results and a printout of the program is called __________.

(a) documentation
(b) output
(c) reporting
(d) spec sheets
(e) Directory

11. Forms that are used to organize business data into rows and columns are called __________.

(a) transaction sheets
(b) registers
(c) business forms
(d) sheet-spreads
(e) spreadsheets

12. In Power Point, the Header and Footer button can be found on the Insert tab in what group ?

(a) Illustrations group
(b) Object group
(c) Text group
(d) Tables group
(e) None of these

Click Here To Download Full Paper PDF

DOWNLOAD IBPS PO Question Papers PDF

DOWNLOAD IBPS CLERK Question Papers PDF

DOWNLOAD ALL BANK EXAMS PAPERS E-BOOKS

Printed Study Material for IBPS PO Exam

13. A(n)_________is a set of programs designed to manage the resources of a computer, including starting the computer, managing programs, managing memory, and coordinating tasks between input and output devices.

(a) application suite
(b) compiler
(c) input/output system
(d) interface
(e) operating system (OS)

14. A typical slide in a slide presentation would not include __________.

(a) photo images charts, and graphs
(b) graphs and clip art
(c) clip art and audio clips
(d) full-motion video
(e) content templates

15. The PC productivity tool that manipulates data organized in rows and columns is called a __________.

(a) spreadsheet
(b) word processing document
(c) presentation mechanism
(d) database record manager
(e) EDI creator

16. In the absence of parentheses, the order of operation is __________.

(a) Exponentiation, addition or subtraction, multiplication or division
(b) Addition or subtraction, multiplication or division, exponentiation
(c) Multiplication or division, exponentiation, addition or subtraction
(d) Exponentiation, multiplication or division, addition or subtraction
(e) Addition or subtraction, exponentiation, Multiplication or division

17. To find the Paste Special option, you use the Clipboard group on the _____ tab of PowerPoint.

(a) Design
(b) Slide Show
(c) Page Layout
(d) Insert
(e) Home

18. An _____ program is one that is ready to run and does not need to be altered in any way.

(a) interpreter
(b) high-level
(c) compiler
(d) COBOL
(e) executable

19. Usually downloaded into folders that hold temporary Internet files, _____ are written to your computer's hard disk by some of the Web sites you visit.

(a) anonymous files
(b) behaviour files
(c) banner ads
(d) large files
(e) cookies

20. What is the easiest way to change the phrase, revenues, profits, gross margin, to read revenues, profits, and gross margin ?

(a) Use the insert mode, position the cursor before the g gross, then type the word and followed by a space
(b) Use the insert mode, position the cursor after the g in gross, then type the word and followed by a space
(c) Use the overtype mode, position the cursor before the g in gross, then type the word and followed by a space
(d) Use the overtype mode, position the cursor after the g in gross, then type the word and followed by a space
(e) None of these

21. A program, either talk or music, that is made available in digital format tor automatic download over the Internet is called a __________.

(a) wiki
(b) broadcast
(c) vodcast
(d) blog
(e) podcast

22. Which PowerPoint view displays each slide of the presentation as a thumbnail and is useful for rearranging slides ?

(a) Slide Sorter
(b) Slide Show
(c) Slide Master
(d) Notes Page
(e) Slide Design

23. Different components on the motherboard of a PC unit are linked together by sets of parallel electrical conducting lines. What are these lines called ?

(a) Conductors
(b) Buses
(c) Connectors
(d) Consecutives
(e) None of these

24. What is the name given to those applications that combine text, sound, graphics, motion video, and/ or animation ?

(a) motionware
(b) anigraphics
(c) videoscapes
(d) multimedia
(e) maxomedia

25. A USB communication device that supports data encryption for secure wireless communication for notebook users is called a __________.

(a) USB wireless network adapter
(b) wireless switch
(c) wireless hub
(d) router
(e) None of these

IBPS / SBI Special TX: 
General: 

(Papers) IBPS PO Exam Paper - 2012 "Held on: 17-06-2012" ::ENGLISH LANGUAGE::

IBPS logo


(Papers) IBPS PO Exam Paper - 2012 "Held on: 17-06-2012"

::ENGLISH LANGUAGE::


DIRECTIONS (Qs. 1-15) : Read the following passage carefully and answer the questions given below it. Certain words/ phrases have been printed in bold to help you locate  them while answering some of the questions.

When times are hard, doomsayers are aplenty. The problem is that if you listen to them too carefully, you tend to overlook the most obvious signs of change. 2011 was a bad year. Can 2012 be any worse? Doomsday forecasts are the easiest to make these days. So let's try a contrarian's forecast instead. Let's start with the global economy. We have seen a steady flow of good news from the US. The employment situation seems to be improving rapidly and consumer sentiment, reflected in retail expenditures on discretionary items like electronics and clothes, has picked up. If these trends sustain, the US might post better growth numbers for 2012 than the 1.5-1.8 percent being forecast currently. Japan is likely to pull out of a recession in 2012 as postearthquake reconstruction efforts gather momentum and the fiscal stimulus announced in 2011 begins to pay off. The consensus estimate for growth in Japan is a respectable 2 per cent for 2012. The "hard-landing" scenario for China remains and will remain a myth. Growth might decelerate further from the 9 per cent that it expected to clock in 2011 but is unlikely to drop below 8-8.5 percent in 2012. Europe is certainly in a spot of trouble. It is perhaps already in recession and for 2012 it is likely to post mildly negative growth. The risk of implosion has dwindled over the last few months - peripheral economies like Greece, Italy and Spain have new governments in place and have made progress towards genuine economic reform. Even with some of these positive factors in place, we have to accept the fact that global growth in 2012 will be tepid. But there is a flipside to this. Softer growth means lower demand for commodities and this is likely to drive a correction in commodity prices. Lower commodity inflation will enable emerging market central banks to reverse their monetary stance. China, for instance, has already reversed its stance and has pared its reserve ratio twice. The RBI also seems poised for a reversal in its rate cycle as headline inflation seems well on its way to its target of 7 per cent for March 2012. That said, oil might be an exception to the general trend in commodities. Rising geopolitical tensions, particularly the continuing face-off between Iran and the US, might lead to a spurt in prices. It might make sense for our oil companies to hedge this risk instead of buying oil in the spot market. As inflation fears abate and emerging market central banks begin to cut rates, two things could happen Lower commodity inflation would mean lower interest rates and better credit availability. This could set a floor to growth and slowly reverse the business cycle within these economies. Second, as the fear of untamed, runaway inflation in these economies abates, the global investor's comfort levels with their markets will increase. Which of the emerging markets will outperform and who will get left behind? In an environment in which global growth is likely to be weak, economies like India that have a powerful domestic consumption dynamic should lead; those dependent on exports should, prima facie, fall behind. Specifically for India, a fall in the exchange rate could not have come at a better time. It will help Indian exporters gain market share even it global trade remains depressed. More importantly, it could lead to massive import substitution that favours domestic producers. Let's now focus on India and start with a caveat. It is important not to confuse a short-run cyclical dip with a permanent de-rating of its long-term structural potential. The arithmetic is simple. Our growth rate can be in the range of 7-10 per cent depending on policy action.  Ten per cent if we get everything right, 7 per cent if we get it all wrong. Which policies and reforms are critical to taking us to our 10 per cent potential ? In judging this, let's again be careful. Lets not go by the laundry list of reforms that FIIs like to wave: increase in foreign equity limits in foreign shareholding, greater voting rights tor institutional shareholders in banks, FDI in retail, etc. These can have an impact only at the margin. We need not bend over backwards to appease the FIIs through these reforms - they will invest in our markets when momentum picks up and will be the first to exit when the momentum flags, reforms or not. The reforms that we need are the ones that can actually raise out. Sustainable long-term growth rate. These have to come in areas like better targeting of subsidies, making projects in infrastructure viable so that they draw capital, raising the productivity of agriculture, improving healthcare and education, bringing the parallel economy under the tax net, implementing fundamental reforms in taxation like GST and the direct tax code and finally easing the myriad rules andregulations that make doing business in India such a nightmare. A number of these things do not require new legislation and can be done through executive order.

1. Which of the following is NOT TRUE according to the passage ?

(a) China's economic growth may decline in the year 2012 as compared to the year 2011
(b) The European economy is not doing very well
(c) Greece is on the verge of bringing about economic reforms
(d) In the year 2012, Japan may post a positive growth and thus pull out of recession
(e) All are true

2. Which of the following will possibly be a result of softer growth estimated for the year 2012 ?

(A) Prices of oil will not increase.
(B) Credit availability would be lesser.
(C) Commodity inflation would be lesser.

(a) Only (B)
(b) Only (A) and (B)
(c) Only (A) and (C)
(d) Only (C)
(e) All (A), (B) and (C)

3 Which of the following can be said about the present status of the US economy ?

(a) There is not much improvement in the economic scenario of the country from the year 2011
(b) The growth in the economy of the country, in the year 2012, would definitely be lesser than 1.8 percent
(c) The expenditure on clothes and electronic commodities, by consumers, is lesser than that in the year 2011
(d) There is a chance that in 2012 the economy would do better than what has been forecast
(e) The pace of change in the employment scenario of the country is very slow.

4. Which of the following is possibly the most appropriate title for the passage ?

(a) The Economic Disorder
(b) Indian Economy Versus The European Economy
(c) Global Trade
(d) The Current Economic Scenario
(e) Characteristics of The Indian Economy

5. According to the author, which of the following would characterize Indian growth scenario in 2012 ?

(A) Domestic producers will take a hit because of depressed global trade scenario.
(B) On account of its high domestic consumption, India will lead.
(C) Indian exporters will have a hard time in gaining market share.

(a) Only (B)
(b) Only (A) and (B)
(c) Only (B) and (C)
(d) Only (A)
(e) All (A), (B) and (C)

6. Why does the author not recommend taking up the reforms suggested by FII's ?

(a) These will bring about only minor growth
(b) The reforms suggested will have no effect on the economy ol our country, whereas will benefit the FII's significantly
(c) The previous such recommendations had backfired
(d) These reforms will be the sole reason for our country's economic downfall
(e) The reforms suggested by them are not to be trusted as they will not bring about any positive growth in India

7. Which of the following is TRUE as per the scenario presented in the passage?

(a) The highest growth rate that India can expect is 7 percent
(b) The fall in the exchange rate will prove beneficial to India
(c) Increased FDI in retail as suggested by Flls would benefit India tremendously
(d) The reforms suggested by the author require new legislation in India
(e) None is true

8. According to the author, which ot the following reform/s is/ are needed to ensure long term growth in India?

(A) Improving healthcare and educational facilities.
(B) Bringing about reforms in taxation.
(C) Improving agricultural productivity.

(a) Only (B)
(b) Only (A) and (B)
(c) Only (B) and (C)
(d) Only (A)
(e) All (A), (B) and (C)

DIRECTIONS (Qs. 9-12) : Choose the word/group of words which is most similar in meaning to the word/group of words printed in bold as used in the passage.

9. DRAW

(a) entice
(b) push
(c) decoy
(d) attract
(e) persuade

10. CLOCK

(a) watch
(b) achieve
(c) time
(d) second
(e) regulate

11. ABATE

(a) rise
(b) gear
(c) hurl
(d) lessen
(e) retreat

12. EMERGING

(a) raising
(b) developing
(c) noticeable
(d) conspicuous
(e) uproaring

DIRECTIONS (Qs. 13-15) : Choose the word/group of words which is most opposite in meaning to the word/group of words printed in bold as used in the passage.

13. MYRIAD

(a) trivial
(b) difficult
(c) few
(d) effortless
(e) countless

14. TEPID

(a) moderate
(b) high
(c) warm
(d) irregular
(e) little

15. MYTH

(a) reality
(b) belief
(c) contrast
(d) idealism
(e) falsehood

Click Here To Download Full Paper PDF

DOWNLOAD IBPS PO Question Papers PDF

DOWNLOAD IBPS CLERK Question Papers PDF

DOWNLOAD ALL BANK EXAMS PAPERS E-BOOKS

Printed Study Material for IBPS PO Exam

DIRECTIONS (Qs. 16-20) : Rearrange the following six sentences (A), (B), (C), (D), (E) and (F) in the proper sequence to form a meaningful paragraph; then answer the questions given below them.

(A) If China is the world's factory, India has become the world's outsourcing centre - keeping in line with this image.
(B) But India's future depends crucially on its ability to compete fully in the Creative Economy - not just in tech and software, but across design and entrepreneurship; arts, culture and entertainment; and the knowledge-based professions of medicine, finance and law.
(C) While its creative assets outstrip those of other emerging competitors, India must address several challenges to increase its international competitiveness as the world is in the midst of a sweeping transformation.
(D) This transformation is evident in the fact that the world is moving from an industrial economy to a Creative Economy that generates wealth by harnessing intellectual labour, intangible goods and human creative capabilities.
(E) Its software industry is the world's second-largest, its tech outsourcing accounts for more than hall of the $ 300 billion global industry, according to a technology expert.
(F) If the meeting of world leaders at Davos is any indication, India is rapidly becoming an economic'rock star'.

16. Which of the following should be the SIXTH (LAST) sentence after the rearrangement ?

(a) A
(b) B
(c) C
(d) D
(e) E

17. Which of the following should be the THIRD sentence after the rearrangement ?

(a) A
(b) B
(c) C
(d) D
(e) E

18. Which of the following should be the FIFTH sentence after the rearrangement ?

(a) A
(b) B
(c) C
(d) F
(e) E

19. Which of the following should be the FIRST sentence after the rearrangement ?

(a) F
(b) B
(c) C
(d) A
(e) E

20. Which of the following should be the SECOND sentence after the rearrangement ?

(a) A
(b) B
(c) C
(d) D
(e) F

DIRECTIONS (Qs. 21-25) : The following questions consist of a single sentence with one blank only. You are given six words denoted by A, B, C, D, E & F as answer choices and from the six choices you have to pick two correct answers, either of which will make the sentence meaningfully complete.

21. ___________ before the clock struck 8 on Saturday night, India Gate was swamped with people wearing black teeshirts and holding candles.

(A) Minutes
(B) Time
(C) Later
(D) Quickly
(E) Since
(F) Seconds

(a) (B) and (E)
(b) (A) and (C)
(c) (A) and(F)
(d) (B) and (D)
(e) (C) and (E)

22. The Stales should take steps to the process of teachers" appointments as the Centre has already sanctioned six lakh posts.

(A) fasten
(B) move
(C) hasten
(D) speed
(E) early
(F) quicken

(a) (D) and (F)
(b) (A) and (C)
(c) (C) and (F)
(d) (D) and (E)
(e) (B) and (D)

23. A senior citizen's son threatened her every day and physically harmed her, forcing her to transfer her properly to him.

(A) superficially
(B) mistakenly
(C) allegedly
(D) miserably
(E) doubtfully
(F) purportedly

(a) (C) and (F)
(b) (A) and (E)
(c) (C) and (E)
(d) (D) and (F)
(e) (A) and (C)

24. Medical teachers said that the management had continued o remain to their cause leading to the stretching of their strike.

(A) unmoved
(B) lethargic
(C) unconcerned
(D) apathetic
(E) indifferent (F) bored

(a) (B) and (C)
(b) (C) and (F)
(c) (A) and (E)
(d) (A) and (D)
(e) (D) and (E)

25. The parents had approached the high court to the government order after their children, who passed UKG, were denied admission by a school.

(A) void
(B) quash
(C) annual
(D) stay
(E) lift
(F) post

(a) (A) and (D)
(b) (B) and (C)
(c) (C) and (E)
(d) (E) and (F)
(e) (C) and (D)

IBPS / SBI Special TX: 
Subject: 
General: 

IBPS (PO) Previous Year Exam Paper - 2013 "Computer Knowledge"

IBPS logo


IBPS (PO) Previous Year Exam Paper - 2013

Subject:  Computer Knowledge


181. Macros stored in the global macro sheet can be used

(1) in the current document only
(2) in any document
(3) can be used only with other macros of  the global macro sheet
(4) not consistent behaviour
(5) None of the above

182. About pasting from the clip board

(1) a part of the clip board contents can be pasted
(2) whole of the contents of clip board can be pasted
(3) sometimes (1) and sometimes (2)
(4) (1) and (2)
(5) None of the above

183. One of the following statements is not true for BUFFERS command

(1) Increasing numbers of BUFFERS can speed program execution, but only to a certain extent
(2) The more buffers that exist the more sectors can be stored in memory; hence fewer accesses of disk are necessary
(3) The BUFFERS command is used to establish the number of disk buffers set up by MS-DOS during booting
(4) All of the above
(5) None of the above

184. EPROM can be used for

(1) erasing the contents of ROM
(2) reconstructing the contents of ROM
(3) erasing and reconstructing the contents of ROM
(4) duplicating the ROM
(5) None of the above

185. Attributes can be defined for

(1) entity
(2) switch board
(3) macro
(4) pages
(5) None of the above

186. Where will we find the referential integrity command ?

(1) Tools
(2) View
(3) Format
(4) Table
(5) None of these

187. Anything that is typed in a worksheet appears

(1) in the formula bar only
(2) in the active cell only
(3) in both active cell and formula bar
(4) in the formula bar first and when we press ENTER it appears in active cell
(5) None of the above

188. Which bar is usually located below the Title Bar that provides categorised options ?

(1) Menu Bar (2) Status Bar
(3) Toolbar (4) Scroll Bar
(5) None of the above

189. A pixel is

(1) a computer program that draws picture
(2) a picture stored in the secondary memory
(3) the smallest resolvable part of a picture
(4) a virus
(5) None of the above

190. How many types of cell references are available in Excel ?

(1) 3
(2) 4
(3) 8
(4) 10
(5) None of these

DOWNLOAD IBPS PO Question Papers PDF

DOWNLOAD IBPS CLERK Question Papers PDF

DOWNLOAD ALL BANK EXAMS PAPERS E-BOOKS

Printed Study Material for IBPS PO Exam

IBPS / SBI Special TX: 
General: 
STUDY KITS: 

IBPS (PO) Previous Year Exam Paper - 2013 "General Awareness"

IBPS logo


IBPS (PO) Previous Year Exam Paper - 2013

Subject:  General Awareness


141. Which of the following is a receipt listed in India and traded in rupees declaring ownership of shares of a foreign company?

(1) Indian Depository Receipt (IDR)
(2) European Depository Receipt (EDR)
(3) Global Depository Receipt (GDR)
(4) American Depository Receipt (ADR)
(5) Luxemburg Depository Receipt (LDR)

142. A bank without any branch network that offers its services remotely through online banking, telephone/mobile banking and interbank ATM network alliances is known as

(1) Universal Banking
(2) Indirect Bank
(3) Door Step Bank
(4) A Direct Bank
(5) Unit Banking

143. Which of the following Indian Universities is Asia’s largest residential university?

(1) Allahabad University
(2) Utkal University
(3) Banaras Hindu University
(4) Anna University
(5) Jawaharlal Nehru University

144. In October, 2013, which country has confirmed plans to create a secure mail service to protect its citizens and businesses against foreign espionage?

(1) Mexico
(2) Brazil
(3) Sweden
(4) Germany
(5) None of these

145. The campaign name ‘Heal India’ aims to create awareness about which of the following diseases?

(1) Mental illness
(2) AIDS
(3) Leprosy
(4) Alzheimer
(5) None of these

146. The target set by the UIDAI for issuance of Aadhaar cards upto 2014 is

(1) 50 crore cards
(2) 55 crore cards
(3) 45 crore cards
(4) 40 crore cards
(5) 60 crore cards

147. Which of the following nations has signed a comprehensive free trade agreement with European Union?

(1) Japan
(2) China
(3) Russia
(4) Canada
(5) None of these

DOWNLOAD IBPS PO Question Papers PDF

DOWNLOAD IBPS CLERK Question Papers PDF

DOWNLOAD ALL BANK EXAMS PAPERS E-BOOKS

Printed Study Material for IBPS PO Exam

IBPS / SBI Special TX: 
General: 

IBPS (PO) Previous Year Exam Paper - 2013 "English Language"

 

IBPS logo

IBPS (PO) Previous Year Exam Paper - 2013

Subject:  English Language

Directions (Q. Nos. 51-58) Read the following passage carefully and answer the questions given below it. Certain words have been printed in bold to help you to locate them while answering some of the questions.

The great fear in Asia a short while ago was that the region would suffer through the wealth destruction already taking place in the U.S as a result of the financial crisis. Stock markets tumbled as exports plunged and economic growth deteriorated. Lofty property prices in China and elsewhere looked set to bust as credit tightened and buyers evaporated. But with surprising speed, fear in Asia swung back to greed as the region shows signs of recovery and property and stock prices are soaring in many parts of Asia. Why should this sharp Asian turnaround be greeted with skepticism? Higher asset prices mean households feel wealthier and better able to spend, which could further fuel the region’s nascent rebound. But just as easily, Asia could soon find itself saddled with overheated markets similar to the U.S. housing market.

In short, the world has not changed, it has just moved placed. The incipient bubble is being created by government policy. In response to the global credit crunch of 2008. Policy makers in Asia I slashed interest rates and flooded financial sectors with cash in frantic attempts to keep loans flowing and economies growing. These steps were logical for central bankers striving to reverse a deepening economic crisis. But there is evidence that there is too much easy money around. It’s winding up in stocks and real estate, pushing prices up too far and too fast for the undenying economic fundamentals. Much of the concern is focused on China where government stimulus efforts have been large and effective, Money in China has been especially easy to find. Aggregate new bank lending surged 201% in first half of 2009 from the same period a year earlier, to nearly 51.1 turn on. Exuberance over a quick recovery which was given a boost by China’s surprisingly strong 7.9% GDP growth in the second quarter has buoyed investor sentiment not just for stocks but also for real estate.

Former U.S. Federal Reserve Chairman Alan Greenspan argued that bubbles could only be recognised in hand sight. But investors who have been well schooled in the dangers of bubbles over the past decade are increasingly wary that prices have risen too far and that the slightest bit of negative, economic news could knock markets for a loop. These fears are compounded by the possibility that Asia’s central bankers will begin taking stops to shut off the money. Rumours that Beijing was on the verge of tightening credit led to Shanghai stocks plunging 5%. Yet many economists believe that, there is close to a zero possibility that the Chinese \ government will do anything this year that constitutes tightening. And without a major shift in thinking, the easy-money conditions will stay in place. In a global economy that has produced more dramatic ups and downs than anyone thought possible over the past two years. Asia may be neading for another disheartening plunge.

51. To which of the following has the author attributed the 2008 Asian financial crisis?

(A) Reluctance or Asian governments to taper off the economic stimulus.
(B) Greed of Asian investors causing them to trade 1 stocks of American companies at high prices.
(C) Inflated real estate prices in Asian countries.

(1) None
(2) Only (A)
(3) Only (C)
(4) (A) and (B)
(5) Only (B)

Download Free E-Books for IBPS, Bank Exams

Buy Printed Study Material for IBPS, SBI Exams

52. What does the author want to convey through the phrase “The world has not changed it has just moved places”?

(1) At present countries are more dependent on Asian economies than on the US economy
(2) Economies have become interlinked on account of globalisation
(3) Asian governments are implementing the same economic reforms as developed countries
(4) All economies are susceptible to recession because of the state of the US economy
(5) None of the above

53. Which of the following can be said about the Chinese government’s efforts to revive the economy?

(1) These were largely unsuccessful as only the housing market improved
(2) The governments only concern was to boost investor confidence in stocks
(3) These efforts were ineffectual as-the economy recovered owing to the US market stabilising
(4) These were appropriate and accomplished the goal of economic revival
(5) They blindly imitated the economic reforms adopted by the US

54. Why do experts predict that Asian policymakers will not withdraw fiscal stimulus?

(A) The US economy is not likely to recover for a long time.
(B) Stock markets are yet to regain their former levels.
(C) Fear of revolt by greedy citizens.
(1) None of these
(2) Only (C)
(3) (A) and (C)
(4) Only (B)
(5) (B) and (C)

55. What do the statistics about loans given by Chinese banks in 2009 indicate?

(1) There was hardly any demand for loans in 2008
(2) The Chinese government has borrowed funds from the US
(3) China will take longer than the US to recover from the economic crisis
(4) The GDP of China was below expectations
(5) None of the above

56. Why has investor confidence in the Chinese stock market been restored?

(A) Existing property prices which are stable and affordable.
(B) The government has decided to tighten credit.
(C) Healthy growth of the economy indicated by GDP figures.

(1) Only (C)
(2) (A) and (B)
(3) All (A), (B) and (C)
(4) Only (B)
(5) None of these

57. What is the author’s main objective in writing the passage?

(1) Illustrating that Asian economies are financially more sound than those of developed countries
(2) Disputing financial theories about how recessions can be predicted and avoided
(3) Warning Asian countries about the dangers of favouring fast growth and profits over sound economic-principles
(4) Extolling China’s incredible growth and urging other countries to emulate it
(5) Advising governments about the changes in policy to strengthen economic fundamentals

58. Why does the author doubt the current resurgence of Asian economics?

(1) Their economies are too heavily reliant on the American economy which is yet to recover
(2) Central banks have slashed interest rates too abruptly which is likely to cause stock markets to crash
(3) With their prevailing economic conditions they are at risk for a financial crisis
(4) Their GDP has not grown significantly during the last financial year
(5) None of the above


Directions (Q.Nos. 59-65) Read the following passage carefully and answer the questions given below it. Certain words have been printed in bold to help you to locate them while answering some of the questions.

Delays of several months in National Rural Employment Guarantee Scheme (NREGS) wage payments and work sites where labourers have lost all hope of being paid at all have become the norm in many states. How are workers who exist on the margins of subsistence supposed to feed their families? Under the scheme, workers must be paid within 15 days, failing which they are entitled, to compensation under the Payment of Wages Act – upto 3000 per aggrieved worker. In reality, compensation is received in only a few isolated instances. It is often argued by officials that the main reason for the delay is the inability of banks and post offices to handle mass payments of NREGS wages. Though there is a grain of truth in this, as a diagnosis it is misleading. The ‘jam’ in the banking system has been the result of the hasty switch to bank payments imposed by the Central Government against the recommendation of the Central Employment Guarantee Council which advocated a gradual transition starting with villages relatively close to the nearest bank. However, delays are not confined solely to the banking system. Operational hurdles include implementing agencies taking more than fifteen days to issue payment orders, viewing of work measurement as a cumbersome process resulting in procrastination by the engineering staff and non maintenance of muster rolls and job card etc. But behind these delays lies a deeper and deliberate ‘backlash’ against the NREGS. With bank payments making it much harder to embezzle NREGS funds, the programme is seen as a headache by many government functionaries the workload has remained without the “inducements”. Slowing down wage payments is a convenient way of sabotaging the scheme because workers will desert NREGS worksites.

The common sense solution advocated by the government is to adopt the business correspondent model. Where in bank agents will go to villages to make cash payments and duly record them on handheld, electronic devices. This solution is based on the wrong diagnosis that distance separating villages from banks is the main issue. In order to accelerate payments, clear timelines for every step of the payment process should be incorporated into the system as Programme Officers often have no data on delays and cannot exert due pressure to remedy the situation. Workers are both clueless and powerless with no provision for them to air their grievances and seek redress. In drought affected areas the system of piece rate work can be dispensed with where work measurement is not completed within a week and wages may be paid on the basis of attendance. Buffer funds can be provided to gram panchayats and post offices to avoid bottlenecks in the flow of funds. Partial advances could also be considered provided wage payments are meticulously tracked. But failure to recognise problems and unwillingness to remedy them will remain major threats to the NREGS.
 

59. What impact have late wage payments had on NREGS workers?

(1) They cannot obtain employment till their dues are cleared
(2) They have benefited from the compensation awarded to them
(3) They have been unable to provide for their families
(4) They have been ostracised by their families who depend on them for sustenance
(5) None of the above

60. Which of the following factors has not been responsible for untimely payment of NREGS wages?

(1) Communication delays between agencies implementing the scheme
(2) Improper record keeping
(3) Behind schedule release of payments by banks
(4) Drought conditions prevalent in the country
(5) Delays in work measurement

61. What has the outcome of disbursing NREGS wages through banks been?

(1) Theft of funds by administration officials responsible for the scheme has reduced
(2) Increased work load for local government officials
(3) Protests by workers who have to travel long distances to the nearest bank to claim their wages
(4) Time consuming formalities have to be completed by workers
(5) None of the above

62. According to the passage, which of the following has/have been the consequence (s) of delayed wage payments?

(A) Compensation to victimised workers has amounted to crores.
(B) Banks will no longer be entrusted with remitting wages.
(C) Regulations to ensure punctual wage payments have come into force.

(1) None of these
(2) Only (A)
(3) (A) and (C)
(4) (A) and (B)
(5) (B) and (C)

63. To which of the following has the author attributed the delay in wage payments?

(1) Embezzlement of funds by corrupt bank staff
(2) Lack of monitoring by the Central Employment Guarantee Council
(3) An attempt to derail the NREGS by vested interests
(4) Overworked bank staff deliberately delay payments to protest against extra work
(5) Engineers’ efforts to wreck the NREGS because of low wages

64. Which of the following is NOT true in the context of the passage?

(A) Workers are reluctant to open bank accounts as branches are not conveniently located.
(B) Local officials often delay wage payments in drought prone areas to benefit workers.
(C) The Government has not implemented every recommendation of the Central Employment Guarantee Council.

(1) Only(B)
(2) (A) and (B)
(3) (B) and (C) (4) (A) and (C)
(5) All of these

65. Which of the following can be considered a deficiency in the NREGS?

(1) Lack of co-ordination among Programme Officers
(2) Local officials are unaware of correct operational procedures
(3) Workers have no means of obtaining redressal for untimely wage payments
(4) Disbursing wages through banks instead of readily accessible post offices
(5) The Central Employment Guarantee Council is reluctant to award compensation to workers
Directions (Q. No. 66-70) Rearrange the following sentences (A), (B), (C), (D) (E) and (F) into a meaningful paragraph and then answer the questions given below it.
(A) Moreover salaries in public sector enterprises are not as competitive as those offered by private or foreign corporates.
(B) This trend should be a wake up call for stakeholders to examine why employees are seeking better opportunities with private companies in India and abroad.
(C) Public Sector Enterprises (PSEs) have been experiencing severe challenges in attracting motivating and retaining their key staff.
(D) Having identified these as the reasons employees leave PSEs it is important empower stakeholders to find ways to remedy the situation.
(E) One reason is that young employees lured away to private firms are more willing to undertake professional risks.
(F) Employees in specialist roles especially have become increasingly difficult to retain.

DOWNLOAD IBPS PO Question Papers PDF

DOWNLOAD IBPS CLERK Question Papers PDF

DOWNLOAD ALL BANK EXAMS PAPERS E-BOOKS

Printed Study Material for IBPS PO Exam

IBPS / SBI Special TX: 
General: 
STUDY KITS: 

IBPS (PO) Previous Year Exam Paper - 2013 "Reasoning"

IBPS logo

IBPS (PO) Previous Year Exam Paper - 2013

Subject:  Reasoning


Directions (Q.No. 1-5) Study the following information carefully and answer the questions given below.

An organization wants to recruit system analysts. The following conditions apply.

The candidate must

(i) be an engineering graduate in computer/IT. with at least 60% marks.
(ii) have working experience in the field of computer at least for 2 yr after acquiring the requisite qualification,
(iii) have completed minimum 25 yr and maximum 30 yr of age As on 1.12.2013.
(iv) be willing to sign a bond for Rs 50000.
(v) have secured minimum 55% marks in selection test.

However, if a candidate fulfils all other conditions Except

(a) at (i) above, but is an Electronics Engineer with 65% or more marks the case is to be referred to the General Manager (GM)-IT.
(b) at (iv) above, but has an experience of atleast 5 yr as a Software Manager, the case is to be referred to the VP.

In each question below, detailed information of candidate is given. You have to carefully study the information provided in each case and take one of the following courses of actions based on the information and the conditions given above. You are not to assume anything other than the information provided in each question. All these cases are given to you as on 01.12.2013. You have to indicate your decision by marking answers to each question as follows

Give Answer

(1) If the case is to be referred to VP
(2) If the case is to be referred to GM
(3) If the data provided is not sufficient to take a decision
(4) If the candidate is to be selected
(5) If the candidate is not to be selected

1. Ms. Suneeta is an IT Engineer with 60% marks at graduation as well as in selection test. She is working as a Software Engineer for last 3 yr after completing engineering degree and has completed 27 yr of age. She is willing to sign the bond of Rs 50000.

2. Rakesh Rao is a Computer Engineer Graduate and thereafter is working as a Software Manager for last 6 yr. He has secured 72% marks at graduation and 67% marks in selection test. His date of birth is 5th December, 1984. He is not willing to sign the bond for Rs50000.

3. Ramkumar is an Engineering graduate in computers with 78% marks passed out in 2007 at the age of 23 yr. Since, then he is working as a Software Manager in an engineering firm. He doesn’t want to sign the bond for Rs 50000. He has cleared the selection test with 72% marks.

4. Nishant is an Electronics Engineer passed out in June, 2010 at the age of 22 yr. Since, then he is working as a Programmer in a software company. He has passed the selection test with 66% marks and is willing to sign the bond.

5. Kalyani is an Engineer with 72% marks in Telecommunication. She has just completed 27 yr of age. She has cleared the selection test with 59% marks. She is willing to sign the bond.

Direction (Q.No. 6) Analyse the following passage and answer the question.


Some words are highly inflammable. Fusion is one of them. You can get two sets of people into a war mode by just uttering the words ‘fusion music’. One set will breathe fire and say it violates the purity of music the other set will tell you earnestly that it opens up the borders of music.

6. From the purists perspective, the ‘war’ between the two set of people can best be

(1) categorized as an ideological conflict between two ideas
(2) termed as a conflict between generations the younger versus the older generation
(3) an attempt to preserve the core principles
(4) seen as an attempt of people at the margin to occupy centre stage
(5) seen as preserving the social identity of purists
 

7. Unlike other retail outlets, where items are purchased in any number of units the customer wants, in super-markets items are grouped in bulk packages. This bulk buying offers saving to the customer. The option to buy at wholesale prices by buying in bulk makes super-market a “practical choice for budget-conscious consumers.

Which of the following assumption may be drived from the above information.
 

(1) Super-markets often have greater buying power and lower overhead costs, so they can offer a greater variety of products than regular retail outlets
(2) Super-markets are often more conveniently located and have better parking facilities
(3) The emergence of super-markets has caused many small retail stores to close down and thus eliminate competitions
(4) It is economically wise to buy single items since bulk packages seldom offer significant savings
(5) The financial savings from purchasing bulk packages may outweigh the inconvenience of being unable to purchase in any number of units that suits the customers’ need

8. Nations do not complete with each other in the way corporations do.

Which of the following most favours the weakness of the argument ?

(1) Trade deficit is a sign of national strength, profits are a sign of corporate strength
(2) Increase in human development index improves national standing, increase in market share improves corporate standing
(3) Climate change negotiations lead to global improvement; CSR initiatives lead to image improvement
(4) Nations go to war to capture territory, corporates contend against each other to capture market share
(5) None of the above

9. Civilization has taught us to be friendlier towards one another
Which of the following most favours the strengthens of the argument?

(1) Cats are loyal to their children, whereas men are loyal to their communities
(2) Elephants move in a herd, whereas men live in nuclear families
(3) Lions protect their own territories, whereas men capture other men’s territories
(4) Nilgai and Cheetal stay together, whereas men of one race dominate another
(5) None of the above

10. The mushrooming of business schools in the country is a cause for shortage of faculty with Ph.D qualification. In addition, the higher pay and generous fringe benefits given by industry has encouraged qualified people to not seek academic positions.
Which of the following statements, if true, would tend to STRENGTHEN the argument?

(1) The average salary for industry positions in Gujarat is more than the average salary for faculty positions in some business schools in Ahmedabad by around 30%
(2) The average salary for industry positions in Gujarat is less than the average salary for faculty positions in a top business school in Ahmedabad by around 30%
(3) The average salary for recent Ph.D graduates in the industry is 20% higher than that in academics
(4) The rate of growth of salaries for the industry positions has been higher than the rate of growth of salaries for academic positions for the past three years
(5) None of the above


Directions (Q. Nos 11 -13) Study the information given below carefully to answer the following questions.
In a certain code language the following lines written as

 

‘lop eop aop fop’ means ‘Traders are above laws’
‘fop cop bop gop’ means ‘Developers were above profitable’
‘aop bop uop qop’ means ‘Developers stopped following traders’
‘cop jop eop uop’ means ‘Following maps were laws’

11. ‘Developers are following laws’ would be correctly written as

(1) ‘bop cop uop eop’
(2) ‘lop bop eop uop’
(3) ‘oup cop lop aop’
(4) ‘gop cop uop qop’
(5) None of these

12. ‘qop gop cop eop’ would correctly mean

(1) profitable laws were stopped
(2) developers stopped following laws
(3) traders were above profitable
(4) were laws profitable traders
(5) None of the above

Click Here To Download Full Paper

DOWNLOAD IBPS PO Question Papers PDF

DOWNLOAD IBPS CLERK Question Papers PDF

DOWNLOAD ALL BANK EXAMS PAPERS E-BOOKS

Printed Study Material for IBPS PO Exam

13. ‘aop qop bop’ would correctly mean

(1) following were above
(2) traders stopped developers
(3) developers are laws
(4) traders above stopped
(5) laws are stopped

Directions (Q.Nos. 14-18) In each of the questions below are given four statements followed by three conclusions numbered I, II and III. You have to take the given statements- to be true even if they seem to be at variance from commonly known facts. Read all the conclusions and then decide which of the given conclusions logically follows from the given statements disregarding commonly known facts.

14. Statements All petals are flowers. Some flowers are buds. Some buds are leaves. All leaves are plants.

Conclusions

I. Some petals are not buds.
II. Some flowers are plants.
III. No flower is plant.

(1) Only I follows
(2) Either II or III follows
(3) I and II follow
(4) Only III follows
(5) None of the above

15. Statements Some pens are keys. Some keys are locks. All locks are cards. No card is paper.

Conclusions

I. No lock is paper.
II. Some cards are keys.
III. Some keys are not paper.

(1) I and II follow
(2) Only I follows
(3) Only II follows
(4) All follow
(5) None follows

16. Statements Some pearls are gems. All gems are diamonds. No diamond is stone. Some stones are corals.

Conclusions

I. Some stones are pearls.
II. Some corals being diamond is a possibility.
III. No stone is pearl.

(1) Only I follows
(2) Only II follows
(3) Either I or III follows
(4) I and II follow
(5) None of these

17. Statements Some apartments are flats. Some flats are buildings. All buildings are bungalows. All bungalows are gardens.

Conclusions

I. All apartments being building is a possibility
II. All bungalows are not buildings.
III. No flat is garden.
(1) None follows
(2) Only I follows
(3) Either I or III follows
(4) II and III follow
(5) Only II follows
 

18.  Statements All chairs are tables. All tables are bottles. Some bottles are jars. No jar is bucket.

Conclusions

 I. Some tables being jar is a possibility.
II. Some bottles are chairs.
III. Some bottles are not bucket.
(1) Only I follows (2) I and II follow
(3) All follow (4) Only II follows
(5) None of these

19. A person starts from point P in East and moves 12 m to point Q. Then, he moves right 8 m to point R. Again he moves right for 6m to point S. Then, he moves 6 m in the North to point T. Finally from there he goes to left for 6 m to point U. Which of three point he would form a triangle whose all the angles are less than 90°.

(1) PTQ
(2) QTR
(3) UTS
(4) TSR
(5) SQR


Directions (Q.Nos. 20-25) Read the following information carefully and answer the questions that follow.
Seven friends A, B, C, D, E, F and G are sitting around a circular table facing either the centre or outside. Each one of them belongs to a different department viz. Finance, Marketing Sales, HR, Corporate Finance, Investment Banking and Operations but not necessarily in the same order.
C sits third to the right of G.C faces the centre. Only one person sits between C and the person working in the HR department immediate neighbours of C face outside. Only one person sits between F and D. Both F and D face the centre. D does not work in the HR department. A works in Investment Banking Department. A faces the centre. Two people sit between the persons who work in Investment Banking and Marketing Departments. The person who works in Corporate Finance sits to the immediate left of E. C faces same direction as E. The person who works in corporate finance sits to the immediate left of the person who works for Operations department.

 

20. For which of the following departments does B work?

(1) Finance
(2) Marketing
(3) HR
(4) Corporate Finance
(5) Operations

21. What is position of B with respect to the person who works for Sales department ?

(1) Immediate right
(2) Third to the left
(3) Second to the right
(4) Second to the left
(5) Fourth to the right

22. Who sits to the immediate right of E ?

(1) The person who works for Marketing department
(2) C
(3) B
(4) The person who works for HR department
(5) A

23. Who amongst the following sits exactly between C and the person who works for HR department?

(1) B
(2) The person who works for Marketing department
(3) The person who works for Operations department
(4) D
(5) G

24. Who amongst the following sit between the persons who work for Marketing and Investment banking departments when counted for the left hand side of the person working for Marketing department?

(1) F and G
(2) E and C
(3) C and B
(4) F and D
(5) B and D

IBPS / SBI Special TX: 
Bank/Organisation: 
General: 
STUDY KITS: 

(Papers) IBPS PO Exam Paper - 2013 "Held on: 27-10-2013" ::REASONING ABILITY::

IBPS logo


(Papers) IBPS PO Exam Paper - 2013 "Held on: 27-10-2013"

::REASONING ABILITY::


DIRECTIONS (Qs. 1-5) : Study the following information carefully and answer the questions given below:

An organization wants to recruit system analysts. The following conditions apply. The candidate must

(i) be an engineering graduate in computer/IT with at least 60% marks.
(ii) have working experience in the field of computer at least for 2 yr after acquiring the requisite qualification.
(iii) have completed minimum 25 yr and maximum 30 yr of age as on 1.12.2013.
(iv) be willing to sign a bond for ` 50000.
(v) have secured minimum 55% marks in selection test. However, if a candidate fulfils all other conditions Except

A. at (i) above, but is an Electronics Engineer with 65% or more marks the case is to be referred to the General Manager (GM)-IT.
B. at (iv) above, but has an experience of atleast 5 yr as a Software Manager, the case is to be referred to the VP.

In each question below, detailed information of candidate is given. You have to carefully study the information provided in each case and take one of the following courses of actions based on the information and the conditions given above. You are not to assume anything other than the information provided in each question. All these cases are given to you as on 01.12.2013. You have to indicate your decision by marking answers to each question as follows:

(a) If the case is to be referred to VP
(b) If the case is to be referred to GM
(c) If the data provided is not sufficient to take a decision
(d) If the candidate is to be selected
(e) If the candidate is not to be selected

1. Ms. Suneeta is an IT Engineer with 60% marks at graduation as well as in selection test. She is working as a Software Engineer for last 3 yr after completing engineering degree and has completed 27 yr of age. She is willing to sign the bond of ` 50000.

2. Rakesh Rao is a Computer Engineer Graduate and thereafter is working as a Software Manager for last 6 yr. He has secured 72% marks at graduation and 67% marks in selection test. His date of birth is 5th December, 1984. He is not willing to sign the bond for ` 50000.

3. Ramkumar is an Engineering graduate in computers with 78% marks passed out in 2007 at the age of 23 yr. Since, then he is working as a Software Manager in an engineering firm. He doesn't want to sign the bond for ` 50000. He has cleared the selection test with 72% marks.

4. Nishant is an Electronics Engineer passed out in June, 2010 at the age of 22 yr. Since, then he is working as a Programmer in a software company. He has passed the selection test with 66% marks and is willing to sign the bond.

5. Kalyani is an Engineer with 72% marks in Telecommunication. She has just completed 27 yr of age. She has cleared the selection test with 59% marks. She is willing to sign the bond.

DIRECTIONS (Qs. 6) : Analyse the following passage and answer the question.

Some words are highly inflammable. Fusion is one them. You can get two sets of people into a war mode by just uttering the words 'fusion music'. One set will breathe fire and say it violates the purity of music the other set will tell you earnestly that it opens up the borders of music.

6. From the purists perspective, the 'war' between the two set of people can best be

(a) categorized as an ideological conflict between two ideas
(b) termed as a conflict between generations the younger versus the older generation
(c) an attempt to preserve the core principles
(d) seen as an attempt of people at the margin to occupy centre stage
(e) seen as preserving the social identity of purists

7. Unlike other retail outlets, where items are purchased in any number of units the customer wants, in super-markets items are grouped in bulk packages. This bulk buying offers saving to the customer. The option to buy at wholesale prices by buying in bulk makes super-market a practical choice for budget-conscious consumers. Which of the following assumption may be drived from the above information.

(a) Super-markets often have greater buying power and lower overhead costs, so they can offer a greater variety of products than regular retail outlets
(b) Super-markets are often more conveniently located and have better parking facilities
(c) The emergence of super-markets has caused many small retail stores to close down and thus eliminate competitions
(d) It is economically wise to buy single items since bulk packages seldom offer significant savings
(e) The financial savings from purchasing bulk packages may outweigh the inconvenience of being unable to purchase in any number of units that suits the customers' need

8. Nations do not complete with each other in the way corporations do. Which of the following most favours the weakness of the argument?

(a) Trade deficit is a sign of national strength, profits are a sign of corporate strength
(b) Increase in human development index improves national standing, increase in market share improves corporate standing
(c) Climate change negotiations lead to global improvement; CSR initiatives lead to image improvement
(d) Nations go to war to capture territory, corporates contend against each other to capture market share
(e) None of the above

9. Civilization has taught us to be friendlier towards one another Which of the following most favours the strengthens of the argument?

(a) Cats are loyal to their children, whereas men are loyal to their communities
(b) Elephants move in a herd, whereas men live in nuclear families
(c) Lions protect their own territories, whereas men capture other men's territories
(d) Nilgai and Cheetal stay together, whereas men of one race dominate another
(e) None of the above

10. The mushrooming of business schools in the country is a cause for shortage of faculty with Ph.D qualification. In addition, the higher pay and generous fringe benefits given by industry has encouraged qualified people to not seek academic positions. Which of the following statements, if true, would tend to STRENGTHEN the argument?

(a) The average salary for industry positions in Gujarat is more than the average salary for faculty positions in some business schools in Ahmedabad by around 30%
(b) The average salary for industry positions in Gujarat is less than the average salary for faculty positions in a top business school in Ahmedabad by around 30%
(c) The average salary for recent Ph. D graduates in the industry is 20% higher than that in academics
(d) The rate of growth of salaries for the industry positions has been higher than the rate of growth of salaries for academic positions for the past three years
(e) None of the above

DIRECTIONS (Qs. 11-13) : Study the information given below carefully to answer the following questions

In a certain code language the following lines written as:
'lop eop aop fop' means 'Traders are above laws'
'fop cop bop gop' means 'Developers were above profitable'
'aop bop uop qop' means 'Developers stopped following traders'
'cop jop eop uop' means 'Following maps were laws'

11. 'Developers are following laws' would be correctly written as

(a) 'bop cop uop eop'
(b) 'lop bop eop uop'
(c) 'oup cop lop aop'
(d) 'gop cop uop qop'
(e) None of these

12. 'qop gop cop eop' would correctly mean

(a) profitable laws were stopped
(b) developers stopped following laws
(c) traders were above profitable
(d) were laws profitable traders
(e) None of the above

13. 'aop qop bop' would correctly mean

(a) following were above
(b) traders stopped developers
(c) developers are laws
(d) traders above stopped
(e) laws are stopped

DIRECTIONS (Qs. 14-18) : In each of the questions below are given four statements followed by three conclusions numbered I, II and III. You have to take the given statements to be true even if they seem to be at variance from commonly known facts. Read all the conclusions and then decide which of the given conclusions logically follows from the given statements disregarding commonly known facts.

14. Statements:

All petals are flowers. Some flowers are buds.
Some buds are leaves. All leaves are plants.

Conclusions:

I. Some petals are not buds.
II. Some flowers are plants.
III. No flower is plant.

(a) Only I follows
(b) Either II or III follows
(c) I and II follow
(d) Only III follows
(e) None of the above

15. Statements:

Some pens are keys. Some keys are locks.
All locks are cards. No card is paper

Conclusions:

I. No lock is paper.
II. Some cards are keys.
III. Some keys are not paper.

(a) I and II follow
(b) Only I follows
(c) Only II follows
(d) All follow
(e) None follows

16. Statements:

Some pearls are gems. All gems are diamonds.
No diamond is stone. Some stones are corals.

Conclusions:

I. Some stones are pearls.
II. Some corals being diamond is a possibility.
III. No stone is pearl.

(a) Only I follows
(b) Only II follows
(c) Either I or III follows
(d) I and II follow
(e) None of these

17. Statements:

Some apartments are flats. Some flats are buildings.
All buildings are bungalows. All bungalows are gardens.

Conclusions:

I. All apartments being building is a possibility
II. All bungalows are not buildings.
III. No flat is garden.

(a) None follows
(b) Only I follows
(c) Either I or III follows
(d) II and III follow
(e) Only II follows

18. Statements:

All chairs are tables. All tables are bottles.
Some bottles are jars. No jar is bucket.

Conclusions:

I. Some tables being jar is a possibility.
II. Some bottles are chairs.
III. Some bottles are not bucket.

(a) Only I follows
(b) I and II follow
(c) All follow
(d) Only II follows
(e) None of these

19. A person starts from point P in East and moves 12 m to point Q. Then, he moves right 8 m to point R. Again he moves right for 6 m to point S. Then, he moves 6 m in the North to point T. Finally from there he goes to left for 6 m to point U. Which of three point he would form a triangle whose all the angles are less than 90°?

(a) PTQ
(b) QTR
(c) UTS
(d) TSR
(e) SQR

DIRECTIONS (Qs. 20-25) : Read the following information carefully and answer the questions that follow.

Seven friends A, B, C, D, E, F and G are sitting around a circular table facing either the centre or outside. Each one of them belongs to a different department viz. Finance, Marketing Sales, HR, Corporate Finance, Investment Banking and Operations but not necessarily in the same order. C sits third to the right of G.G faces the centre. Only one person sits between C and the person working in the HR department immediate neighbours of C face outside. Only one person sits between F and D. Both F and D face the centre. D does not work in the HR department. A works in Investment Banking Department. A faces the centre. Two people sit between the persons who work in Investment Banking and Marketing Departments. The person who works in Corporate Finance sits to the immediate left of E. C faces same direction as E. The person who works in corporate finance sits to the immediate left of the person who works for Finance department.

20. For which of the following departments does B work?

(a) Finance
(b) Marketing
(c) HR
(d) Corporate Finance
(e) Operations

21. What is position of B with respect to the person who works for Sales department?

(a) Immediate right
(b) Third to the left
(c) Second to the right
(d) Second to the left
(e) Fourth to the right

22. Who sits to the immediate right of E ?

(a) The person who works for Marketing department
(b) C
(c) B
(d) The person who works for HR department
(e) A

23. Who amongst the following sits exactly between C and the person who works for HR department?

(a) B
(b) The person who works for Marketing department
(c) The person who works for Operations department
(d) D
(e) G

24. Who amongst the following sit between the persons who work for Marketing and Investment Banking departments when counted for the left hand side of the person working for Marketing department?

(a) F and G
(b) E and C
(c) C and B
(d) F and D
(e) B and D

25. How many people sit between the person who works for Operations department and A, when counted from the right hand side of A?

(a) One
(b) Two
(c) Three
(d) Four
(e) More than four

DOWNLOAD IBPS PO Question Papers PDF

DOWNLOAD IBPS CLERK Question Papers PDF

DOWNLOAD ALL BANK EXAMS PAPERS E-BOOKS

Printed Study Material for IBPS PO Exam

IBPS / SBI Special TX: 
General: 

(Papers) IBPS PO Exam Paper - 2013 "Held on: 27-10-2013" ::ENGLISH LANGUAGE::

IBPS logo

(Papers) IBPS PO Exam Paper - 2013 "Held on: 27-10-2013"

::ENGLISH LANGUAGE::

DIRECTIONS (Qs. 1-8) : Read the following passage carefully and answer the questions given below it. Certain words have been printed in bold to help you to locate them while answering some of the questions.

The great fear in Asia a short while ago was that the region would suffer through the wealth destruction already taking place in the U.S as a result of the financial crisis. Stock markets tumbled as exports plunged and economic growth deteriorated. Lofty property prices in China and elsewhere looked set to bust as credit tightened and buyers evaporated. But with surprising speed, fear in Asia swung back to greed as the region shows signs of recovery and property and stock prices are soaring in many parts of Asia. Why should this sharp Asian turnaround be greeted with skepticism? Higher asset prices mean households feel wealthier and better able to spend, which could further fuel the region's nascent rebound. But just as easily, Asia could soon find itself saddled with overheated markets similar to the U.S. housing market. In short, the world has not changed, it has just moved placed. The incipient bubble is being created by government policy. In response to the global credit crunch of 2008. Policy makers in Asi< slashed interest rates and flooded financial sectors with cash in frantic attempts to keep loans flowing and economies growing. Thesf steps were logical for central bankers striving to reverse a deepening economic crisis. But there is evidence that there is too much easy money around. It's winding up in stocks and real estate, pushing prices up too far and too fast for the undenying economic fundamentals. Much of the concern is focused on China where government stimulus efforts have been large and effective, Money ir China has been especially easy to find. Aggregate new bank lending surged 201% in first half of 2009 from the same period a yea earlier, to nearly 51.1 turn on. Exuberance over a quick recovery which was given a boost by China's surprisingly strong 7.9% GDI growth in the second quarter has buoyed investor sentiment not just for stocks but also for real estate. Former U.S. Federal Reserve Chairman Alan Greenspan argued that bubbles could only be recognised in hand sight. But investor who have been well schooled in the dangers of bubbles over the past decade are increasingly wary that prices have risen too far ant that the slightest bit of negative, economic news could knock markets for a loop. These fears are compounded by the possibility that Asia's central bankers will begin taking stops to shut off the money. Rumours that Beijing was on the verge of tightening cred led to Shanghai stocks plunging 5%. Yet many economists believe that, there is close to a zero possibility that the Chines government will do anything this year that constitutes tightening. And without a major shift in thinking, the easy-money condition will stay in place. In a global economy that has produced more dramatic ups and downs than anyone thought possible over the pas two years. Asia may be neading for another disheartening plunge.

1. To which of the following has the author attributed the 2008 Asian financial crisis?

A. Reluctance or Asian governments to taper off the economic stimulus.
B. Greed of Asian investors causing them to trad stocks of American companies at high prices.
C. Inflated real estate prices in Asian countries.

(a) None
(b) Only (A)
(c) Only (C)
(d) (A) and (B)
(e) Only (B)

2. What does the author want to convey through the phrase "The world has not changed it has just moved places"?

(a) At present countries are more dependent on Asian economies than on the US economy
(b) Economies have become interlinked on account of globalisation
(c) Asian governments are implementing the same economic reforms as developed countries
(d) All economies are susceptible to recession because of the state of the US economy
(e) None of the above

3. Which of the following can be said about the Chinese government's efforts to revive the economy?

(a) These were largely unsuccessful as only the housing market improved
(b) The governments only concern was to boost investor confidence in stocks
(c) These efforts were ineffectual as the economy recovered owing to the US market stabilising
(d) These were appropriate and accomplished the goal of economic revival
(e) They blindly imitated the economic reforms adopted by the US

4. Why do experts predict that Asian policymakers will not withdraw fiscal stimulus?

A. The US economy is not likely to recover for a long time.
B. Stock markets are yet to regain their former levels.
C. Fear of revolt by greedy citizens.

(a) None of these
(b) Only (C)
(c) (A) and (C)
(d) Only (B)
(e) (B) and (C)

5. What do the statistics about loans given by Chinese banks in 2009 indicate?

(a) There was hardly any demand for loans in 2008
(b) The Chinese government has borrowed funds from the US
(c) China will take longer than the US to recover from the economic crisis
(d) The GDP of China was below expectations
(e) None of the above

6. Why has investor confidence in the Chinese stock market been restored?

A. Existing property prices which are stable and affordable.
B. The government has decided to tighten credit.
C. Healthy growth of the economy indicated by GDP figures.

(a) Only (C)
(b) (A) and (B)
(c) All (A), (B) and
(C) (d) Only (B)
(e) None of these

7. What is the author's main objective in writing the passage?

(a) Illustrating that Asian economies are financially more sound than those of developed countries
(b) Disputing financial theories about how recessions can be predicted and avoided
(c) Warning Asian countries about the dangers of favouring fast growth and profits over sound economicprinciples
(d) Extolling China's incredible growth and urging other countries to emulate it
(e) Advising governments about the changes in policy to strengthen economic fundamentals

8. Why does the author doubt the current resurgence of Asian economics?

(a) Their economies are too heavily reliant on the American economy which is yet to recover
(b) Central banks have slashed interest rates too abruptly which is likely to cause stock markets to crash
(c) With their prevailing economic conditions they are at risk for a financial crisis
(d) Their GDP has not grown significantly during the last financial year
(e) None of the above

DIRECTIONS (Qs. 9-15) : Read the following passage carefully and answer the questions given below it. Certain words have been printed in bold to help you to locate them while answering some of the questions.

Delays of several months in National Rural Employment Guarantee Scheme (NREGS) wage payments and work sites where labourers have lost all hope of being paid at all have become the norm in many states. How are workers who exist on the margins of subsistence supposed to feed their families? Under the scheme, workers must be paid within 1 5 days, failing which they are entitled, to compensation under the Payment of Wages Act - upto 3000 per aggrieved worker. In reality, compensation is received in only a few isolated instances. It is often argued by officials that the main reason for the delay is the inability of banks and post offices to handle mass payments of NREGS wages. Though there is a grain of truth in this, as a diagnosis it is misleading. The 'jam' in the banking system has been the result of the hasty switch to bank payments imposed by the Central Government against the recommendation of the Central Employment Guarantee Council which advocated a gradual transition starting with villages relatively close to the nearest bank. However, delays are not confined solely to the banking system. Operational hurdles include implementing agencies taking more than fifteen days to issue payment orders, viewing of work measurement as a cumbersome process resulting in procrastination by the engineering staff and non maintenance of muster rolls and job card etc. But behind these delays lies a deeper and deliberate 'backlash' against the NREGS. With bank payments making it much harder to embezzle NREGS funds, the programme is seen as a headache by many government functionaries the workload has remained without the "inducements". Slowing down wage payments is a convenient way of sabotaging the scheme because workers will desert NREGS worksites. The common sense solution advocated by the government is to adopt the business correspondent model. Where in bank agents will go to villages to make cash payments and duly record them on handheld, electronic devices. This solution is based on the wrong diagnosis that distance separating villages from banks is the main issue. In order to accelerate payments, clear timelines for every step of the payment process should be incorporated into the system as Programme Officers often have no data on delays and cannot exert due pressure to remedy the situation. Workers are both clueless and powerless with no provision for them to air their grievances and seek redress. In drought affected areas the system of piece rate work can be dispensed with where work measurement is not completed within a week and wages may be paid on the basis of attendance. Buffer funds can be provided to gram panchayats and post offices to avoid bottlenecks in the flow of funds. Partial advances could also be considered provided wage payments are meticulously tracked. But failure to recognise problems and unwillingness to remedy them will remain major threats to the NREGS.

9. What impact have late wage payments had on NREGSworkers?

(a) They cannot obtain employment till their dues are cleared
(b) They have benefited from the compensation awarded to them
(c) They have been unable to provide for their families
(d) They have been ostracised by their families who depend on them for sustenance
(e) None of the above

10. Which of the following factors has not been responsible for untimely payment of NREGS wages?

(a) Communication delays between agencies implementing the scheme
(b) Improper record keeping
(c) Behind schedule release of payments by banks
(d) Drought conditions prevalent in the country
(e) Delays in work measurement

11. What has the outcome of disbursing NREGS wages through banks been?

(a) Theft of funds by administration officials responsible for the scheme has reduced
(b) Increased work load for local government officials
(c) Protests by workers who have to travel long distances to the nearest bank to claim their wages
(d) Time consuming formalities have to be completed by workers
(e) None of the above

12. According to the passage, which of the following has/have been the consequence (s) of delayed wage payments?

A. Compensation to victimised workers has amounted to crores.
B. Banks will no longer be entrusted with remitting wages.
C. Regulations to ensure punctual wage payments have come into force.

(a) None of these
(b) Only (A)
(c) (A) and (C)
(d) (A) and (B)
(e) (B) and (C)

13. To which of the following has the author attributed the delay in wage payments?

(a) Embezzlement of funds by corrupt bank staff
(b) Lack of monitoring by the Central Employment Guarantee Council
(c) An attempt to derai1 the NREGS by vested interests
(d) Overworked bank staff deliberately delay payments to protest against extra work
(e) Engineers efforts to wreck the NREGS because of low wages

14. Which of the following is NOT true in the context of the passage?

A. Workers are reluctant to open bank accounts as branches are not conveniently located.
B. Local officials often delay wage payments in drought prone areas to benefit workers.
C. The Government has not implemented ever)' recommendation of the Central Employment Guarantee Council.

(a) Only (B)
(b) (A) and (B)
(c) (B) and (C)
(d) (A) and (C)
(e) All of these

15. Which of the following can be considered a deficiency in the NREGS?

(a) Lack of co-ordination among Programme Officers
(b) Local officials are unaware of correct operational procedures
(c) Workers have no means of obtaining redressal for untimely wage payments
(d) Disbursing wages through banks instead of readily accessible post offices
(e) The Central Employment Guarantee Council is reluctant to award compensation to workers

DIRECTIONS (Qs. 16-20) : Rearrange the following sentences (A), (B), (C), (D) (E) and (F) into a meaningful paragraph and then answer the questions given below it.

(A) Moreover salaries in public sector enterprises are not as competitive as those offered by private or foreign corporates, connection
(B) This trend should be a wake up call for stakeholders to examine why employees are seeking better opportunities with private companies in India and abroad.
(C) Public Sector Enterprises (PSEs) have been experiencing severe challenges in attracting motivating and retaining their key staff.
(D) Having identified these as the reasons employees leave PSEs it is important empower stakeholders to find ways to remedy the situation.
(E) One reason is that young employees lured away to private firms are more willing to undertake professional risks.
(F) Employees in specialist roles especially have become increasingly difficult to retain.

16. Which of the following should be the FIRST sentence after rearrangement?

(a) A
(b) B
(c) C
(d) D
(e) E

17. Which of the following should be the SECOND sentence after rearrangement?

(a) A
(b) B
(c) C
(d) D
(e) F

18. Which of the following should be the THIRD sentence after rearrangement?

(a) A
(b) B
(c) C
(d) D
(e) E

19. Which of the following should be FIFTH sentence after rearrangement?

(a) A
(b) B
(c) C
(d) D
(e) E

20. Which of the following should be the LAST (SIXTH) sentence after rearrangement?

(a) A
(b) B
(c) C
(d) D
(e) E

DOWNLOAD IBPS PO Question Papers PDF

DOWNLOAD IBPS CLERK Question Papers PDF

DOWNLOAD ALL BANK EXAMS PAPERS E-BOOKS

Printed Study Material for IBPS PO Exam

IBPS / SBI Special TX: 
Subject: 
General: 

(Papers) IBPS PO Exam Paper - 2013 "Held on: 27-10-2013" ::QUANTITATIVE APTITUDE::

IBPS logo



(Papers) IBPS PO Exam Paper - 2013 "Held on: 27-10-2013"

::QUANTITATIVE APTITUDE::



DIRECTIONS (Qs. 1-5) : In each of the following questions, a question is followed by information given in three Statements I, II and III. You have to study the question along with the statements and decide the information given in which of the statement(s) is necessary to answer the question.

1. In how many days 10 women can finish the work?

I. 10 men finish the work in 6 days.
II. 10 women and 10 men finish the work in 24/7 days.
III. If 10 men work 3 days and after that 10 women are deployed to work for men, the rest work is finished in 4 days.

(a) I and II
(b) Any two of three
(c) I and III
(d) II and III
(e) None of these

2. What is the present age of Sabir?

I. The present age of Sabir is half of his father's age.
II. After five years the ratio of ages of Sabir and his father is 6 : 11.
III. Sabir is younger to his brother by five years.

(a) I and II
(b) I and III
(c) II and III
(d) All of these
(e) Cannot be determined

3. What is two digit number?

I. The difference between the number and the number formed by interchanging the digit is 27.
II. The difference between two digits is 3.
III. The digit at unit's place is less than that at ten place by 3.

(a) I and II
(b) I and either II or III
(c) I and III
(d) All of these
(e) None of these

4. What is the rate of interest Percent per annum?

I. An amount doubles itself in 5 yr on simple interest;
II. Difference between the compound interest and the simple interest earned on a certain amount in two years is ` 400.
III. Simple interest earned per annum is ` 2000.

(a) Only I
(b) II and III
(c) Any two of three
(d) I or II and III
(e) Only I or II and III

5. What is the cost of flooring the rectangular hall?

I. Length and the breadth of the hall are in the ratio of 3 : 2
II. Length of the hall is 48 m and cost of flooring is ` 850 per sq m.
III. Perimeter of the hall is 160 m and cost of flooring is ` 850 per sq m.

(a) I and II
(b) I and III
(c) Only III
(d) I and either II or III
(e) Any two of the three

6. If the numerator of a fraction is increased by 20% and the denominator is increased by 25%, the fraction obtained is 3/5. What was the original fraction?

(a) 5/7
(b) 4/7
(c) 3/8
(d) Cannot be determined
(e) None of these

7. If the positions of the digits of a two-digit number are interchanged, the number obtained is smaller than the original number by 27. If the digits of the number are in the ratio of 1 : 2, what is the original number?

(a) 36
(b) 63
(c) 48
(d) Cannot be determined
(e) None of these

8. One of the angles of a quadrilateral is thrice the smaller angle of a parallelogram. The respective ratio between the adjacent angles of the parallelogram is 4:5. Remaining three angles of the quadrilateral are in ratio 4 : 11: 9 respectively. What is the sum of the largest and the smallest angles of the quadrilateral?

(a) 255°
(b) 260°
(c) 265°
(d) 270°
(e) None of these

9. An aeroplane flies with an average speed of 756 km/h. A helicopter takes 48 h to cover twice the distance covered by aeroplane in 9 h. How much distance will the helicopter cover in 18 h? (Assuming that flights are non-stop and moving with uniform speed.)

(a) 5010 km
(b) 4875 km
(c) 5760 km
(d) 5103 km
(e) None of these

DIRECTIONS (Qs. 10-14) : Study the following table carefully and answer the question given below.

Number of People taking Fresh Loans from Different Banks over the Year and the Percentage of Defaulters Amongst them each Year

10. Approximately how many people taking a loan from Bank S in the year 2006 were defaulters?

(a) 6490
(b) 6210
(c) 5020
(d) 6550
(e) 5580

11. Approximately what was the difference between the number of defaulters of Bank Q in the year 2004 and 2005?

(a) 175
(b) 125
(c) 190
(d) 205
(e) 140

12. In which of the following years was the number of defaulters of Bank R, the maximum among the given years?

(a) 2005
(b) 2006
(c) 2007
(d) 2010
(e) None of these

13. In which of the following years was the difference in number of people taking loan from Bank P from the previous year the highest?

(a) 2008
(b) 2006
(c) 2007
(d) 2005
(e) None of these

Click Here To Download Full Paper

DOWNLOAD IBPS PO Question Papers PDF

DOWNLOAD IBPS CLERK Question Papers PDF

DOWNLOAD ALL BANK EXAMS PAPERS E-BOOKS

Printed Study Material for IBPS PO Exam

IBPS / SBI Special TX: 
General: 

(Papers) IBPS PO Exam Paper - 2013 "Held on: 27-10-2013" ::GENERAL AWARENESS::

IBPS logo


(Papers) IBPS PO Exam Paper - 2013 "Held on: 27-10-2013"

::GENERAL AWARENESS::



1. Which of the following is a receipt listed in India and traded in rupees declaring ownership of shares of a foreign company?

(a) Indian Depository Receipt (IDR)
(b) European Depository Receipt (EDR)
(c) Global Depository Receipt (GDR)
(d) American Depository Receipt (ADR)
(e) Luxemburg Depository Receipt (LDR)

2. A bank without any branch network that offers its services remotely through online banking, telephone/mobile banking and interbank ATM network alliances is known as

(a) Universal Banking
(b) Indirect Bank
(c) Door Step Bank
(d) A Direct Bank
(e) Unit Banking

3. Which of the following Indian Universities is Asia's largest residential university?

(a) Allahabad University
(b) Utkal University
(c) Banaras Hindu University
(d) Anna University
(e) Jawaharlal Nehru University

4. In October, 2013, which country has confirmed plans to create a secure mail service to protect its citizens and businesses against foreign espionage?

(a) Mexico
(b) Brazil
(c) Sweden
(d) Germany
(e) None of these

5. The campaign name 'Heal India' aims to create awareness about which of the following diseases?

(a) Mental illness
(b) AIDS
(c) Leprosy
(d) Alzheimer
(e) None of these

6. The target set by the UIDAI for issuance of Aadhaar cards upto 2014 is

(a) 50 crore cards
(b) 55 crore cards
(c) 45 crore cards
(d) 40 crore cards
(e) 60 crore cards

7. Which of the following nations has signed a com-prehensive free trade agreement with European Union?

(a) Japan
(b) China
(c) Russia
(d) Canada
(e) None of these

8. According to the provisions of the income Tax Act, 1961 a resident individual is categorised as a 'very senior citizen' when he is

(a) 80 yr of age or older
(b) 75 yr of age or older
(c) 90 yr of age or older
(d) 85 yr of age or older
(e) 65 yr of age or older

9. Who among the following has recently been conferred with the first Yash Chopra Memorial Award?

(a) Other than those given as options
(b) Madhuri Dixit Nene
(c) Ramesh Sippy
(d) Lata Mangeshkar
(e) AR Rahman

10. Which of the following is an investment advisory discipline?

(a) Corporate Industrial Finance
(b) Offshare Banking
(c) Wholesale Banking
(d) Wealth Management
(e) Trade Finance

DOWNLOAD IBPS PO Question Papers PDF

DOWNLOAD IBPS CLERK Question Papers PDF

DOWNLOAD ALL BANK EXAMS PAPERS E-BOOKS

Printed Study Material for IBPS PO Exam

IBPS / SBI Special TX: 
General: 

(Papers) IBPS PO Exam Paper - 2013 "Held on: 27-10-2013" ::COMPUTER KNOWLEDGE::

IBPS logo


(Papers) IBPS PO Exam Paper - 2013 "Held on: 27-10-2013"

::COMPUTER KNOWLEDGE::


1. Macros stored in the global macro sheet can be used

(a) in the current document only
(b) in any document
(c) can be used only with other macros of the global macro sheet
(d) not consistent behaviour
(e) None of the above

2. About pasting from the clip board

(a) a part of the clip board contents can be pasted
(b) whole of the contents of clip board can be pasted
(c) sometimes (a) and sometimes (b)
(d) (a) and (b)
(e) None ol the above

3. One of the following statements is not true for BUFFERS command

(a) increasing numbers of BUFFERS can speed program execution, but only to a certain extent
(b) the more buffers that exist the more sectors can be stored In memory; hence fewer accesses of disk are necessary
(c) The BUFFERS command is used to establish the number of disk buffers set up by MS-DOS during booting
(d) All of the above
(e) None of the above

4. EPROM can be used for

(a) erasing the contents of ROM
(b) reconstructing the contents of ROM
(c) erasing and reconstructing the contents of ROM
(d) duplicating the ROM
(e) None of the above

5. Attributes can be defined for

(a) entity
(b) switch board
(c) macro
(d) pages
(e) None of the above

6. Where will we find the referential integrity command ?

(a) Tools
(b) View
(b) Format
(d) Table
(e) None of these

7. Anything that is typed in a worksheet appears

(a) in the formula bar only
(b) in tho active cell only
(c) in both active cell and formula bar
(d) in tho formula bar first and when we press ENTER it appears in active cell
(e) None of the above

8. Which bar is usually located below the Title Bar that provides categorised options?

(a) Menu Bar
(c) Status Bar
(b) Toolbar
(d) Scroll Bar
(e) None of the above

9. A pixel is

(a) a computer program that draws picture
(b) a picture stored in the secondary memory
(c) the smallest resolvable part of a picture
(d) a virus
(e) None of the above

10. How many types of cell references are available in Excel?

(a) 3
(b) 4
(c) 8
(d) 10
(e) None of these

Click Here To Download Full Paper

DOWNLOAD IBPS PO Question Papers PDF

DOWNLOAD IBPS CLERK Question Papers PDF

DOWNLOAD ALL BANK EXAMS PAPERS E-BOOKS

Printed Study Material for IBPS PO Exam

IBPS / SBI Special TX: 
General: 

IBPS (PO) Previous Year Exam Paper - 2014 "Computer Knowledge" Held on: 19-10-2014

IBPS logo



IBPS CWE (PO/MT) Previous Year Exam Paper - 2014

Subject: Computer Knowledge



1. Which of the following places the common data elements in order from smallest to largest?

(1) Bit, byte, character, record, field, file, database
(2) Database, file, record, field, character
(3) Character, record, field, database, file
(4) Character, file, record, field, database
(5) Character, field, record, file, database

2. Specialised programs that assist users in locating information on the web are called

(1) Web browsers
(2) Information engines
(3) Data engines
(4) Search engines
(5) None of these

3. What is the term for unsolicited e-mail?

(1) Flamming
(2) Usenet
(3) Spam
(4) Backbone
(5) News Group

4. Processor speed is measured in

(1) bytes
(2) gigabytes
(3) gigahertz
(4) megabytes
(5) kilobytes

5. Which of the following can be used to select the entire document?

(1) Ctrl + H
(2) Alt + S
(3) Shift + E
(4) Ctrl+A
(5) Ctrl + K

6. The buying or selling of goods over the internet is termed as

(1) e-selling
(2) e-buying
(3) e-business
(4) e-commerce
(5) e-transaction

7. What are the programs that block access to selected websites known as?

(1) Channels
(2) Filters
(3) Browsers
(4) Telnet
(5) Drivers

8. Which of the following is not a valid version of MS-Office?

(1) Office 2000
(2) Office XP
(3) Office 2003
(4) Office 2007
(5) Office Vista

9. Which of the following is used to write Web pages?

(1) URL
(2) HTML
(3) Telnet
(4) HTTP
(5) FTP

10. A computer chip is also called a

(1) register
(2) switch
(3) server
(4) modem
(5) microprocessor

Click Here To Download Full Paper

DOWNLOAD IBPS PO Question Papers PDF

DOWNLOAD IBPS CLERK Question Papers PDF

DOWNLOAD ALL BANK EXAMS PAPERS E-BOOKS

Printed Study Material for IBPS PO Exam

IBPS / SBI Special TX: 

IBPS (PO) Previous Year Exam Paper - 2014 "General Awareness" Held on: 19-10-2014

IBPS logo



IBPS CWE (PO/MT) Previous Year Exam Paper - 2014

Subject: General Awareness



1. Which of the following is the boundary line between India and Pakistan?’

(1) 17th Parallel
(2) Radcliffeline
(3) Other than those given as options
(4) Hindenburg Line
(5) McMahon Line

2. Banks generally insist account holders to nominate persons to whom the money lying in their accounts should go in the event of

(1) death of account holder
(2) illness of account holder
(3) account holder migrates
(4) account holder turns bankrupt
(5) Other than those given as options

3. Credit Risk refers to the risk that

(1) borrower may opt to get necessary permit/licences
(2) wrong strategy is adopted
(3) a borrower will default on any type of debt
(4) loan processing may be faulty
(5) interest rate in the markets may increase

4. Which of the following is the currency of Cambodia?

(1) Cambodian Dollar
(2) Ringgit
(3) Peso
(4) Riel
(5) Other than those given as options

5. Which of the following is a source of collecting money from the public for a company for the first time?

(1) Rights issue
(2) Bonus shares
(3) Follow on offering
(4) Initial public offer
(5) Secondary offering

6. Which of the following concepts of banking involves booking of web-enabled touch point for basic banking services?

(1) Extension Conters
(2) Retail Banking
(3) Kiosk Banking
(4) Satellite Banking
(5) Mobile Banking

7. Face value of a Government security is the amount that is to be paid to an investor at the maturity date of the security. The face value is not the same as the

(1) Par value
(2) Market Price
(3) Redemption value
(4) Repayment amount
(5) Principal value

8. As per the 2011 Census report, India’s population below the age of 35 years is

(1) between 30 & 35%
(2) between 40 & 50%
(3) between 35 & 40%
(4) more than 65%
(5) between 50 & 65%

9. The facility to help insurance policy holders buy and keep policies in electronic form rather than as a paper document is called

(1) Insurance Warehousing
(2) Insurance Cache
(3) Insurance Stockpile
(4) Insurance Depository
(5) Insurance Repository

10. Based on the recovery of the dues, banks are required to classify non-performing assets (NPAs) in the books of the bank under the categories

(1) Substandard, Doubtful and Bad Debts
(2) Standard, Doubtful and Written-Off Assets
(3) Standard, Substandard and Doubtful Assets
(4) Standard, Doubtful and Loss Assets
(5) Substandard, Doubtful and Loss Assets

 

Click Here To Download Full Paper

DOWNLOAD IBPS PO Question Papers PDF

DOWNLOAD IBPS CLERK Question Papers PDF

DOWNLOAD ALL BANK EXAMS PAPERS E-BOOKS

 

11. A mutual fund scheme in which the investors commit their money for a particular period is known as

(1) Long-End Scheme
(2) Closed-End Scheme
(3) Long-Term Fund
(4) Open-End Scheme
(5) Back-End Scheme

12. In the Asian Games who among the following won the silver Medal for India in Squash?

(1) Geetika Jakhar
(2) Yogeshwar Dutt
(3) Vinesh Phogat
(4) Saurav Ghosal
(5) Other than those given as options

13. Plant Genome Saviour Community Awards are given by Protection of Plant Varieties and Farmer’s Right Authority (PPV & FRA) for

(1) getting the highest yield of crops
(2) Other than those given as options
(3) engagement in the conservation of genetic resources of economic plants and their wild relatives
(4) ensuring soil conservation and preventing use of chemicals
(5) using traditional seeds and fertilizers to protect the soil

14. CP is an unsecured money market instrument issued in the form of a promissory note. In the abbreviation CP, letter ‘P’ stands for

(1) Paper
(2) Portfolio
(3) Payment
(4) Promissory
(5) Position

15. Vishal Sikka is the CEO of

(1) Tata Group of Companies
(2) Cognizant
(3) Oracle
(4) Infosys
(5) Other than those given as options

16. The process that has to be undertaken by banks and other financial institutions to prevent them from being used by criminal elements for money laundering is

(1) Credit Monitoring Process
(2) Credit Rating Process
(3) KYC Process
(4) Due diligence Process
(5) Credit Appraisal Process

17. CVV is an anti-fraud security feature that helps verify that the customer is in posession of her card. The abbreviation of CVV stands for

(1) Card Virtual Valuation
(2) Confidential Virtual Verification
(3) Card Verification Value
(4) Core Virtual Value
(5) Coded Vulnerability Value

18. Which of the foIIowing is the capital of Argentina?

(1) Addis Ababa
(2) Other than those given as options
(3) Buenos Aires
(4) Seoul
(5) Phnom Penh

19. With introduction of ATMs, telebanking and internet banking, banking hours is not a constraint for transacting banking business, which is known as

(1) Universal Banking
(2) Worldwide Banking
(3) Anywhere Banking
(4) Global Banking
(5) Other than those given as option

20. Which of the following institutions is a Credit Information Company?

(1) ARCIL
(2) CRISIL
(3) CIBIL
(4) ICRA
(5) CARE

IBPS / SBI Special TX: 

IBPS (PO) Previous Year Exam Paper - 2014 "Quantitative Aptitude" Held on: 19-10-2014

IBPS logo



IBPS CWE (PO/MT) Previous Year Exam Paper - 2014

Subject: Quantitative Aptitude



1. A bakery bakes cake with the expectation that it will earn a profit of 40% by selling each cake at marked price. But during the delivery to showroom 16% of the cakes were completely damaged and hence could not be sold. 24% of the cakes were slightly damaged and hence could be sold at 80% of the cost price. The remaining 60% of the cakes were sold at marked price. What is the percentage profit in the whole consignment?

(1) 3.2
(2) 2.4
(3) 2.8
(4) 4.2
(5) 3.6

2. A professional institute’s total expenditure on students for a particular course is partly fixed and partly varies linearly with the number of students. The average expense per student is Rs. 615 when there are 24 students and Rs. 465 when there are 40 students. What is the average expense when there are 60 students?

(1) Rs. 370
(2) Rs. 450
(3) Rs. 350
(4) Rs. 420
(5) Rs. 390

3.

(1) 4 : 5
(2) 14 : 15
(3) 6 : 7
(4) 18 : 25
(5) 22 : 25

4. A, B and C have to type 506 pages to finish an assignment. A can type a page in 12 minutes, B in 15 minutes and C in 24 minutes. If they divide the task into three parts so that all three of them spend equal amount of time in typing what is the number of pages that B should type?

(1) 172
(2) 176
(3) 154
(4) 168
(5) 164

Directions (Q. 5-9): The question consists of a question and two statements I and II given below it. You have to decide whether the data provided in the statements are sufficient to answer the question. Read both the statements and choose the appropiate option.

5. What is the base of a triangle PQR with PQ as base?

(1) The data in statement II alone are sufficient to answer the question, while the data in statement I alone are not sufficient to answer the question.
(2) The data either in statement I alone or in statement II alone are sufficient toanswer the question.
(3) The data in statement I alone are sufficient to answer the question, while the data in statement II alone are not sufficient to answer the question.
(4) The data in both statements I and II together are necessary to answer the question
(5) The data even in both statements I and II together are not sufficient to answer the question.

6. What is the speed of the train?

I. A train crosses another train coming from opposite direction at the speed of 45 kmph in 20 seconds.
II. The train crosses another train running in the same direction at the speed of 42 kmph in 1 minute 18 seconds.

(1) The data either in statement I alone or in statement II alone are sufficient to answer the question.
(2) The data in statement I alone are sufficient to answer the question, while the data in statement II alone are not sufficient to answer the question.
(3) The data even in both statements I and II together are not sufficient to answer the quesiton.
(4) The data in both statements I and II togethere are necessary to answer the question
(5) The data in statement II alone are sufficient to answer the question, while the data in statement I alone are not sufficient to answer the question.

7. What is the rate of interest pcpa?

I. An amount invested on simple interest becomes four times in 24 years.
II. The difference between compound interest and simple interest for two years on an amount of Rs. 10000 at that rate is Rs. 156.25.

(1) The data in statement II alone are sufficient to answer the question, while the data in statement I alone are not sufficient to answer the question.
(2) The data in statement I alone are sufficient to answer the question, while the data in statement II alone are not sufficient to answer the question.
(3) The data in both statements I and II together are necessary to answer the question
(4) The data even in both statements I and II together are not sufficient to answer the question.
(5) The data either in statement I alone or in statement II alone are sufficient to answer the question.

8. What is the total strength of institute X?

I. Out of the total strength of Institute X, 35% are females and rest are males. The number of females in Institute X is equal to the number of males in Institute Y.
II. Out of the total strength of Institute Y, the no. of males is 560, which is 28% of the total strength.

(1) The data either in statement I alone or in statement II alone are sufficient to answer the question.
(2) The data in statement II alone are sufficient to answer the question, while the data in statement I alone are not sufficient to answer the question.
(3) The data even in both statements I and II together are not sufficient to answer the question.
(4) The data in statement I alone are sufficient to answer the question, while the data in statement II alone are not sufficient to answer the question.
(5) The data in both statements I and II together are necessary to answer the question.

9. What is the cost of fencing a rectangular plot along all four sides @ Rs. 450 per metre?

I. The area of the plot is 1458 m2, which is 54 times the numerical value of its breadth.
II. The length of the plot is 200% of its breadth and the breadth is 50% of its length.

(1) The data even in both statements I and II together are not sufficient to answer the question.
(2) The data in statement II alone are sufficient to answer the question, while the data in statement I alone are not sufficient to answer the question.
(3) The data in statement I alone are sufficient to answer the question, while the data in statement II alone are not sufficient to answer the question.
(4) The data either in statement I alone or in statement II alone are sufficient to answer the question.
(5) The data in both statements I and II together are necessary to answer the question.

10. In the figure given below, the perimeter of the circle is 220 cm. What is the area of the shaded portion in cm2?

Directions (Q. 11-1(5): Study the table to answer the given questions.

Click Here To Download Full Paper PDF

DOWNLOAD IBPS PO Question Papers PDF

DOWNLOAD IBPS CLERK Question Papers PDF

DOWNLOAD ALL BANK EXAMS PAPERS E-BOOKS

Printed Study Material for IBPS PO Exam


11. What is the difference between the total number of people living in City R, Q and T together who do not watch Arrow and the total number of people living in these three cities together who watch Arrow?

(1) 47200
(2) 45300
(3) 47400
(4) 47600
(5) 45600

12. What is the average number of males who watch Big Bang Theory in all the cities together?

(1) 6320
(2) 6380
(3) 6340
(4) 6350
(5) 6360

13. The ratio of the total number of males to the total number of females in City P is 5 : 3. What percent of the female population watches Breaking Bad in City P?

14. The total population (males and females) of City R watching Mentalist is what percent more than the total population (male and female) of City T watching the same TV Series?

15. What is the ratio of the number of females who watch Breaking Bad in City Q and City S together to the number of females who watch Mentalist in the same cities together?

(1) 59 : 47
(2) 55 : 48
(3) 59 : 42
(4) 55 : 43
(5) 59 : 45

16. Raghu invested a certain sum in Scheme X for 4 years. Scheme X offers simple interest @ 12 pcpa for the first two years and compound intreset (compounded annually) @ 20 pcpa for the next two years. The total intreset earned by him after 4 years is Rs. 11016. What was the sum invested by Raghu in Scheme X?

(1) Rs. 17400
(2) Rs. 18400
(3) Rs. 16200
(4) Rs. 11400
(5) Rs. 9400

Directions (Q. 17): This question is based on the graph below:



17. What is the difference between the average number of male and female teachers in the given schools?

(1) 10
(2) 20
(3) 5
(4) 25
(5) 15

Directions (Q. 18-22): In this question two equations numbered I & II are given. You have to solve both the equations and find out the correct option.

18. I. 6x2 + 41x + 63 = 0
II. 4y2 + 8y + 3 = 0

19. I. x2 + 10x + 24 = 0
II. 4y2 – 17y + 18 = 0

20. I. 24x2 + 38x + 15 = 0
II. 12y2 + 28y + 15 = 0

21. I. 3x2 – 20x – 32 = 0
II. 2y2 – 3y – 20 = 0

22. I. x2 – 20x + 91 = 0
II. y2 – 32y + 247 = 0

Directions (Q. 23): In the figure given below:

GHI is an equitateral triangle with side 14 cm. G is the midpoint of JL. What is the area of the shaded portion (in cm(2)?

23.

 

Directions (Q. 24-28): Refer to the pie-chart and answer the given questions:



24. What is the ratio of the number of novels (Romantic and Horror) sold by store E to the total number of Horror novels sold by stores C and F together?

(1) 35 : 32
(2) 45 : 32
(3) 35 : 24
(4) 35 : 26
(5) 45 : 34

25. What is the average number of Horror novels sold by stores B, C, E and F together?

(1) 2960
(2) 3060
(3) 2680
(4) 3240
(5) 3180

IBPS / SBI Special TX: 

IBPS (PO) Previous Year Exam Paper - 2014 "English" Held on: 19-10-2014

IBPS logo



IBPS CWE (PO/MT) Previous Year Exam Paper - 2014

Subject: Quantitative Aptitude



1. A bakery bakes cake with the expectation that it will earn a profit of 40% by selling each cake at marked price. But during the delivery to showroom 16% of the cakes were completely damaged and hence could not be sold. 24% of the cakes were slightly damaged and hence could be sold at 80% of the cost price. The remaining 60% of the cakes were sold at marked price. What is the percentage profit in the whole consignment?

(1) 3.2
(2) 2.4
(3) 2.8
(4) 4.2
(5) 3.6

2. A professional institute’s total expenditure on students for a particular course is partly fixed and partly varies linearly with the number of students. The average expense per student is Rs. 615 when there are 24 students and Rs. 465 when there are 40 students. What is the average expense when there are 60 students?

(1) Rs. 370
(2) Rs. 450
(3) Rs. 350
(4) Rs. 420
(5) Rs. 390

3.

(1) 4 : 5
(2) 14 : 15
(3) 6 : 7
(4) 18 : 25
(5) 22 : 25

4. A, B and C have to type 506 pages to finish an assignment. A can type a page in 12 minutes, B in 15 minutes and C in 24 minutes. If they divide the task into three parts so that all three of them spend equal amount of time in typing what is the number of pages that B should type?

(1) 172
(2) 176
(3) 154
(4) 168
(5) 164

Directions (Q. 5-9): The question consists of a question and two statements I and II given below it. You have to decide whether the data provided in the statements are sufficient to answer the question. Read both the statements and choose the appropiate option.

5. What is the base of a triangle PQR with PQ as base?

(1) The data in statement II alone are sufficient to answer the question, while the data in statement I alone are not sufficient to answer the question.
(2) The data either in statement I alone or in statement II alone are sufficient toanswer the question.
(3) The data in statement I alone are sufficient to answer the question, while the data in statement II alone are not sufficient to answer the question.
(4) The data in both statements I and II together are necessary to answer the question
(5) The data even in both statements I and II together are not sufficient to answer the question.

6. What is the speed of the train?

I. A train crosses another train coming from opposite direction at the speed of 45 kmph in 20 seconds.
II. The train crosses another train running in the same direction at the speed of 42 kmph in 1 minute 18 seconds.

(1) The data either in statement I alone or in statement II alone are sufficient to answer the question.
(2) The data in statement I alone are sufficient to answer the question, while the data in statement II alone are not sufficient to answer the question.
(3) The data even in both statements I and II together are not sufficient to answer the quesiton.
(4) The data in both statements I and II togethere are necessary to answer the question
(5) The data in statement II alone are sufficient to answer the question, while the data in statement I alone are not sufficient to answer the question.

7. What is the rate of interest pcpa?

I. An amount invested on simple interest becomes four times in 24 years.
II. The difference between compound interest and simple interest for two years on an amount of Rs. 10000 at that rate is Rs. 156.25.

(1) The data in statement II alone are sufficient to answer the question, while the data in statement I alone are not sufficient to answer the question.
(2) The data in statement I alone are sufficient to answer the question, while the data in statement II alone are not sufficient to answer the question.
(3) The data in both statements I and II together are necessary to answer the question
(4) The data even in both statements I and II together are not sufficient to answer the question.
(5) The data either in statement I alone or in statement II alone are sufficient to answer the question.

8. What is the total strength of institute X?

I. Out of the total strength of Institute X, 35% are females and rest are males. The number of females in Institute X is equal to the number of males in Institute Y.
II. Out of the total strength of Institute Y, the no. of males is 560, which is 28% of the total strength.

(1) The data either in statement I alone or in statement II alone are sufficient to answer the question.
(2) The data in statement II alone are sufficient to answer the question, while the data in statement I alone are not sufficient to answer the question.
(3) The data even in both statements I and II together are not sufficient to answer the question.
(4) The data in statement I alone are sufficient to answer the question, while the data in statement II alone are not sufficient to answer the question.
(5) The data in both statements I and II together are necessary to answer the question.

9. What is the cost of fencing a rectangular plot along all four sides @ Rs. 450 per metre?

I. The area of the plot is 1458 m2, which is 54 times the numerical value of its breadth.
II. The length of the plot is 200% of its breadth and the breadth is 50% of its length.

(1) The data even in both statements I and II together are not sufficient to answer the question.
(2) The data in statement II alone are sufficient to answer the question, while the data in statement I alone are not sufficient to answer the question.
(3) The data in statement I alone are sufficient to answer the question, while the data in statement II alone are not sufficient to answer the question.
(4) The data either in statement I alone or in statement II alone are sufficient to answer the question.
(5) The data in both statements I and II together are necessary to answer the question.

10. In the figure given below, the perimeter of the circle is 220 cm. What is the area of the shaded portion in cm2?

Click Here To Download Full Paper PDF

DOWNLOAD IBPS PO Question Papers PDF

Printed Study Material for IBPS PO Exam

DOWNLOAD IBPS CLERK Question Papers PDF

DOWNLOAD ALL BANK EXAMS PAPERS E-BOOKS

Printed Study Material for IBPS PO Exam

Directions (Q. 11-1(5): Study the table to answer the given questions.



11. What is the difference between the total number of people living in City R, Q and T together who do not watch Arrow and the total number of people living in these three cities together who watch Arrow?

(1) 47200
(2) 45300
(3) 47400
(4) 47600
(5) 45600

12. What is the average number of males who watch Big Bang Theory in all the cities together?

(1) 6320
(2) 6380
(3) 6340
(4) 6350
(5) 6360

13. The ratio of the total number of males to the total number of females in City P is 5 : 3. What percent of the female population watches Breaking Bad in City P?

14. The total population (males and females) of City R watching Mentalist is what percent more than the total population (male and female) of City T watching the same TV Series?

15. What is the ratio of the number of females who watch Breaking Bad in City Q and City S together to the number of females who watch Mentalist in the same cities together?

(1) 59 : 47
(2) 55 : 48
(3) 59 : 42
(4) 55 : 43
(5) 59 : 45

16. Raghu invested a certain sum in Scheme X for 4 years. Scheme X offers simple interest @ 12 pcpa for the first two years and compound intreset (compounded annually) @ 20 pcpa for the next two years. The total intreset earned by him after 4 years is Rs. 11016. What was the sum invested by Raghu in Scheme X?

(1) Rs. 17400
(2) Rs. 18400
(3) Rs. 16200
(4) Rs. 11400
(5) Rs. 9400

Directions (Q. 17): This question is based on the graph below:



17. What is the difference between the average number of male and female teachers in the given schools?

(1) 10
(2) 20
(3) 5
(4) 25
(5) 15

Directions (Q. 18-22): In this question two equations numbered I & II are given. You have to solve both the equations and find out the correct option.

18. I. 6x2 + 41x + 63 = 0
II. 4y2 + 8y + 3 = 0

19. I. x2 + 10x + 24 = 0
II. 4y2 – 17y + 18 = 0

20. I. 24x2 + 38x + 15 = 0
II. 12y2 + 28y + 15 = 0

21. I. 3x2 – 20x – 32 = 0
II. 2y2 – 3y – 20 = 0

22. I. x2 – 20x + 91 = 0
II. y2 – 32y + 247 = 0

Directions (Q. 23): In the figure given below:

GHI is an equitateral triangle with side 14 cm. G is the midpoint of JL. What is the area of the shaded portion (in cm(2)?

23.

 

Directions (Q. 24-28): Refer to the pie-chart and answer the given questions:



24. What is the ratio of the number of novels (Romantic and Horror) sold by store E to the total number of Horror novels sold by stores C and F together?

(1) 35 : 32
(2) 45 : 32
(3) 35 : 24
(4) 35 : 26
(5) 45 : 34

25. What is the average number of Horror novels sold by stores B, C, E and F together?

(1) 2960
(2) 3060
(3) 2680
(4) 3240
(5) 3180

 

IBPS / SBI Special TX: 
Subject: 

IBPS (PO) Previous Year Exam Paper - 2014 "Reasoning" Held on: 19-10-2014

IBPS logo



IBPS CWE (PO/MT) Previous Year Exam Paper - 2014

Subject: Reasoning Ability



Directions (Q. 1-5): Each of the questions below consists of a question and two statements numbered, I and II given below it. You have to decide whether the data given in the statements are sufficient to answer the question. Read both statements and choose the most appropriate option.

1. How many persons are standing between Land K in a straight line of 19 persons? (Note: All are standing in a straight line, facing north).

I. Y stands on the extreme left end of the line. Only five persons stand between Y and K. Only six persons stand between K and R. Only four persons stand between Rand L.
II. J stands exactly in the middle of the line. Only two persons stand between I and J. Only five persons stand between I and L. I stands to the left of L. K stands third to the left of J.

(1) The data even in both statements I and II together are not sufficient to answer the question.
(2) The data in both statements I and II together are necessary to answer the question.
(3) The data in statement II alone are sufficient to answer the question, while the data in statement I alone are not sufficient to answer the question.
(4) The data either in statement I alone or in statement II alone are sufficient to answer the question.
(5) The data in statement I alone are sufficient to answer the question while the data in statement II alone are not sufficient to answer the question.

2. Among six persons A, B, C, D, E and F standing around a circle, some ofthem are facing the centre while others are facing outside (ie opposite to the centre.). What is the position of A with respect to E?

(Note: Facing the same direction means, if one is facing the centre then the other is also facing the centre and vice versa. Facing the opposite directions means, if one is facing the centre then the other is facing outside and vice versa).

I. C stands second to the right of E. E faces outside. C is an immediate neighbour of both D and B. F stands second to the left ofD. D faces the same direction as E.
II. Only two persons stand between B and E. Both B and E face outside. E is an immediate neighbour of both D and F. B is an immediate neighbour of both C and A. A is not an immediate neighbour of D.

(1) The data in both statements I and II together are necessary to answer the question.
(2) The data in statement II alone are sufficient to answer the question while the data in statement I alone are not sufficient to answer the question.
(3) The data even in both statements I and II together are not sufficient to answer the question.
(4) The data either in statement I alone or in statement II alone are sufficient to answer the question.
(5) The data in statement I alone are sufficient to answer the question, while the data in statement II alone are not sufficient to answer the question.

3. How is X related to N?

I. X is mother of J. T is married to Z. N is daughter of T. Z is brother of J.
II. X is married to Y. Y is father of J. J is married to L. J is uncle of N.

(1) The data even in both statements I and II together are not sufficient to answer the question.
(2) The data in statement I alone are sufficient to answer the question, while the data in statement II alone are not sufficient to answer the question.
(3) The data either in statement I alone or in statement II alone are sufficient to answer the question.
(4) The data in both statements I and II together are necessary to answer the question.
(5) The data in statement II alone are sufficient to answer tlie question, while the data in statement I alone are not sufficient to answer the question.

4. Among mobiles R, S, T, U, V and W, which is the costliest?

I. T is costlier than only two mobiles. S is costlier than R but not the costliest. V is costlier than only W.
II. R is cheaper than only two mobiles.V is costlier than W but cheaper than T. T is cheaper than R. S is cheaper than U.

(1) Thedata:insatanentiI alone are sufficient to answer the question, while the data in statement I alone are not sufficient to answer the question.
(2) The data even in both statements I and II together are not sufficient to answer the question.
(3) The data either in statement I alone or in statement II alone are sufficient to answer the question.
(4) The data in statement I alone are sufficient to answer the question, while the data in statement II alone are not sufficient to answer the question.
(5) The data in both statements I and II together are necessary to answer the question.

5. In a six-storey building (consisting of floors number I to 6, wherein the topmost floor is number 6 and the ground floor is number I) each of the six friends, namely M, N, O, P, Q and R, lives on a different floor (not necessarily in the same order). Who amongst them lives on the lowermost floor?

I. M lives on floor number five. Only two persons live between M and N. Q lives immediately above P.
II. P lives on floor number three. Only two persons live between P and O. N lives immediately above R. N lives on an even-numbered floor.

(1) The data even in both statements I and II together are not sufficient to answer the question.
(2) The data in statement II alone are sufficient to answer the question while the data in statement I alone are not sufficient to answer the question.
(3) The data in statement I alone are sufficient to answer the question, while the data in statement II alone are not sufficient to answer the question.
(4) The data either in statement I alone or in statement II alone are sufficient to answer the question.
(5) The data in both statements I and II together are necessary to answer the question.

Directions (Q. 6-10): In this question are given four statements followed by five conclusions, one ofwhich definitely does not logically follow (or is not a possibility of occurrence) from the given statements. That concusion is your answer.
(Note: You have to take the four given statements to be true even it they seem to be at variance with commonly known facts and then decide which ofthe given conclusions logically does not follow from the given statements disregarding commonly known facts.)

6. Statements:

No toy is a doll.
All guns are toys.
All houses are dolls.
All dolls are baskets.

Conclusions:

(1) All baskets are toys.
(2) No gun is a house.
(3) All gun’s being baskets is a possibility.
(4) All houses are baskets.
(5) No doll is a gun.

7. Statements:

Some logics are reasons.
All reasons are arguments.
All arguments are fights.
No fight is a discussion.

Conclusions:

(1) All discussions being logic is a possibility.
(2) No discussion is an argument.
(3) All logics being discussion is a possibility.
(4) All reasons are fights.
(5) No reason is a discussion.

8. Statements:

All references are mails.
All mentions are references.
All comments are mentions.
No mail is a declaration.

Conclusions:

(1) No reference is a declaration.
(2) All comments are mails.
(3) No mention is a declaration.
(4) All declarations being comments is a possibility.
(5) At least some mails are mentions.

9. Statements:

Some moments are flashes.
All moments are seconds.
All flashes are instances.
No instance is an hour.

Conclusions:

(1) All hours being seconds is a possibility.
(2) No second isa flash.
(3) No hour is a flash.
(4) At least some moments are instances.
(5) At least some seconds are jnstances.

10. Statements:

All circles are spheres.
All spheres are rectangles.
No rectangle is a pyramid.
No pyramid is a triangle.

(1) At least some circles are pyramids.
(2) All triangles being circles is a possibility.
(3) All rectangles being triangles is a possibility.
(4) At least some rectangles are circles.
(5) No pyramid is a sphere.

11. Statements:

No hotel is a motel.
All motels are apartments.
All apartments are inns.
No inn is a guesthouse.

(1) All hotels being inns is a possibillity.
(2) No motel is a guesthouse.
(3) All hotels being apartments is a possibility.
(4) No motel is an inn.
(5) No guesthouse is an apartment.

Click Here To Download Full Paper PDF

DOWNLOAD IBPS PO Question Papers PDF

IBPS PO (Probationary Officer) Exam Study Materials

DOWNLOAD IBPS CLERK Question Papers PDF

DOWNLOAD ALL BANK EXAMS PAPERS E-BOOKS

Directions (Q. 12): Read the following statements carefully and answer the given question.
Cocoa and chocolate products have been used as medicine in many cultures for centuries. Chocolate is made from plants, which means it contains many of the health benefits of leafy vegetables.

Which of the following statements weakens the above argument?

(A) Dark chocolate contains a large number of antioxidants which slow down the ageing process.
(B) A small study revealed that regular intake of chocolate increases insulin sensivity, thus lowering the chances of diabetes.
(C) Green leafy vegetables have substances which protectskin from UV rays.
(D) Chocolates have three types of fats, one of which increases the cholesterol level.
(E) Cocoa increases blood flow to the retina, thus giving a boost to vision.

(1) Only D
(2) Only A and E
(3) Only C
(4) None of the given statements
(5) Both C and D

Directions (Q. 13-18): Study the following information and answer the questions.
Seven friends, namely L, M, N, O, P, Q and R, like different animated movies, namely Finding Nemo, Rio, Frozen, Up, Lion King, Shrek and Cars, but not necessarily in the same order. Each friend also has a presentation on topics of different subjects, namely Civics, History, English, Geography, Chemistry, Physics and Biology but not necessarily in the same order.
Q has a presentation on Civics and likes neither Frozen nor Up. The one who likes Finding Nemo has a presentation on History. L likes Rio and has a presentation neither on Geography nor on Chemistry. The one who likes Cars has a presentation on Biology. M has a presentation on Physics and does not like Up. The one who likes Up does not have a presentation on Chemistry. O likes Lion King. R does not have a presentation on History and does not like Up. P does not like Up.

13. On which of the following subjects does P have a presentation?

(1) Chemistry
(2) English
(3) Biology
(4) Other than those given as options
(5) Geography

14. Four of the following five form a group as per the given arrangement. Which of the following does not belong to that group?

(1) R - Cars
(2) Q - Shrek
(3) N - Up
(4) M - Frozen
(5) P - Rio

15. Which of the following combinations is definitely correct?

(1) N - Chemistry
(2) R - History
(3) L - English
(4) All the given combinations are definitely correct
(5) P - Geography

16. Which orthe following combinations of movie and subject is definitely correct with respect to N?

(1) Up-Chemistry
(2) Other than those given as options
(3) Shrek - Geography
(4) Up - Geography
(5) Finding Nemo - History

17. Four of the following five form a group as per the given arrangement, Which of the following does not belong to that group?

(1) Biology - Cars
(2) Chemistry - Lion King
(3) Civics - Shrek
(4) English - Frozen
(4) Geography - Up

18. Which of the following movies does Q like?

(1) Shrek
(2) Other than those given as options
(3) Finding Nemo
(4) Lion King
(5) Cars

Directions (Q. 19): The given information is followed by two statements, Read them carefully and answer the given questions.

Many parents have written a plea to the administration department of school X discontinue the rule of wearing ties to school.

(A) The school has kept different coloured ties for different academic scorers as part of their uniform. Thus the low-scoring children of school feel discriminated.
(B) Thesports uniform of the school does not have a tie; it is to be worn only on Wednesdays.

(1) Statement A weakens but Statement B strengthens the argument
(2) Both Statement A and Statement B weaken the argument
(3) Statement B weakens but Statement A strengthens theargument
(4) Both Statement A and Statement B strengthen the argument
(5) Statement A strengthens the argument and Statement B is a neutral.statement.

Directions (Q. 20-25): In the given questions, assuming the given statementsto be true. Find which of the given four conclusions mimbered I, II, III and IV is/are definitely true and give your answer accordingly.

20. Staterilent:

Conclusions:

I. S > H
II. W > H
III. R < W
IV. M > T

(1) Only I, II and III are true.
(2) Only II is true.
(3) Only I and II are true.
(4) Only I and either II or IV are true.
(5) All I, II, III and IV are true.

21. Statements:

Conclusions:

I. Y < N
II. M > N
III. N = Y
IV. M > A

(1) Only either II or III is true.
(2) Only IV and either I or III are true.
(3) Only IV is true. (4) Only II is true.
(5) Only III is true.

22. Statements:

Conclusions:

I. M < J
II. J > L
III. D > L
IV. E < M

(1) Only II is true.
(2) Only I and III are true.
(3) None is true
(4) Only II and IV are true.
(5) Only I and II are true.

23. Statements:

Conclusions:

I. Y > P
II. T < F
III. O > T
IV. P < U

(1) Only I is true.
(2) Only II is true.
(3) Only III is true .
(4) None is true.
(5) Only I and IV are true.

24.

(1) Only I and II are true.
(2) Only IV is true.
(3) None is true.
(4) Only II and IV are true.
(5) Only II and III are true.

25.

(1) None is true
(2) Only IV is true
(3) Only either I or III is true
(4) Only II and I are true
(5) Only I is true

Directions (Q. 26-32): Study the following information carefully and answer the given questions.
A, B, C, D, E, F, G and H are sitting around a square table in such a way that four of them sit at four comers of the square while four sit in the middle of each of the four sides. The ones who sit at the four comers face the centre of the table while those who sit in the middle of the sides face outside. Each ofthem likes a different subject, viz Mathematics, Hindi, English, Biology, Chemistry, Physics, History and Geography. (None of the information given is necessarily in the same order.)
· C sits third to the left of the one who likes Geography. The one who likes Geography faces outside. Only two persons sit between C and H.
· The one who likes Mathematics sits on the immediate right ofH. The one who likes Chemistry sits second to the right of G. G is neither an immediate neighbour of H nor ofC. G does not like Geography.
· Only one person sits between A and the one who likes Chemistry.
· D sits on the immediate left ofthe one who likes Physics. G does not like Physics.
· E likes History. E is not an immediate neighbour of A. The one who likes Hindi is an immediate neighbour of E.
· The one who likes Biology is an immediate neighbour of F.

IBPS / SBI Special TX: 
Subject: 
General: 

(Papers) IBPS PO Exam Paper - 2014 "Held on: 1-11-2014" ::REASONING ABILITY::

IBPS logo


(Papers) IBPS PO Exam Paper - 2014 "Held on: 1-11-2014"

::REASONING ABILITY::


DIRECTIONS (Qs. 1–5): Every question below has a three statement, followed by four conclusions numbered I, II, III and IV. You have to consider every given statement as true, even if it does not conform to the well known facts. Read the conclusions and then decide which of the conclusions can be logically derived.

1. Statements

I. Some toys are pens.
II. Some pens are papers.
III. Some papers are black.

Conclusions

I. Some toys are black.
II. No pen is black.
III. No toy is black.
IV. Some pens are black.

(a) None follows
(b) Either II or IV
(c) Either I or III and either II or IV
(d) Either I or IV
(e) All of the above

2. Statements

I. Some books are copies.
II. All copies are green.
III. Some green are yellow.

Conclusions

I. All copies are yellow.
II. Some yellow are green.
III. Some copies are yellow.
IV. All green are copies.

(a) Only II
(b) Either III or IV only
(c) Either II or IV only
(d) All of these
(e) None of these

3. Statements

I. All jugs are glasses.
II. All glasses are cups.
III. All jugs are pens.

Conclusions

I. All pens are jugs.
II. Some glasses are pens.
III. Some cups are pens.
IV. All pens are cups.

(a) All follow
(b) Only II
(c) Only II and III
(d) Only III and IV
(e) None of these

4. Statements

I. All ACs are DCs.
II. Some DCs are ECs.
III. All ECs are YYs.

Conclusions

I. Some ACs are ECs.
II. Some YYs are DCs.
III. No ACs is ECs.
IV. All DCs are ACs.

(a) I and III
(b) Only II
(c) I and II
(d) II and either I or III
(e) None of these

5. Statements

I. Some newspapers are radios.
II. Some radios are televisions.
III. No television is a magazine.

Conclusions

I. No newspaper is a magazine.
II. No radio is a magazine.
III. Some radios are not magazine.
IV. Some newspapers are televisions.

(a) Only I follows
(b) Only III follows
(c) Either I or II follows
(d) Both I and II follow
(e) None of these

DIRECTIONS (Qs. 6-10): Study the following information carefully and answer the questions based on it.

Ten students A, B, C, D, E, F, G, H, I and J are sitting in a row facing west.

(i) B and F are not sitting on either of the edges.
(ii) G is sitting to the left of D and H is sitting to the right of J.
(iii) There are four persons between E and A.
(iv) I is to the right of B and F is to the left of D.
(v) J is in between A and D and G is in between E and F.
(vi) There are two persons between H and C.

6. Who is sitting at the seventh place counting from left?

(a) H
(b) C
(c) J
(d) Either H or C
(e) None of these

7. Who among the following is definitely sitting at one of the ends?

(a) C
(b) H
(c) E
(d) Cannot be determined
(e) None of these

8. Who are immediate neighbours of I?

(a) BC
(b) BH
(c) AH
(d) Cannot be determined
(e) None of these

9. Who is sitting second left of D?

(a) G
(b) F
(c) E
(d) J
(e) None of these

10. If G and A interchange their positions, then who become the immediate neighbours of E?

(a) G and F
(b) Only F
(c) Only A
(d) J and H
(e) None of these

Click Here To Download Full Paper

DOWNLOAD IBPS PO Question Papers PDF

DOWNLOAD IBPS CLERK Question Papers PDF

DOWNLOAD ALL BANK EXAMS PAPERS E-BOOKS

Printed Study Material for IBPS PO Exam

DIRECTIONS (Qs. 11 - 15): Study the following information and answer the questions that follow:-

Twelve people Abhishek, Binit, Chand, Dhiraj, Eshita, Fatima, Garima, Hena, Ishan, Jatin, Kamal and Lalit are sitting around a rectangular table. The following information is known- The table has 12 chairs numbered from 1 to 12. 6 seats on one side of the table and 6 on the opposite side. The chairs are arranged in such a way that chair number 1 is just opposite to 12, 6 is opposite to 7 and so on- Abhishek is sitting opposite to Kamal who is the only person sitting between Chand and Jatin. Eshita is sitting opposite to
Ishan who is the only person sitting between Binit and Lalit. Fatima, sitting at chair number 1, is diagonally opposite to Chand who is sitting opposite to Dhiraj.

11. If Garima is sitting opposite to Fatima then who is sitting opposite to Hena?

(a) Lalit
(b) Binit
(c) Ishan
(d) Uniquely not determined.
(e) None of these

12. If Lalit is sitting opposite to Hena, then who is sitting opposite to Garima?

(a) Eshita or Fatima
(b) Jatin or Fatima
(c) Jatin or Eshita
(d) None of these
(e) All of the above

13. How many persons are sitting between Binit and Dhiraj, if they are on the same side of the table?

(a) 2 or 3
(b) 1 or 2
(c) 1 or 3
(d) None of these
(e) All of the above

14. Which one of the following is correct?

(a) Lalit is sitting at seat number 12
(b) Lalit is sitting at seat number 10
(c) Kamal is sitting at seat number 8
(d) Kamal is sitting at seat number 9
(e) None of these

15. Which one of the following is not correct?

(a) Lalit can be opposite to Jatin.
(b) Jatin can be opposite to Hena.
(c) Lalit is adjacent to Chand.
(d) There are three person sitting between A and F.
(e) None of these

DIRECTIONS (Qs. 16 - 20): This group of questions is based on a set of conditions. In answering some of the questions, it may be useful to draw a rough diagram. Choose the response that most accurately and completely answers each question.

A circular field, with inner radius of 10 meters and outer radius of 20 meters, was divided into five successive stages for ploughing. The ploughing of each stage was handed over to a different farmer.
(i) Farmers are referred to by the following symbols: F1, F2, F3, F4 and F5.
(ii) The points between different stages of project are referred to by the following symbols: P1, P2, P3, P4, and P5, not necessarily in the order.
(iii) Farmer F5 was given the work of ploughing stage starting at point P4.
(iv) The stage from point P5 to point P3 was not the first stage.
(v) Farmer F4 was given the work of the fourth stage.
(vi) Stage 3 finished at point P1, and the work of which was not given to farmer F1.
(vii) Farmer F3 was given work of stage ending at point P5.

16. Which was the finish point for farmer F2?

(a) P1
(b) P2
(c) P3
(d) P4
(e) P5

17. Which stage was ploughed by farmer F5?

(a) First
(b) Second
(c) Third
(d) Fourth
(e) Fifth

18. Which were the starting and finish points of stage 2?

(a) P2 and P5
(b) P5 and P3
(c) P3 and P1
(d) P5 and P4
(e) P3 and P2

19. For which farmer was P2 a finishing point?

(a) F1
(b) F2
(c) F3
(d) F4
(e) F5

20. Which was the starting point for farmer F3?

(a) P2
(b) P3
(c) P4
(d) P1
(e) None of these

DIRECTIONS (Qs. 21–25) : In the following question * ,Å , $ , £ and @ are used according to following meaning.

‘P*Q’ means, ‘P is neither equal nor smaller than Q’
‘PÅQ’ means, ‘P is not smaller than Q’
‘P$Q’ means, ‘P is neither greater nor smaller than Q’
‘P£Q’ means, ‘P is neither greater nor equal to Q’
‘P@Q’ means, ‘P is not greater than Q’
Now according to the folllowing statement if they are true, judge their Conclusions I, II and III follow definiately true.

21. Statements

E @ F , O Å F , P @ E , P $ R

Conclusions

I. O $ P
II. E Å R
III. P £ O

(a) Only I is true
(b) Only II is true
(c) Either I or II is true
(d) Only III is true
(e) None of these

22. Statements

A * B , B @ C , A $ D , D £ E

Conclusions

I. E * B
II. C $ A
III. D @ E

(a) Only I is true
(b) I and II are true
(c) Only III is true
(d) No one is true
(e) None of these

23. Statements

I Å H , H $ T , S £ T , S @ R

Conclusions

I. I * T
II. I $ T
III. S * H

(a) All are true
(b) Either I or II is true
(c) Only I is true
(d) Only II is true
(e) None of these

24. Statements

S @ T , Q $ N , T £ N , Q * O

Conclusions

I. S $ N
II. N Å O
III. N * O

(a) None is true
(b) Either I or III is true
(c) Only I is true
(d) Only II is true
(e) None of these

25. Statements

H Å J , J * K , L $ K , K @ M

Conclusions

I. K £ M
II. L $ J
III. H Å L

(a) I and III are true
(b) Only II is true
(c) Only III is true
(d) None is true
(e) None of these

Printed Study Material for IBPS PO Exam

IBPS / SBI Special TX: 
General: 

(Papers) IBPS PO Exam Paper - 2014 "Held on: 1-11-2014" ::QUANTITATIVE APTITUDE::

IBPS logo


(Papers) IBPS PO Exam Paper - 2014 "Held on: 1-11-2014"

::QUANTITATIVE APTITUDE::


1. Three containers A, B and C are having mixtures of milk and water in the ratio 1 : 5, 3 : 5 and 5 : 7, respectively. If the capacities of the containers are in the ratio 5 : 4 : 5, then find the ratio of the milk to the water if the mixtures of all the three containers are mixed together.

(a) 51 : 115
(b) 52 : 115
(c) 53 : 115
(d) 54 : 115
(e) None of these

2. Groundnut oil is now being sold at ` 27 per kg. During last month its cost was ` 24 per kg. Find by how much % a family should reduce its consumption, so as to keep the expenditure same.

(a) 100/9 %
(b) 122/11 %
(c) 119/10 %
(d) 91/10 %
(e) None of these

3. An ice-cream company makes a popular brand of ice-cream in rectangular shaped bar 6 cm long, 5 cm wide and 2 cm thick. To cut the cost, the company has decided to reduce the volume of the bar by 20%, the thickness remaining the same, but the length and width will be decreased by the same percentage amount. The new length L will satisfy :

(a) 5.5 < L < 6
(b) 5 < L < 5.5
(c) 4.5 < L < 5
(d) 4 < L < 4.5
(e) None of these

4. A sum of  ` 725 is lent in the beginning of a year at a certain rate of interest. After 8 months, a sum of  ` 362.50 more is lent but at the rate twice the former. At the end of the year, ` 33.50 is earned as interest from both the loans. What was the original rate of interest?

(a) 3.6%
(b) 4.5%
(c) 5%
(d) 3.46%
(e) None of these

5. The difference between compound interest and simple interest on a sum for 2 years at 10% per annum, when the interest is compounded annually is ` 16. If the interest were compounded half-yearly, the difference in two interests would be:

(a) ` 24.81
(b) ` 26.90
(c) ` 31.61
(d) ` 32.40
(e) None of these

6. A person lent out a certain sum on simple interest and the same sum on compound interest at certain rate of interest per annum. He noticed that the ratio between the difference of compound interest and simple interest of 3 years and that of 2 years is 25 : 8. The rate of interest per annum is:

(a) 10%
(b) 11%
(c) 12%
(d) 25/2 %
(e) None of these

7. A contract is to be completed in 46 days and 117 men were set to work, each working 8 hours a day. After 33 days, 4/7 of the work is completed. How many additional men may be employed so that the work may be completed in time, each man now working 9 hours a day ?

(a) 80
(b) 81
(c) 82
(d) 83
(e) None of these

8. Two pipes A and B can fill a cistern in 30 minutes and 40 minutes respectively. Both the pipes are opened. Find when the second pipe B must be turned off so the cistern may just be full in 10 minutes.

(a) 80/3 min
(b) 25 min
(c) 122/ 3 min
(d) 128/3 min
(e) None of these

9. A train leaves station X at 5 a.m. and reaches station Y at 9 a.m. Another train leaves station Y at 7 a.m. and reaches station X at 10: 30 a.m. At what time do the two trains cross each other ?

(a) 7 : 36 am
(b) 7 : 56 am
(c) 8 : 36 am
(d) 8 : 56 am
(e) None of these

10. A boat running upstream takes 8 hours 48 minutes to cover a certain distance, while it takes 4 hours to cover the same distance running downstream. What is the ratio between the speed of the boat and speed of water current respectively?

(a) 2 : 1
(b) 3 : 2
(c) 8 : 3
(d) Cannot be determined
(e) None of these

Click Here To Download Full Paper

DOWNLOAD IBPS PO Question Papers PDF

DOWNLOAD IBPS CLERK Question Papers PDF

DOWNLOAD ALL BANK EXAMS PAPERS E-BOOKS

Printed Study Material for IBPS PO Exam

IBPS PO (Probationary Officer) Exam Study Materials

DIRECTIONS (Qs. 11-15) : Study the pie charts given below and answer the following questions.

11. What is the cost of Gear Box?

(a) ` 9000
(b) ` 6000
(c) ` 3000
(d) `15,000
(e) None of these

12. What percentage of total cost is contributed by the brake?

(a) 5.5%
(b) 6.6%
(c) 6%
(d) 5.4%
(e) None of these

13. If the price of tyres goes up by 25%, by what amount should be the sale price be increased to maintain the amount of profit?

(a) ` 750
(b) ` 2250
(c) ` 3750
(d) ` 375
(e) None of these

14. If transmission cost increases by 20%, by what amount is the profit reduced (total price of car remains same)?

(a) ` 3000
(b) ` 4000
(c) ` 6000
(d) Cannot be determined
(e) None of these

15. What % of sale price is contributed by clutch?

(a) 6%
(b) 2%
(c) 3%
(d) Cannot be determined
(e) None of these

DIRECTIONS (Qs. 16 - 20) : Read the following information and answer the questions that follow.

In a huge Jewellery shop, the electric gadgets being used are 17 tubelights of 40 W each, 14 fans of 80 W each, 16 bulbs of 60 W each, 11 bulbs of 100 W each, 11 AC’s of 2100 W each, 9 laptops of 200 W each and 10 TV’s of 120 W each. In a day, tube lights and TV’s are used for 8 h but 60 W bulbs are used for 7 h and 100 W bulbs are used for 9 h whereas laptops and AC's are used for 5 h and 9 h respectively. However, fans are used for 11 h. (Note: 1000 W = 1 unit, 1 month = 30 days).

16. What is the total electric energy consumed (in units) by60 W bulbs in the whole month?

(a) 432
(b) 576
(c) 67.2
(d) 201.6
(e) None of these

17. Electricity consumed by all fans is what per cent of energy consumed by all the laptops?

(a) 132.2%
(b) 136.88%
(c) 122.68
(d) 169.62%
(e) None of these

18. Out of the following, which type of gadgets consume more electricity in the shop?

(a) Fans
(b) Tubelights
(c) Laptops
(d) TV’s
(e) None of these

19. If one electric unit costs ` 2.70 and power (used by AC's) unit costs ` 3.70, then what money is paid to the electricit department for one month?

(a) ` 27368
(b) ` 28683
(c) ` 78600
(d) ` 2900
(e) None of these

20. What is the ratio of consumption of electricity in units by 60W and 100 W bulbs in a month?

(a) 5 : 6
(b) 4 : 5
(c) 3 : 4
(d) 2 : 3
(e) None of these

DIRECTIONS (Qs.21–25): In each of the following questions two equations are given. You have to solve them and give answer accordingly.

(a) If x > y
(b) If x < y
(c) If x = y
(d) If x > y
(e) If x < y

DOWNLOAD IBPS PO Question Papers PDF

DOWNLOAD IBPS CLERK Question Papers PDF

DOWNLOAD ALL BANK EXAMS PAPERS E-BOOKS

Printed Study Material for IBPS PO Exam

IBPS PO (Probationary Officer) Exam Study Materials

IBPS / SBI Special TX: 
General: 

(Papers) IBPS PO Exam Paper - 201 "Held on: 1-11-2014" ::ENGLISH LANGUAGE::

IBPS logo


(Papers) IBPS PO Exam Paper - 201 "Held on: 1-11-2014"

::ENGLISH LANGUAGE::


DIRECTION (Qs. 1-15): Read the following passage carefully and answer the questions given below it. Certain words/phrases have been printed in bold to help you locate them while answering some of the questions.

Governments have traditionally equated economic progress with steel mills and cement factories. While urban centers thrive and city dwellers get rich, hundreds of millions of farmers remain mired in poverty. However, fears of food shortages, a rethinking of antipoverty priorities and the crushing recession in 2008 arecausing a dramatic shift in world economic policy in favour of greater support for agriculture. The last time when the world’s farmers felt such love was in the 1970s. At that time, as food prices spiked, there was real concern that the world was facing a crisis in which the planet was simply unable to produce enough grain and meat for an expanding population. Governments across the developing world and international aid organisations plowed investment into agriculture in the early 1970s, while technological breakthroughs, like high-yield strains of important food crops, boosted production. The result was the Green Revolution and food production exploded. But the Green Revolution became a victim of its own success.
Food prices plunged by some 60% by the late 1980s from their peak in the mid- 1970s. Policymakers and aid workers turned their attention to the poor’s other pressing needs, such as health care and education. Farming got starved of resources and investment. By 2004, aid directed at agriculture sank to 3.5% and “Agriculture lost its glitter.” Also, as consumers in highgrowth giants such as China and India became wealthier, they began eating more meat, so grain once used for human consumption got diverted to beef up livestock. By early 2008, panicked buying by importing countries and restrictions slapped on grain exports by some big producers helped drive prices upto heights not seen for three decades. Making matters worse, land and resources got reallocated to produce cash crops such as biofuels and the result was that voluminous reserves of grain evaporated. Protestsbroke out across the emerging world and fierce food riots toppled governments. This spurred global leaders into action. This made them aware that food security is one of the fundamental issues in the world that has to be dealt with in order to maintain administrative and political stability. This also spurred the U.S. which traditionally provisioned food aid from American grain surpluses to help needy nations, to move towards investing in farm sectors around the globe to boost productivity. This move helped countries become more productive for themselves and be in a better position to feed their own people. Africa, which missed out on the first Green Revolution due to poor policy and limited resources, also witnessed a ‘change’. Swayed by the success of East Asia, the primary povertyfighting method favoured by many policymakers in Africa was to get farmers off their farms and into modern jobs in factories and urban centers. But that strategy proved to be highly insufficient. Income levels in the countryside badly trailed those in cities while the FAO estimated that the number of poor going hungry in 2009 reached an all time high at more than one billion. In India on the other hand, with only 40% of its farmland irrigated, entire economic boom currently underway is held hostage by the unpredictable monsoon. With much of India’s farming areas suffering from drought this year, the government will have a tough time meeting its economic growth targets. In a report, Goldman Sachs predicted that if this year too receives weak rains, it could cause agriculture to contract by 2% this fiscal year, making the government’s 7% GDP-growth target look “a bit rich”. Another green revolution is the need of the hour and to make it a reality, the global community still has much backbreaking farm work to do.

1. What is the author’s main objective in writing the passage

(a) Criticising developed countries for not bolstering economic growth in poor nations
(b) Analysing the disadvantages of the Green Revolution
(c) Persuading experts that a strong economy depends on industrialization and not agriculture
(d) Making a case for the international society to engineer a second Green Revolution
(e) Rationalising the faulty agriculture policies of emerging countries

2. Which of the following is an adverse impact of the Green Revolution ?

(a) Unchecked crop yields resulted in large tracts of land becoming barren
(b) Withdrawal of fiscal impetus from agriculture to other sectors
(c) Farmers began soliciting government subsidies for their produce
(d) Farmers rioted as food prices fell so low that they could not make ends meet
(e) None of these

3. What is the author trying to convey through the phrase “making the government’s 7% GDP growth target look “a bit rich”?

(a) India is unlikely to achieve the targeted growth rate
(b) Allocation of funds to agriculture has raised India’s chances of having a high GDP
(c) Agricultural growth has artificially inflated India’s GDP and such growth is not real
(d) India is likely to rave one of the highest GDP growth rates
(e) A large portion of India’s GDP is contributed by agriculture

4. Which of the following factors was/were responsible for the neglect of the farming sector after the green revolution?

(A) Steel and cement sectors generated more revenue for the government as compared to agriculture.
(B) Large scale protests against favouring agriculture at the cost of other important sectors such as education and healthcare.
(C) Attention of policy makers and aid organizations was diverted from agriculture to other sectors.

(a) None
(b) Only (C)
(c) Only (B) & (C)
(d) Only (A) & (B)
(e) All (A), (B) & (C)

5. What prompted leaders throughout the world to take action to boost the agriculture sector in 2008?

(a) Coercive tactics by the U.S. which restricted food aid to poor nations
(b) The realization of the link between food security and political stability
(c) Awareness that performance in agriculture is necessary in order to achieve the targeted GDP
(d) Reports that high-growth countries like China and India were boosting their agriculture sectors to capture the international markets
(e) Their desire to influence developing nations to slow down their industrial development.

6. What motivated the U.S. to focus on investing in agriculture across the globe ?

(a) To make developing countries become more reliant on U.S. aid
(b) To ensure grain surpluses so that the U.S. had no need to import food
(c) To make those countries more self sufficient to whom it previously provided food
(d) To establish itself in the market before the highgrowth giants such as India and China could establish themselves
(e) None of these

7. What impact did the economic recession of 2008 have on agriculture ?

(a) Governments equated economic stability with industrial development and shifted away from agriculture
(b) Lack of implementation of several innovative agriculture programmes owing to shortage of funds
(c) It prompted increased investment and interest in agriculture
(d) The GDP as targeted by India was never achieved because of losses in agriculture
(e) None of these

8. What encouraged African policymakers to focus on urban jobs ?

(a) Misapprehension that it would alleviate poverty as it did in other countries
(b) Rural development outstripped urban development in many parts of Africa
(c) Breaking out of protests in the country and the fear that the government would topple
(d) Blind imitation of western models of development
(e) None of these

9. Which of the following had contributed to exorbitant food prices in 2008 ?

(A) Hoarding of food stocks by local wholesalers which inadvertently created a food shortage.
(B) Export of foodgrains was reduced by large producers.
(C) Diverting resources from cultivation of foodgrains to that of more profitable crops.

(a) None
(b) Only (C)
(c) Only (B)
(d) All (A), (B) & (C)
(e) Only (B) & (C)

10. Which of the following is true about the state of agriculture in India at present ?

(A) Of all the sectors, agriculture needs the highest allocation of funds.
(B) Contribution of agriculture to India’s GDP this year would depend greatly upon the monsoon rains.
(C) As India is one of the high-growth countries, it has surplus food reserves to export to other nations.

(a) Only (A) and (C)
(b) Only (C)
(c) Only (B)
(d) Only (B) and (C)
(e) None of these

DIRECTION (Qs. 11- 13 ): Choose the word/group of words which is most similar it meaning to the word printed in bold as used in the passage.

11. STARVED

(a) Deprived
(b) Disadvantaged
(c) Hungry
(d) Fasting
(e) Emaciated

12. SLAPPED

(a) Beaten
(b) Imposed
(c) Withdrawn
(d) Avoided
(e) Persuaded

13. PLOWED

(a) Cultivated
(b) Bulldozed
(c) Recovered
(d) Instilled
(e) Withdrew

DIRECTION (Qs. 14 and 15): Choose the word/phrase which is most opposite in meaning to the word printed in bold as used in the passage.

14. PRESSING

(a) Unpopular
(b) Undemanding
(c) Unobtrusive
(d) Unsuitable
(e) Unimportant

15. EVAPORATED

(a) Absorbed
(b) Accelerated
(c) Grew
(d) Plunged
(e) Mismanaged

DIRECTIONS (Qs. 16-20) : The sentences given in each question, when properly sequenced, form a coherent paragraph. Each sentence is labelled with a letter. Choose
the most logical order of the sentences from amongst the given choices so as to form a coherent paragraph.

16. P : In the past, the customised tailoring units were localised to the township or city and catered exclusively to domestic demand.

Q : Traditionally, Indians preferred custom-made clothing and the concept of ready-to-wear is a relatively recent one.
R : Consumer awareness of styling issues and the convenience afforded by ready-to-wear helped RMG industry make small inroads into the domestic market in the 1980s.
S : The customised tailoring outfits have always been a major source of clothing for domestic market.

(a) Q R S P
(b) Q S P R
(c) R S Q P
(d) S Q P R
(e) None of these

17. P : Such a system will help to identify and groom executives for positions of strategists.

Q : Evaluation of performance is more often than not done for the purpose of reward or punishment for past performance.
R : They must become an integral part of the executive system’ .
S : Even where the evaluation system is for one’s promotion to assume higher responsibilities, it rarely includes terms that are a key for playing the role of strategist effectively, e.g., the skills of playing the role of change agent and creative problem solving.

(a) S Q P R
(b) S R Q P
(c) R S Q P
(d) Q S R P
(e) None of these

18. P : Participation involves more than the formal sharing of decisions.

Q : Through anticipation individuals or organisations consider trends and make plans, shielding institutions from trauma of learning by shock.
R : Innovative learning involves both anticipation and participation.
S : It is an attitude characterised by the cooperation, dialogue and empathy.

(a) Q R S P
(b) P Q R S
(c) R Q P S
(d) S P Q R
(e) None of these

19. P : Almost a century ago, when the father of the modem automobile industry, Henry Ford, sold the first Model T car, he decided that only the best would do for his customers.

Q : Today, it is committed to delivering the’ finest quality with over six million vehicles a year in over 200 countries across the world.
R : And for over 90 years, this philosophy has endured in the Ford Motor company.
S : Thus a vehicle is ready for the customers only, if it passes the Ford ‘Zero Defect Programme’.

(a) P Q R S
(b) P R Q S
(c) R S P Q
(d) P R S Q
(e) None of these

20. P: Finish specialists recommended a chewing gum containing xylitol-a natural sweetener present in birch, maple, corn and straw-to be used several times a day by young children.

Q : Chewing gum is a new solution that “may work for parents whose children suffer from chronic ear infections.
R : An experiment was conducted involving three hundred and six children between two and six years.
S : After Finish studies showed that xylitol is effective in preventing cavities, a team of researchers decided to investigate its effects on a very similar type of bacteria which causes ear infections.

(a) Q R S P
(b) P Q R S
(c) R Q P S
(d) Q P S R
(e) None of these

Click Here To Download Full Paper

DOWNLOAD IBPS PO Question Papers PDF

DOWNLOAD IBPS CLERK Question Papers PDF

DOWNLOAD ALL BANK EXAMS PAPERS E-BOOKS

Printed Study Material for IBPS PO Exam

IBPS PO (Probationary Officer) Exam Study Materials

IBPS / SBI Special TX: 
Subject: 
General: 

(Papers) IBPS PO Exam Paper - 2014 "Held on: 1-11-2014" ::COMPUTER KNOWLEDGE::

IBPS logo


(Papers) IBPS PO Exam Paper - 2014 "Held on: 1-11-2014"

::COMPUTER KNOWLEDGE::


1. In computer, what are the .xls, .doc. are

(a) File names
(b) Extensions
(c) Viruses
(d) Binary files
(e) None of these

2. What is the shortcut key to insert new document?

(a) ctrl+a
(b) ctrl+n
(c) ctrl+s
(d) ctrl+d
(e) None of these

3. What is the full form of LAN?

(a) Local Area Network
(b) Land Area Network
(c) Long Area Network
(d) Line Area Network
(e) None of these

4. Hard copy is a term used to describe...?

(a) Printed output
(b) Writing on a hard board
(b) Storing information on the hard disk
(c) All of the above
(d) None of these

5. The operating system is also called the ______ between the user and the computer.

(a) interrelate
(b) interface
(c) interference
(d) intermediate
(e) None of these

6. If a computer is constantly rebooting itself, what is most commonly the problem?

(a) Insufficient Power-Supply Unit
(b) Bad Sectors on Hard Drive
(c) Processor Overheating
(d) Defective/Bad Memory
(d) None of these

7. Networks are monitored by security personnel and supervised by __________ who set(s) up accounts and passwords for authorized network users.

(a) IT managers
(b) the government
(c) network administrators
(d) password administrators
(e) None of these

8. What is a benefit of networking your computer with other computers?

(a) Increase in the computer’s speed
(b) Sharing of cables to cut down on expenses and clutter
(c) You have another computer if yours has a breakdown
(d) Increase in the speed of the network
(e) Sharing of resources to cut down on the amount of equipment needed

9. In a synchronous modem, the digital-to-analog converter transmits signal to the___________.

(a) equalizer
(b) modulator
(c) demodulator
(d) terminal
(e) None of these

10. The term “host” with respect to the internet, means __________.

(a) A computer that is a stand along computer
(b) A computer that is connected to the Internet
(c) A computer reserved for use by the host
(d) A large collection of computers
(e) Hyperlink

Click Here To Download Full Paper

DOWNLOAD IBPS PO Question Papers PDF

DOWNLOAD IBPS CLERK Question Papers PDF

DOWNLOAD ALL BANK EXAMS PAPERS E-BOOKS

Printed Study Material for IBPS PO Exam

IBPS PO (Probationary Officer) Exam Study Materials

IBPS / SBI Special TX: 
General: 

(Papers) IBPS PO Exam Paper - 2014 "Held on: 1-11-2014" ::GENERAL AWARENESS::

IBPS logo


(Papers) IBPS PO Exam Paper - 2014 "Held on: 1-11-2014"

::GENERAL AWARENESS::


1. To combat the menace of money laundering, which of the following financial institutions has introduced the 'Know Your Customer' Scheme?

(a) IDBI
(b) RBI
(c) NABARD
(d) SIDBI
(e) None of these

2. Which of the following sectors contributes maximum in deciding the growth in income of the states in India?

(a) Energy
(b) Tourism
(c) Service
(d) Transport
(e) Agriculture

3. Which of the following is the state where the number of people living below poverty line is maximum?

(a) Bihar
(b) Andhra Pradesh
(c) Uttar Pradesh
(d) Rajasthan
(e) Odisha

4. Who is the person closely associated with operation flood programmes and was honoured by Padma Vibhushan recently?

(a) Dr. V. Kurien
(b) Dr. M. S. Swaminathan
(c) Dr. Amartya Sen
(d) Dr. A. P. J. Abdul Kalam
(e) None of these

5. What is the full form of MTSS?

(a) Money Transfer Service Scheme
(b) Money Transparency Service Scheme
(c) Market Transfer Service Scheme
(d) Market Tax Service Scheme
(e) None of these

6. What is the full form of EFT?

(a) Electric funds transfer
(b) Electronic finance transaction
(c) Electronics Fund Transfer
(d) Emergency fund transfer
(e) None of these

7. RuPay is an Indian domestic card scheme conceived and launched by

(a) National Payments Corporation of India (NPCI)
(b) Industrial Finance Corporation of India (IFCI)
(c) National Minorities Development & Finance Corporation (NDMC)
(d) National Handicapped Finance and Development Corporation (NHFDC)
(e) None of these

8. Which of the following is refered as Fastest mode of transaction?

(a) Transfer funds into different bank's accounts using NEFT(National Electronic Funds Transfer).
(b) Transfer funds into other bank accounts using RTGS (Real Time Gross Settlement).
(c) Transfer funds into various accounts using IMPS (Immediate Payment Service).
(d) Transfer funds into different account of the same bank
(e) None of these

9. What is the full for of CBS?

(a) Customer Bond Solution
(b) Core banking System
(c) Core Banking Solution
(d) Customer Bond system
(e) None of these

10. What do you mean by Customer relationship management (CRM)?

I. It is a system for managing a company's interactions with current and future customers.
II. It is a system for managing a company's interactions with current and past customers.
III. It often involves using technology to organize, automate, and synchronize sales, marketing, customer service, and technical support.

(a) Only I
(b) Only II
(c) Both II and III
(d) Both I and III
(e) None of these

11. How RBI measured to liquidate the market?

(a) By Reverse Repo rate
(b) By Repo rate
(c) By Cash Reserve Ratio
(d) By Statutory liquidity ratio
(e) None of these

12. The Bank should comply and intimate the compliance of Award to Ombudsman?

(a) Within Two Months
(b) Within 3 months
(c) Within one month
(d) Within one year
(e) None of these

13. Cheque truncation can be done by?

(a) Using MICR data
(b) Sending cheque by speed post
(c) Using image processing
(d) Both (a) & (b)
(e) None of the above

14. Which of the following is not insured by the DICGC (Deposit Insurance and Credit Gurantee Corporation)?

(a) All Indian commercial Banks
(b) Foreign Banks branches functioning in India
(c) Local Area Banks
(d) Cooperative Banks
(e) Primary cooperative societies

15. Who is the current Secretary General of UNO?

(a) Ban Ki-moon
(b) Kofi Annan
(c) U Thant
(d) Trygve Halvdan Lie
(e) None of these

16. What is the new name given to the Cadbury's India?

(a) Ferrero Rocher Ltd
(b) Dante Confections
(c) Tootsie Roll Industries
(d) Mondelez India Foods Ltd
(e) None of these

17. Which day would be celebrated as Safe Motherhood Day as announced by Health Ministry of India?

(a) 7th March
(b) 8th March
(c) 9th March
(d) 10 March
(e) None of these

18. Which Hollywood personality got awarded at IIFA 2014?

(a) John Travolta
(b) Arnold Sch
(c) Vin Diesel
(d) Kevin Spacey
(e) None of these

19. Which author had received Padmabhushan award 2014?

(a) Shobha Dey
(b) Ruskin Bond
(c) Ravinder Singh
(d) Chetan Bhagat
(e) None of these

20. What is the full form of IBSA ?

(a) Indonesia, Brazil, South Africa
(b) Italy, Brazil, Sudan
(c) India, Brazil, South Africa
(d) India, Belgium, Saudi Arabia
(e) None of these

Click Here To Download Full Paper

DOWNLOAD IBPS PO Question Papers PDF

DOWNLOAD IBPS CLERK Question Papers PDF

DOWNLOAD ALL BANK EXAMS PAPERS E-BOOKS

Printed Study Material for IBPS PO Exam

IBPS PO (Probationary Officer) Exam Study Materials

IBPS / SBI Special TX: 
General: 

(Papers) IBPS PO Exam Paper - 2015 "Quantitative Aptitude Held on: 11-06-2015"

IBPS logo



(Papers) IBPS PO Exam Paper - 2015 "Held on: 11-06-2015"

:: Quantitative Aptitude ::



1. At present, Pia is 6 years older to Roy. The respective ratio between the present age of Pia and Minis is 3 : 4. At present, Ray is 14 years younger to Mini. What is Ray's present age?

(1) 16 yr
(2) 20 yr
(3) 14 yr
(4) 18 yr
(5) 24 yr

Directions (2-6): In the following questions, two equations numbered I and II are given. You have to solve both the equations and mark the appropriate answer.

Give answr

(c) if x > y                  (d) if x < y
(e) if relationship between x and y cannot be established

2. I. 4x2 + 17x + 15 = 0
II. 3y2 + 19y + 28 = 0

3. I. 3x2 – 17x + 22 = 0
II. 5y2 – 21y + 22 = 0

4. I. 3x2 + 11x + 10 = 0
II. 2y2 + 13y + 21 = 0

5. I. 3x2 + 13x + 14 = 0
II. 8y2 + 26y + 21 = 0

6. I. 3x2 – 14x + 15 = 0
II. 15y2 – 34y + 15 = 0

Directions (7-11): Study the table to answer the given questions.

Note: Other than the mentioned expenses under the given heads, there may also be some other expenses. For calculation purposes, please consider the given expenses theads only.

7. What is the respective ratio between the Marketing expenses of company A and the Marketing expenses of company C?

(1) 34 : 35
(2) 32 : 33
(3) 35 : 36
(4) 36 : 37
(5) 37 : 38

8. Total expenses of company E in October, 2012 was 20% more than the total given expenses of the same company in April, 2012. How much was the Marketing expenses of the same company in October, 2012, if it was 25% of the total expenses of the same month? (in Rs. Lakh)

(1) 27
(2) 24
(3) 26
(4) 21
(5) 18

9. Marketing expenses of company D is less than marketing expenses of company B, by approximately what percent?

(1) 20%
(2) 40%
(3) 39%
(4) 59%
(5) 29%

10. What is the average total expenses of all the companies? (in Rs. Lakh)

(1) 75.25
(2) 75.8
(3) 75
(4) 72.8
(5) 72.25

11. What is the difference between the total expenses made by company C in Legal and Marketing together and the total expenses mady company E in Legal and Marketing together? (in Rs. Lakh)

(1) 24.5
(2) 23.5
(3) 22.9
(4) 23
(5) 23.9

Directions (12-16): What approximae value will come in place of the questions mark (?) in the given questions? (You are not expected to calculate the exact value.)

12. 6399 × + 353 ÷ ? = 12025

(1) 14
(2) 22
(3) 2
(4) 16
(5) 8

13.

(1) 670
(2) 570
(3) 710
(4) 510
(5) 610

14. 8461 ÷ 11.99 - 24.1 ÷ 5/100= ?

(1) 625
(2) 400
(3) 25
(4) 900
(5) 225

Click Here To Download Full Paper

DOWNLOAD IBPS PO Question Papers PDF

DOWNLOAD IBPS CLERK Question Papers PDF

DOWNLOAD ALL BANK EXAMS PAPERS E-BOOKS

Printed Study Material for IBPS PO Exam

IBPS PO (Probationary Officer) Exam Study Materials

IBPS / SBI Special TX: 
General: 

(Papers) IBPS PO Exam Paper - 2015 "English Held on: 11-06-2015"

IBPS logo



(Papers) IBPS PO Exam Paper - 2015 "Held on: 11-06-2015"

:: English Language ::



Directions (1-5): Rearrange the following six sentence A, B, C, D, E and F is a proper sequence to form a meaningful paragraph, then answer the given questions.

A. Take for example the market for learning dancing.
B. This could never happen if there was a central board of dancing education which enforced strict standards of what will be taught and how such thing are to be taught.
C. The Indian education system is built on the presumption that if something is good for one child, it is good for all children.
D. More importantly, different teachers and institutes have developed different ways of teaching dancing.
E. There are very different dance forms that attract students with different tastes.
F. If however, we can effectively decentralize education, and if the government did not obsessively control what would be the ‘syllabus’ and what will be the method of instruction, there could be an explosion of new and innovative courses geared towards serving various riches of learners.

1. Which of the following should be the LASST (SIXTH) sentence after the arrangement?

(1) A
(2) F
(3) D
(4) C
(5) B

2. Which of the following should be the THIRD sentence after the arrangement?

(1) A
(2) B
(3) F
(4) D
(5) E

3. Which of the following should be the SECOND sentence after the arrangement?

(1) A
(2) B
(3) C
(4) D
(5) F

4. Which of the following should be the FIRST sentence after the arrangement?

(1) A
(2) B
(3) C
(4) E
(5) E

5. Which of the following should be the FOURTH sentence after the arrangement?

(1) A
(2) B
(3) C
(4) E
(5) D

Directions (6-15): Read the following passage carefully and answer the given questions. Certain words/phrases have been given in bold to help you locate them while answering some of the questions.

Manufactures of Consumer Packaged goods (CPGs) face two key challenges this year. The first continued slow or negative growth in people’s disposable incomes. The second is changing consumer attitudes toward products and brands, as the great fragmentation of consumer markets take another turn. In response, companies must dramatically shift the route they take to reach consumers in terms of both product distribution and communications. In many markets, consumer wages have been static for five years now. Even where economies are starting to perform better, the squeeze on after-tax wages, especially for the middle class younger people and families, is depressing consumer spending. Although growth in developing countries is still better than in the United States and Europe, a slowdown in emerging countries such as China-where many countries had hoped for higher sales-has translated quickly into lower-than expected consumer spending growth.

Meanwhile, what we call the great fragmentation is manifested in consumer behavior and market response. In both developed and emerging markets, there is a wider variety among consumers now than at any time in the recent past. Growth is evident both at the top of the market (where more consumers are spending for higher-quality food and other packaged goods) and at the lower end (where an increasing number of consumers are concentrating on value). But the traditional middle of the market is shrinking.
Further, individual consumer behavior is more pluralistic. We are used to seeing, for example, spirits buyers purchasing a premium band in bar, a less costly label at home for personal consumption and yet another when entertaining guest. But, this type of variegated shopping has now spread to the grocery basket as well. Fewer consumer are making one big stocking up trip each week instead shoppers are visiting a premium store and a discounter as well as a supermarket, in multiple weekly stops – in addition to making frequent purchases online. During recession, more shoppers became inclined to spend time hunting for bargains and as some traditional retailers either went out of business or shuttered down, retail space was freed up and was often filled by convenience stores, specially shops, and discounters.

A decade ago, CPG companies had only ‘a handful of’ sales channels to consider supermarkets, convenience stores, hypermarkets in advanced economies and traditional small and large retailers in emerging countries. Since, then various discounters have made significant inroads, including no frills, low variety outlets, such as Europe’s Aldi and Lidi, which sell a limited range of private lable grocery items in smaller stores and massive warehouse clubs, such as Costco and Sam’s club, which initially operated solely in the USA but are now expanding internationally. In addition, dollars stores, specialized retailers, and online merchants are having an impact on the CPG landscape. Economizing consumers have been pleasantly, surprised by the savings generated by spreading their business among multiple channels, as well as by the variety and product quality they find. The result has been greater demand for more product and brands, with different sizes, packaging and sales methods. At most CPG companies, SKUs are proliferating despite there being little increase in overall consumption. A better outcome can be seen at smaller food and beverage suppliers, which are benefitting from consumer demand for variety and authenticity. A recent report found that in the USA, small manufactures (with revenues of less than $1 billion) grew at twice the compound annual rate of large manufactures (with revenues of less than $3 billion) between 2009 and 2012.

Consumers media usage has also fragmented with the rise of digital content and the proliferation of online devices. Each channel – from the web, mobile and social sites for radio, TV, and print – has its own requirements, audience appeal and economics, needing specialized attention. But, at the same time, media campaigns need to be closely coordinated for effective consumer messaging.

Collectively, these shifts challenges the sway CPS companies manage their brand and business portfolios and call for a rethinking of their go-to-market approach, with an emphasis on analytics. Our work with INSEAD shows that among business leaders, applying analytics–especially for tracking consumer behavior and product and promotional performance–considered one of the most effective ways to improve results and outpace the competition. But it’s not just about insight. It’s also about using the insight wisely to determine how to manage costs. The more knowledgeable about customer needs and preferences a company is, the smarter and more focused it must be in managing its own economics to cost-effectively deliver both variety and value to be squeezed consumer.

6. The central theme of the given passage is

(1) The Shrinking Market
(2) Shift towards offering Luxury Goods to Consumers
(3) Products to offer consumers with squeezed packets
(4) To highlight products consumer by the middle class
(5) Gaining insight into changing consumer behavior towards CPGs

7. In the context of the passage, which of the following brands existed otherwise but is now being manifested in buying groceries as well?

(1) Consumer purchasing the same products for over a period of time.
(2) Consumer willing to purchase goods for a longer period of time.
(3) Consumers preferring luxury goods over regular goods.
(4) Consumers are more aware of their rights.
(5) Consumers prefer buying goods from a variety of stores.

8. Which of the following is most nearly the opposite in meaning to the word Depressing as used in the passage?

(1) Encouraging
(2) Sunny
(3) Doubtful
(4) Light
(5) Nil

9. As mentioned in the passage, CPG companies may have to reassess their present strategies of operating to

(A) retain their customers
(B) keep pace with changing consumer preference as they have access to multiple media channels
(C) make more cost-effective decisions.

(1) Only A
(2) B and C
(3) All of these
(4) Only C
(5) A and B

10. Which of the following is most nearly the opposite in meaning to the word Depressing as used in the passage?

(1) Encouraging
(2) Sunny
(3) Doubtful
(4) Light
(5) Nil

11. Which of the following is true in the context of the passage?

(1) In the USA, during the three year period after 2009, small manufacturing did not fare well as compared to their larger counterparts.
(2) Impact on dispensable incomes of people barely affects the CPG manufacturing industry.
(3) Post-tax wages, especially for the middle class, are one of the critical factors which have reduced spending behavior of consumers.
(4) The CPG have always been a favourite among consumers.
(5) None of the given options is true.

12. As mentioned in the passage, one of the most critical factors that aids in creating to the needs of consumers, is

(1) persuading then to purchase goods produced by the organization
(2) assess their requirements and appropriately plan to meet them
(3) offering them products that an organization regularly manufactures.
(4) concentrating only on being aware about changing presences of consumers
(5) None of the above

13. Which of the following is most nearly the same in meaning to the word above Shrinking as used in the passage?

(1) Developing
(2) Annoying
(3) Narrowing
(4) Wasting
(5) Rising

14. Which of the following is most nearly the same in meaning of the word Variegated as used in the passage?

(1) Diverse
(2) Composite
(3) Strong
(4) Narrow
(5) Valued

15. Which of the following is most nearly the opposite in meaning to the word Shuttered as used in the passage?

(1) Closed
(2) Retail
(3) Flourished
(4) Gratified
(5) Nearest

Click Here To Download Full Paper

DOWNLOAD IBPS PO Question Papers PDF

DOWNLOAD IBPS CLERK Question Papers PDF

DOWNLOAD ALL BANK EXAMS PAPERS E-BOOKS

Printed Study Material for IBPS PO Exam

IBPS PO (Probationary Officer) Exam Study Material

IBPS / SBI Special TX: 
General: 

(Papers) IBPS PO Exam Paper - 2015 "Reasoning Held on: 11-06-2015"

IBPS logo



(Papers) IBPS PO Exam Paper - 2015 "Held on: 11-06-2015"

:: Reasoning Ability ::



Directions (1-5): Study the information carefully and answer the given questions.

A, B, C, D, E, F, G and H are sitting around a circular area of equal distances between each other, but not necessary in the same order. Some of teh people are facing in the centre while some face outside. (i.e.in a direction opposite to the centre).

D sits third tothe right of B. E sits second to the left of B. Immediate neighbours of B face the same direction (i.e. if one neighbour faces the centre, the other neighbour also faces the centre and vice-versa). C sits second to the left of E. E faces the centre. F sits third to the right of C.G sits second to the left of H. H is not an immediate neighbour of B. G faces the same direction as D. Immediate neighbours of E faces opposite directions (i.e. if one neighbour faces the centre, the other neighbour faces outside and vice-versa.) Immediate neighbours of F face opposite directions (i.e. if one neighbour faces the centre, the other neighbour face outside and vice-versa.)

1. How man people in the given arrangement face the centre?

(1) Two
(2) Three
(3) One
(4) Four
(5) Five

2. Which of teh following is true regarding A as per the given sitting arrangement?

(1) H sits second to the left of A.
(2) A faces outsides.
(3) Only two people sit between A and B.
(4) C is one of the immediate neighbours of A.
(5) Only three people sit between A and G.

3. Four of the following five are alike in a certain way based on the given sitting arrangement and so form a group. Which is the one that does not belong to that group?

(1) F
(2) H
(3) B
(4) G
(5) D

4. What is E's postion with respect to H?

(1) Third to the left
(2) To the immediate left
(3) To the Immediate right
(4) Second to the right
(5) Third to the right

5. Who sits second to the left of G?

(1) H
(2) A
(3) B
(4) Other than those as options
(5) F

Directions (6-10): Study the given information carefully to answer the given questions.

A, B, C, D K, L and M live on seven different floors of a building but not necessarily in the same order. The lowermost floor of the building is numbered one, the one above that is numbered two and so on till the typmost floor is numbered seven.

Each one of them also likes dfferent games and Polo (but not necessarily in the same order). Only three people live between B and K. B lives on one of the floosrs above K. K does not live on the lowermsot floor. Only one person live between B and the one who likes Chess. The one who likes Polo lives on one of the even numbered floors above the one who likes Chess. Only two people live between M and the one who like Chess.

The one who likes snooker lives immediately above M. A lives immediately above L. A does not like Chess. The one who likes Ludo lives on one of the odd numbered floors below L. M does not like Ludo. D lives on one of teh floors above C. Only one person lives between the one who likes Cricket and the one who likes Hockey. D does not like Cricket. M does not like Badminton.

6. Which of the following game does B like?

(1) Snooker
(2) Ludo
(3) Polo
(4) Badminton
(5) Chess

7. Who amongst the following lives on the floor numbered 4?

(1) The one who likes Hockey
(2) The one who likes Chess
(3) A
(4) L
(5) B

8. Which of the following statements is true with respect to the given arrangement?

(1) Only two people live between K and M.
(2) The one who likes Hockey lives immediately above K.
(3) C likes Chess.
(4) C lives on an even numbered floor.
(5) None of the given options is true.

9. If all the people are made to sit in alphabetical order from top to bottom, the positions of how many people will remain unchanged?

(1) None
(2) Three
(3) Two
(4) One
(5) Four

10. Which of the following combinations is true with respect to the given arrangement?

(1) Polo-C
(2) Ludo-B
(3) Cricket-K
(4) Chess-L
(5) Snooker-A

11. A person starts walking from his office towards a partly hall. He walks for 30m towards East. He takes a 90° right turn and walks for 15m. He again takes a 90° right turn, and walks for another 20m. He then walks for 25m after taking a 90° left turn. Turning 90° towards his right he walks for 10m to reach the party hall. How far and in which direction is the party hall from his office?

(1) 40 m towards West
(2) 40 m towards South
(3) 45 m towards South
(4) 45 m towards North
(5) 40 m towards North

Directions (12-16): In these questions, two/three statements followed by two conclusions numbered I and II have been given. You have to take the given statements to be true even if they seem to be at variance from the commonly known facts and then decide which of the given conclusions logically follows from the given statements disregarding commonly known facts.
Give answer

(a) if only concklusion I is ture
(b) if only conclusion II is true
(c) if either conclusion I and II is true
(d) if both conclusions are true
(e) if neither conclusion I not II is true

12. Statements

Some winds are trophies.
Some trophies are cups.
No cup is a prize.

Conclusions:

(I) At least some cups are wins.
(II) All prizes being trophies is a possibility.

13. Statements

No layer is a coat.
All coats are deposits.
All deposits are sheets.

Conclusions:

(I) All coats are depostis.
(II) All deposits can never by layers.

14. Statemetns

Some flats are apartments.
No apartments is a hall.
Some halls are rooms.

Conclusions:

(I) At least some rooms are flats.
(II) No apartment is a room.

15. Statements

Some wins ae trophies.
Some trophies are cups.
No cup is a prize.

Conclusions :

(I) No trophy is a prize.
(II) All secrets are puzzles.

 

DOWNLOAD IBPS PO Question Papers PDF

DOWNLOAD IBPS CLERK Question Papers PDF

DOWNLOAD ALL BANK EXAMS PAPERS E-BOOKS

Click Here to Buy Printed Study Material for IBPS PO Exam

IBPS PO (Probationary Officer) Exam Study Materials

IBPS / SBI Special TX: 
Subject: 
General: 

(Papers) IBPS PO Exam Paper - 2015 "Held on: 31-10-2015" ::REASONING ABILITY::

IBPS logo


(Papers) IBPS PO Exam Paper - 2015 "Held on: 31-10-2015"

::REASONING ABILITY::


DIRECTIONS (Qs. 1-5) : Each of the questions below cionsists of a question and two statements numbered I and II given below it. You have to decide whether the data provided in the statements are sufficient to answer the question. Read both the statements and____

Give answer (a) If the data in statement I alone are sufficient to answer the questions, while the data in statement II alone are not sufficient to answer the question.
Give answer (b) If the data in statement II alone are sufficient to answer the question, while the data in statement I alone are not sufficient to answer the question.
Given answer (c) If the data either in statement I alone or in statement II alone are sufficient to answer the question.
Give answer (d) If the data even in both statement I and II together are not sufficient to answer the question.
Give answer (e) If the data in both statements I and II together are necessary to answer the question

1. How is ‘party’ coded in the language ?

I. ‘going to a party’ is coded as ‘la fa qu tu’ and ‘for a party’ is coded as ‘fa me tu’.
II. ‘start the party’ is coded as ‘tu co ra’ and ‘going to start’ is coded as ‘qu co la’.

2. On which day of the week is Priya definitely travelling ?

I. Amita correctly remembers that Priya is travelling after Tuesday but before Saturday. Rohit correctly remembers that Priya is travelling before Friday but after Monday.
II. Priya does not travel on a Friday.

3. How is P related to A ?

I. A is the daughter of M and the sister of S.
II. K is the son of P and the husband of M.

4. Four movies are screened from Monday to Thursday, (one on each day starting on Monday and ending on Thursday) viz - Crash, Social Network, Ice Age and Notting Hill. On which day is the movie Crash screened ?

 I. Social Network is screened on the last day.
II. Neither Ice Age nor Notting Hill are screened on Monday.

5. Who sits to the immediate right of Ayesha ?

I. Four friends Shradha, Tania, Vimla and Ayesha are sitting around a circular table facing the centre.
II. Shardha sits second to the right of Tania. Vimla is an immediate neighbour of Shradha.

DIRECTIONS (Qs. 6-10) : Study the following information to answer the given questions

A word and number arrangement machine when given an input line of words and numbers rearranges them following a particular rule. The following is an illustration of input
and rearrangement. (Single digit numbers are preceded by a zero. All other numbers are two digit numbers) Input : Good 18 to raise 02 12 money 28 for 57 charity 09.

Step I : to good 18 raise 02 12 money 28 for charity 09 57.
Step II : to raise good 18 02 12 money for charity 09 28 57.
Step III : to raise money good 02 12 for charity 09 18 28 57.
Step IV : to raise money good 02 for charity 09 12 18 28 57.
Step V : to raise money good for charity 02 09 12 18 28 57.
Step V : is the last Step of the arrangement of the above input as the intended arrangement is obtained.

DIRECTIONS (Qs. 6-7) : These questions are based on the following input: Input : always 19 give 21 84 for 62 14 worthy cause

6. Which of the following would be step III for the above input ?

(a) worthy give for always 19 14 cause 84 62 21.
(b) worthy give for always 14 19 cause 21 62 84.
(c) always give for worthy 19 14 cause 21 62 84.
(d) worthy give for always 19 14 cause 21 62 84.
(e) always give for cause 19 14 worthy 21 62 84.

7. How many steps would be needed to complete the arrangement for the above input ?

(a) VI
(b) V
(c) IV
(d) VII
(e) None of these

DIRECTIONS (Qs. 8-10) : These questions are based on the following input : Input : 50 62 tips on 67 how can 42 stay young 17 89 forever 03.

8. The following stands for which step of the arrangement ? young tips stay 50 on how can 42 17 forever 03 62 67 89.

(a) Step III
(b) Step V
(c) Step VI
(d) Step IV
(e) None of these

9. Which of the words/numbers below would be at the fifth position (from the right end) in Step V of the input ?

(a) forever
(b) 42
(c) 50
(d) young
(e) None of these

10. Which of the following would be the last step for the input?

(a) young tips stay on how forever can 03 17 42 50 62 67 89.
(b) young tips stay on how forever can 89 67 62 50 42 17 03.
(c) can forever how on stay tips forever 89 67 62 50 42 17 03.
(d) young tips stay on how forever can 03 17 42 50 67 62 89.
(e) can forever how on stay tips young 03 17 42 50 62 67 89.

DIRECTIONS (Qs. 11-13) : Read the followign information carefully and answer the quesitons that follow.

Ruling governments in the west are being punished by the voters for ever rising unemployment rates. Their parliament is abuzz with campaigns, marked by criticism of India as an outsourcing hub. India is seen by many in the west as a land of call centres and back offices with cheap labour that cost people in the west, their jobs.

11. Which of the following statements would weaken the argument given in the passage ?

(a) Outsourced jobs do not require highly skilled and qualified employees.
(b) Nearly 34% of the unemployed people would secure jobs in the west if outsourced jobs were in sourced by their organisations
(c) After suffering heavy losses in the elections the governments in the west are expected to change their decision on outsourcing to India.
(d) Outsourcing, a dynamic, two-way relationship has created jobs and growth in India as well as the west.
(e) Although outsourcing to India allowed many companies in west to focus on their core operations, they heavily compromised the quality and the standards of their back office jobs.

12. Which of the following can be a possible repercussion of the opposition to outsourcing in the west ?

(a) Jobs which are currently outsourced to India would be transferred to another country which in all probability would be China.
(b) Call centres and back offices employees in India would expect a salary at par with their western counterparts.
(c) In-sourcing of jobs by the west would render thousands of Indians unemployed.
(d) If in-sourced in the west itself, the companies would not be able to employ professionals and experts of same quality as available in India.
(e) The western companies which earlier outsourced to India would benefit financially as offices would not have to be set overseas.

13. Which of the following can be inferred? (An infernece is something which is not directly stated but can be inferred from the given facts)

(a) Unemployment in India is not as severe a problem as that in the west
(b) Employees working in the back offices and call centres in the west earn much more than their counterparts in India
(c) Developing countries such as China and India do not outsource their back office jobs at all to other countries
(d) Countries which do not outsource jobs do not face the problem of unemployment
(e) One of the main reasons for high unemployment rate in India is its clean of call centres and back offices which undertake outsourced work from the west

DIRECTIONS (Qs. 14-19) : Read the following information carefully and answer the questions that follow.

(K, L, M, P, Q, R, S and T are sitting around a square table in such a way that four of them sit at four corners of the square while four sit in the middle of each of the four sides. The ones who sit at the four corners face outside while those who sit in the middle of the sides face the centre of the table.) P sits third to the right of S. S faces the centre. Q sits third to the left of M. M does not sit in the middle of the sides. Only one person sits between Q and R. R is not an immediate neighbour of M. T faces the centre. K is not an immediate neighbour of R.

14. What is position of M with respect to L ?

(a) Third to the right
(b) M and L sit diagonally opposite to each other
(c) Second to the right.
(d) Second to the left
(e) Fifth to the right

15. Who sits exactly between Q and R ?

(a) T
(b) P
(c) K
(d) M
(e) S and K

16. Which of the following pairs represents the persons seated in the middle of the sides who face each other ?

(a) S, Q
(b) K, L
(c) M, P
(d) R, T
(e) T, Q

17. Who amongst the following sits between R and K when counted in anti-clockwise direction from K ?

(a) No one sits between R and K as R and K are immediate neighbours of each other
(b) S, P and
(c) P and Q
(d) L and R
(e) M, S and T

18. If K is made to face the opposite direction, who would sit to his immediate right ?

(a) R
(b) Q
(c) P
(d) T
(e) S

19. Four of the following five are alike in a certain way and so form a group. Which is the one that does not belong to that group ?

(a) L
(b) M
(c) K
(d) P
(e) R

DIRECTIONS (Qs. 20-23) : Read the following information and five statements given below it carefully and answer the questions which follow.

Exodus from rural areas to the urban hubs in search of job opportunities has now declined to nearly twenty-six percent of what it was at the turn of 21st century. (A) Since the last decade, the rural economy has transformed itself into a bankable, profit making and commercially viable venture. (B) Job opportunities differ in urban and rural areas (C) The load on infrastructure and resources in the urban areas which had remained unmanageable for a long time has been eased a little since the last decade. (D) This trend of reverse migration which was seen only in developed countries till now has entered the scenes of developing nations as well (E) According to a recent report more than eighty
percent of the professionals having roots in rural areas prefer to work in urban cities rahter than their home villages.

20. Which of the statements numbered (A), (B), (C), (D) and (E) mentioned above respresents an effect of the given information most appropriately ?

(a) B
(b) C
(c) D
(d) C
(e) Either D or E

21. Which of the statements numbered (A), (B), (C), (D) and (E) mentioned above represents a cause of the given information most appropriately ?

(a) E
(b) D
(c) C
(d) B
(e) A

22. Which of the statements numbered (A), (B), (C), (D) and (E) mentioned above represents an assumption most appropriately ? (An assumption is something supposed or taken for granted)

(a) A
(b) B
(c) C
(d) Both A and C
(e) D

23. Which of the statements numbered (A), (B), (C), (D) and (E) mentioned above would weaken / contradict the facts presented in the paragraph ?

(a) A
(b) B
(c) C
(d) D
(e) E

DIRECTIONS (Qs. 24) : Question below is followed by three arguments numbered I, II and III. You have to decide which of the argument(s) is/are “strong” and indicate your answer accordingly.

24. Should the Government restrict use of electricity for each household depending upon the requirement?

Arguments :

I. Yes, this will help government tide over the problem of inadequate generation of electricity.
II. No, every citizen has right to consume electricity as per their requirement as they pay for using electricity.
III. No, the Government does not have the machinery to put such a restriction on use of electricity.

(a) Only I is strong
(b) Only II is strong
(c) Only I and II are strong
(d) Only II and III are strong
(e) All I, II and III are strong

25. Statement : Many patients suffering from malaria were administered anti-malarial drug for a week. Some of them did not respond to the traditional drug and their condition deteriorated after four days. Which of the following would weaken the findings mentioned in the above statement?

(a) Those patients who responded to the traditional drugs and recovered were needed to be given additional doses as they reported relapse of symptoms.
(b) The mosquitoes carrying malaria are found to be resistant to traditional malarial drugs.
(c) Majority of the patients suffering from malaria responded well to the traditional malarial drugs and recovered from the illness.
(d) Many drug companies have stopped manufacturing traditional malarial drugs.
(e) None of these

Printed Study Material for IBPS PO Exam

IBPS PO (Probationary Officer) Exam Study Materials

IBPS / SBI Special TX: 
General: 

Pages

Subscribe to RSS - user8's blog